Download Case 7-09 Electrolytes Patient ID 41-year

Document related concepts

Public health genomics wikipedia , lookup

Disease wikipedia , lookup

Fetal origins hypothesis wikipedia , lookup

Artificial pancreas wikipedia , lookup

Epidemiology wikipedia , lookup

Maternal physiological changes in pregnancy wikipedia , lookup

Prenatal testing wikipedia , lookup

Epidemiology of metabolic syndrome wikipedia , lookup

List of medical mnemonics wikipedia , lookup

Multiple sclerosis research wikipedia , lookup

Transcript
Case 7-09
Electrolytes
Patient ID
41-year-old female
Patient Location
Medical Outpatients
Clinical Notes on Request Form
Polyuria and polydipsia. Cause?
Case Details
Early morning spot urine
Osmolality
611 mmol/kg
Additional Information
Other laboratory results taken at the same time:
Serum
Sodium
145 mmol/L (134–146)
Potassium
4.3 mmol/L (3.4–5.0)
Urea
8.5 mmol/L (3.0–8.0)
Creatinine
84 µmol/L
(45–90)
Glucose
5.1 mmol/L (3.5–5.4)
Ionised Calcium
1.23 mmol/L (1.12–1.32)
Osmolality
302 mmol/kg (275–295)
Suggested Comment
Suboptimal urine concentration in the setting of raised serum osmolality suggests possible partial diabetes
insipidus. Suggest document degree of polyuria and consider fluid deprivation challenge with DDAVP
administration to differentiate central from nephrogenic causes. Further endocrine review may be appropriate
depending upon findings.
Rationale
Timed urine collection helps document the degree of polyuria, and by definition should be >2.5–3 L/day. The
present case shows raised serum osmolality, which makes primary polydipsia less likely. Urine is inadequately
concentrated as one would expect a urine osmolality of >750 mmol/kg with this serum osmolality. This
suggests (partial) diabetes insipidus, of either central or nephrogenic origin. The former is caused by
inadequate antidiuretic hormone (ADH) production while the latter is due to inadequate ADH action on the
renal tubules.
Formal fluid deprivation protocol (1) can help to resolve differential diagnosis with lack of further urine
concentration following DDAVP in nephrogenic cases. Hypokalaemia, hyperglycaemia and renal impairment
have been excluded as causes, although drugs, particularly lithium, may be contributory. Endocrine review
with further pituitary studies may be appropriate depending on the findings.
Reference
1. Baylis PH. Investigation of suspected hypothalamic diabetes insipidus. Clin Endocrinol (Oxf)
1995;43:507-10.
PREFERRED KEY WORDS
Consistent with dehydration
Consider diabetes insipidus
Suggest water deprivation test ± DDAVP
Inadequately concentrated urine
?Nephrogenic diabetes insipidus
?Central diabetes insipidus
Suggest endocrinology consultation
?Drug therapy
?Lithium Rx
LESS RELEVANT KEY
WORDS
Serum osmolality raised
?Diuretic use
Suggest urine Na/electrolytes
Slightly increased urea
Suggest 24 h urine collection
Normal serum Na/glu/Ca/K
Diabetes mellitus excluded
Suggest ADH
No osmolar gap
?Renal disease
?Ethanol
High normal sodium
Normal kidney function
No evidence 1°/psychogenic polydipsia
Normal biochemical profile
Suggest urine MC&S
Creatinine upper end reference range
Suggest eGFR
Consider trial DDAVP
Renal disease
UNACCEPTABLE KEY
WORDS
Appropriate concentration of urine
?Osmotic diuresis
No evidence of diabetes insipidus
Suggest assess patient fluid status
Suggest repeat tests
1°/psychogenic polydipsia
Suggest thyroid studies
?Salt and water overload
Assess risk/investigate diabetes mellitus
Suggest other tests
Elevated urine osmolality
Suggest CNS imaging/investigation
Suggest urine urea
?SIADH
Increased osmolar gap
Suggest ACTH, cortisol
?Pregnant
Suggest urine sodium
Serum osmolality normal
?Hypoaldosteronism with salt wasting
Suggest aldosterone:renin ratio
?Fanconi syndrome
?Hyperthyroidism
No comment
?Hyperalimentation
Avoid nicotine, alcohol, caffeine
Suggest urine drug screen
?Unmeasured solute
SIADH unlikely
Suggest abdominal imaging
No polyuria by urine:serum osmolality
Normal urea, creatinine
Case 8-05
Electrolytes
Patient ID
51-year-old male
Patient Location
Emergency Department
Clinical Notes on Request Form
Acute confusional state. On risperidone.
Case Details
Plasma
Sodium
Potassium
Creatinine
Urea
Osmolality
108 mmol/L
4.1 mmol/L
80 µmol/L
2.0 mmol/L
227 mmol/kg
(136–146)
(3.5–5.5)
(50–110)
(2.7–7.8)
(280–300)
Urine
Sodium
Potassium
Osmolality
12 mmol/L
8 mmol/L
70 mmol/kg
Suggested Comment
Severe hyponatraemia and a hypo-osmotic plasma with appropriately dilute urine and no evidence of salt
wasting. This pattern is in keeping with primary (psychogenic) polydipsia with water intoxication. While
risperidone is not necessarily the cause of the polydipsia, an alternative atypical antipsychotic may be more
effective in improving psychogenic polydipsia.
Rationale
Hyponatraemia is classified based on hydration status and plasma osmolality. The kidney’s response allows
assessment of whether the cause is renal salt wasting. In this instance renal sodium loss is close to the <10
mmol/L usually applied to exclude renal salt loss and certainly below the >20 mmol/L used to rule-in salt
wasting. The urine is appropriately almost maximally dilute, and is not suggestive of SIADH or dehydration
(1). The results are most consistent with polydipsia, most commonly due to psychiatric illness and frequently
affecting chronic schizophrenic patients.
While antipsychotics are a well-recognised cause of hyponatraemia (secondary to SIADH), the literature
surrounding their effects on polydipsia is controversial. ‘Typical’ antipsychotics have been associated with
aggravating polydipsia, while atypical antipsychotics have been reported to be useful in treating these patients
(2). Risperidone has not been shown to be clearly effective in this group. At least one case in the literature cites
risperidone as the likely cause (3).
References
1. Burtis CA, Ashwood ER, Bruns DE (eds). Tietz Textbook of Clinical Chemistry and Molecular
Diagnostics. 4th Edition. St Louis: Elsevier Saunders; 2006. pp. 1751-53.
2. Bersani G, Pesaresi L, Orlandi V, et al. Atypical antipsychotics and polydipsia: a cause or a treatment?
Hum Psychopharmacol 2007;22:103-7.
3. Kar N, Sharma PS, Tolar P, et al. Polydipsia and risperidone. Aust N Z J Psychiatry 2002;36:268-70.
PREFERRED KEY WORDS
Severe hyponatraemia
Low urine osmolality
Low urine sodium
Consistent with polydipsia
?Psychogenic polydipsia
Suggests water intoxication
Secondary water intoxication
Acute water overload
?Polydipsia
?Due to risperidone
Urine appropriately dilute
Not suggestive of SIADH
Evaluate total body water
Dilutional hyponatraemia
Suggest withdrawal risperidone
LESS RELEVANT KEY
WORDS
Low plasma osmolality
Hyponatraemia
?Other drugs
Confusion due to hyponatraemia
?Extra-renal loss
Suggest thyroid function tests
Due to risperidone
Low plasma urea
Action required
Patient history required
Cerebral oedema risk
Suggest fluid restriction
Suggest glucose
Monitor results
Suggest liver function tests
Central pontine myelinolysis risk with
rapid correction
Risk of death
Check results repeated
Patient likely requires hypertonic saline
Renal function tests normal
Check protein/lipids
Risk rhabdomyolysis during treatment
Low plasma sodium
?Patient on fluid replacement
Suggest drug screen
Contact Pathologist
?Beer potomania
UNACCEPTABLE KEY
WORDS
?SIADH
Suggest water deprivation test
?Adrenal insufficiency
?Hypothyroidism
Suggest cortisol / synacthen stimulation
test
?Cardiac/liver failure
?Renal loss
?Hyperglycaemia
Urine not maximally dilute
Suggest ADH
?Alcoholism
?Disease causing oedema
?Diabetes insipidus
Check pituitary function
Suggest serum/urine myoglobin
?Patient non-compliance
?Increased sodium loss
Sodium depletion unlikely
Suggest renal function tests
?Pseudohyponatraemia
?Low salt intake
?Renal failure
?Reset osmostat
Case 8-09
Electrolytes
Patient ID
78-year-old man
Patient Location
Nursing Home
Clinical Notes on Request Form
Demented, percutaneous endoscopic gastrostomy (PEG) fed
Case Details
Sodium
Potassium
Chloride
Bicarbonate
Urea
Creatinine
169 mmol/L
3.6 mmol/L
121 mmol/L
37 mmol/L
18.0 mmol/L
90 µmol/L
(135–145)
(3.5–5.0)
(98–107)
(21–32)
(2.0–8.5)
(60–110)
Suggested Comment
Profound hypernatraemia with raised urea/creatinine ratio is suggestive of marked dehydration. Suggest urgent
review of clinical volume status and fluid balance.
Rationale
Marked hypernatraemia with elevated urea to creatinine ratio is consistent with dehydration, specifically water
depletion. Percutaneous endoscopic gastrostomy (PEG) feeding tubes are increasingly used for patients who
cannot maintain adequate nutrition with oral intake. They include patients with neurological disorders, other
conditions where swallowing is impaired, or there is obstruction of the upper gastrointestinal tract.
Artificial feeding may cause metabolic problems, including deficiencies or excess of fluid, electrolytes,
vitamins and trace elements. Hypernatraemia is usually due to inadequate water intake. Loss of water in excess
of sodium may also be contributory. It may develop rapidly in patients who are unable to experience or
communicate their thirst, and especially in institutionalised subjects, who are reliant on caregivers to maintain
their fluid intake.
Reference
1. Gault MH, Dixon ME, Doyle M, et al. Hypernatremia, azotemia, and dehydration due to high-protein
tube feeding. Ann Intern Med 1968;68:778-91.
PREFERRED KEY WORDS
Consistent with marked dehydration
Marked hypernatraemia
?Inadequate water intake
?Hypovolaemic hypernatraemia
Increased urea/creatinine ratio
?Dehydration
Water/hypotonic fluid depletion
Check hydration status
Review feed composition
Suggest fluid replacement
?Insufficient water in PEG feed
Suggest IV fluid replacement
Urgent treatment required
Check urine volume
LESS RELEVANT KEY
WORDS
Elevated urea
Hypernatraemia
Elevated bicarbonate
Suggest urine/plasma osmolality
Elevated chloride
?Metabolic alkalosis
?Diarrhoea/stomal loss
?Vomiting
Consider urine sodium
?Diabetes insipidus
Suggest urine electrolytes
Suggest glucose estimation
?Increased salt intake
?Diuresis
?Pre-renal renal failure
Suggest clinical review
Suggest blood gas analysis
Consistent with tube feeding syndrome
Low normal potassium level
?Skin/respiratory water loss
?Secondary hyperaldosteronism
Excessive salt intake is rare
?Hyperaldosteronism
Suggest repeat urea and electrolytes
Haemoconcentration
?Head injury
?Potassium depletion
Monitor results during fluid treatment
?Diuretic therapy
Exclude hypertonic fluid therapy
?Gastrointestinal bleed
?Loss of thirst reflect
Risk coma/death
?Meningitis/meningoencephalitis
?Creatinine low due to muscle mass
Suggest urine creatinine
?Malnourished
?Saline-responsive metabolic alkalosis
Artefact unlikely
Suggest review fluid management
?Mineralocorticoid Rx
UNACCEPTABLE KEY
WORDS
?Drip arm
?Renal failure
?Catabolism
?Cushing’s disease
Suggest occult blood
No comment
Suggest cortisol/aldosterone
?Compensated respiratory acidosis
Suggest CK
?Rhabdomyolysis
Case 9-07
Electrolytes
Patient ID
55-year-old male
Patient Location
General Practice
Clinical Notes on Request Form
Increased blood pressure
Case Details
Sodium
Potassium
Chloride
Bicarbonate
Urea
Creatinine
Cholesterol
Triglyceride
146 mmol/L
3.6 mmol/L
106 mmol/L
29 mmol/L
5.1 mmol/L
100 µmol/L
5.8 mmol/L
1.4 mmol/L
(135–145)
(3.5–5.0)
(96–109)
(23–32)
(3.5–7.5)
(40–120)
(3.9–5.5)
(0.6–2.0)
Suggested Comment
In the setting of borderline hypokalaemia and high-normal bicarbonate, consider possible secondary causes of
hypertension. Measurement of renin and aldosterone might be helpful and further investigations of
mineralocorticoid excess, if clinically indicated. Suggest repeat full fasting lipid profile and evaluate in context
of absolute cardiovascular risk status.
Rationale
Borderline hypokalaemia and high-normal bicarbonate in the setting of hypertension should prompt review for
secondary causes such as diuretic therapy. Renin and aldosterone would be appropriate to investigate possible
mineralocorticoid excess. This should be considered in patients with blood pressures of >160 mmHg (systolic
and >100 mmHg (diastolic), drug-resistant hypertension, hypertension with: spontaneous hypokalaemia or
diuretic-induced hypokalaemia, adrenal incidentaloma, family history of early-onset hypertension or
cerebrovascular accident at a young age (<40 years), and patients with first degree relatives diagnosed with
primary aldosteronism.
Investigations for Cushing’s syndrome would depend on clinical index of suspicion. Full fasting lipid profile
should be repeated and any intervention determined by the absolute cardiovascular risk status, which depends
on full clinical evaluation and cannot be assigned on the basis of the results alone. It is not appropriate to
comment on any possible drug intervention.
Reference
1. Chao CT, Wu VC, Kuo CC, et al. Diagnosis and management of primary aldosteronism: an updated
review. Ann Med 2013;45:375-83.
PREFERRED KEY WORDS
Mild hypernatraemia
High normal bicarbonate
Low normal potassium
?1°/2° hyperaldosteronism
Suggest renin/aldosterone
?Cushing’s syndrome
Suggest mid-stream urine test
Suggest further investigations
Assess cardiovascular disease risk factors
Cholesterol and triglyceride insufficient
to assess cardiovascular disease risk
Assess 2° causes hyperlipidaemia
Review lifestyle associated risk factors
Suggest fasting glucose
Mildly elevated cholesterol
Suggest fasting lipid profile
Suggest HDL/LDL
?Drugs
?Diuretic therapy
LESS RELEVANT KEY
WORDS
Hypertension noted
Increased risk cardiovascular disease
Assess 2° causes hypertension
?Diabetes mellitus
Suggest TSH
Suggest repeat testing
Suggest cortisol/ACTH
Consider drug therapy if clinically
indicated
Exclude liquorice ingestion
eGFR=67 mL/min/1.73m2
?Phaeochromocytoma
Suggest liver function tests
Suggest repeat urea and electrolytes
Triglycerides within normal range
Hypertension probably idiopathic
Patient most likely fasting
Relative hypochloraemia
Normal bicarbonate
Normal urea
Moderately elevated cholesterol
Suggest 24 hour urine cortisol
Suggest baseline liver function tests
Suggest fT4
?Renal artery stenosis
Review history of familial
hypercholesterolaemia
Proceed to glucose tolerance test if
glucose ≥6.1 mmol/L
Mild renal disease not excluded
Suggest calcium
eGFR=72 mL/min/1.73m2
?Stage II chronic kidney disease
Hypokalaemic metabolic alkalosis
?Dehydration
Suggest plasma metanephrines
UNACCEPTABLE KEY
WORDS
Suggest microalbumin
Suggest urate
Suggest angiotensin
Suggest urine and serum osmolality
National Heart Foundation target for high
risk total cholesterol <4.0 mmol/L
Chronic renal failure excluded
Suggest ECG
Treat cardiovascular risk factors
aggressively
No comment
Normal electrolytes, urea, creatinine
Case 10-02
Electrolytes
Patient ID
32-year-old woman
Patient Location
Emergency Department
Clinical Notes on Request Form
Marked weakness – poor diet, but no anorexia; denies laxatives and diuretics use.
Case Details
Sodium
Potassium
Chloride
Bicarbonate
Urea
Creatinine
eGFR
Calcium
Magnesium
Phosphate
Total protein
Albumin
Bilirubin
ALT
GGT
ALP
TSH
fT4
139 mmol/L (134–146)
1.4 mmol/L (3.4–5.0)
110 mmol/L (98–108)
15 mmol/L
(22–32)
6.5 mmol/L (3.0–8.0)
118 µmol/L (45–90)
49 mL/min/1.73m2 (>60)
2.44 mmol/L (2.15–2.60)
1.26 mmol/L (0.70–1.10)
0.42 mmol/L (0.80–1.40)
83 g/L
(60–80)
41 g/L
(35–50)
10 μmol/L
(<20)
92 U/L
(<35)
84 U/L
(<40)
104 U/L
(35–135)
20.6 pmol/L (0.4–4.0)
9.0 mU/L
(9.0–19.0)
Additional Information
Hb
157 g/L
WCC
23.5 x109/L
Platelet
528 x109/L
Hct
0.44
MCV
91 fL
MCH
32 pg
(115–160)
(4.0–11.0)
(150–400)
(0.37–0.47)
(80–100)
(27–32)
Suggested Comment
Severe hypokalaemia and hypophosphataemia with a normal anion-gap metabolic acidosis and renal
impairment. The differential includes renal tubular acidosis (RTA, distal or proximal), and acute diarrhoea or
laxative abuse. Hypothyroidism, leucocytosis and thrombocytosis and raised ALT noted. Suggest blood gases,
urine pH, urine ammonia, and electrolytes. Possible muscle source for ALT - suggest measurement of CK to
confirm.
Rationale
This complex case involves a 32-year-old woman with profound hypokalaemia and a normal anion gap
metabolic acidosis. Additionally she has moderate hypophosphataemia, hypermagnesaemia, renal impairment,
hypothyroidism, a leucocytosis/thrombocytosis and a raised ALT. The comment should focus on the major
abnormality (severe hypokalaemia) and should discuss the differential diagnoses of hypokalaemic acidosis
rather than the much broader differential of hypokalaemia. Renal tubular acidosis (RTA, distal or proximal),
acute diarrhoea, and dialysis treatment are all possibilities. Carbonic anhydrase inhibitors or transplantation of
the ureters into the colon should be excluded also.
Further investigations should include blood gases, urine pH (>5.5 in distal RTA; <5.5 in proximal RTA), urine
ammonia (<33 μmol/min in distal RTA; >33 μmol/min in proximal RTA), electrolytes, and a laxative screen if
suspicion remains. The urine pH and ammonia should be measured when patient is in acidosis. Muscle is the
possible cause of the raised ALT; a CK measurement would be useful to confirm this. Recommend repeat
thyroid function tests after resolution of the acute episode would be useful.
PREFERRED KEY WORDS
LESS RELEVANT KEY
WORDS
UNACCEPTABLE KEY
WORDS
Marked hypokalaemia
Moderate hypophosphataemia
Acidosis
Normal anion gap metabolic acidosis
Hypokalaemic metabolic acidosis
Renal tubular acidosis
?Renal potassium loss
?GI loss ?diarrhoea ?vomiting
?Laxative abuse
Meds review ?Carbonic anhydrase
inhibitors
Suggest blood gas analysis
Suggest urine/plasma pH
Suggest urine K, pH, PO4, Mg, Ca, anion
gap
?Drug use/screen
Hypothyroidism
Repeat thyroid function tests after illness
settled
Potassium replacement required
Hypokalaemia with associated myopathy
Suggest serum CK/AST
?Ureteral diversion and/or obstruct
?Fanconi’s syndrome
Suggest urine amino acids
Renal impairment
Suggest glucose/insulin/glucose tolerance
test/urine glucose
Hepatocellular injury
?Autoimmune disease ?Hashimoto’s
Suggest thyroid antibodies
Hyperchloraemia
?Sepsis, suggest further investigation
?Diuretic use/abuse
Leucocytosis and thrombocytosis
Hypermagnesaemia
?Malnutrition
?Antacids use/abuse - high magnesium
Exclude urinary tract infection/
microscopy, culture and sensitivity
?Lithium - suggest levels
?Diabetic ketoacidosis/insulin
Physician consultation required - urgent
Suggest urine protein
?Hypokalaemic periodic paralysis
Hypothyroid hypokalaemic periodic
paralysis
Sick euthyroid syndrome
Suggest urine uric acid/PO4/NH4
Faecal electrolytes/Mg confirm high Mg
Possible refeeding syndrome
Infection/allergy/inflammation - suggest
CRP
NH4Cl loading test for renal tubular
acidosis
Repeat renal function tests
Potassium redistribution to cells insulin/B-agonists
Phosphate supplement required
Hypokalaemia
Toluene abuse
Exclude artefactual potassium reduction
Exclude transient cause of low potassium
?Respiratory alkalosis
Exclude hypertension
Suggest 24 h urine cortisol/creatinine/K
?Past treatment for hyperthyroidism
?Respiratory alkalosis
?Carcinoma ?villous tumour
No evidence of poor diet
Suggest urine protein
Marked hypophosphataemia
?Ethanol- induced liver dysfunction
Suggest cortisol/ACTH
?Malignancy
?Wilson’s disease, investigate further
?Adrenal insufficiency
Suggest cortisol/synacthen test
?Cushing’s syndrome
?Secondary hyperthyroidism
?Steroids
?Possible hyperparathyroid
Marked hyponatraemia
?Pituitary adenoma with 2°
hypothyroidism
?Excessive liquorice ingestion
Suggest HIV screen
Suggest TRH test for differential
diagnosis
Suggest B12/folate and iron
?Liddle’s/Bartter’s syndrome
Normal magnesium level
Suggest TSH with different method
?heterophile antibodies
Suggest serum/urine electrophoresis
Suggest autoimmune screen - ANA/ENA
Suggest renin/aldosterone
?Hyperaldosteronism
Suggest serum electrolytes, Mg, Ca
Suggest urine/plasma osmolality
?Light chains
Case 14-10
Electrolytes
Patient ID
78-year-old woman
Patient Location
General Practice
Clinical Notes on Request Form
Polyuria.
Case Details
Sodium
Potassium
Chloride
Bicarbonate
Urea
Creatinine
eGFR
Serum Osmolality
Urine Osmolality
146 mmol/L (136–145)
3.9 mmol/L (3.5–5.1)
104 mmol/L (98–107)
31 mmol/L
(22–29)
4.8 mmol/L (3.5–7.2)
86 µmol/L
(44–80)
55 mL/min/1.73m2 (>90)
299 mOsm/kg (280–300)
166 mOsm/kg (40–1400)
Suggested Comment
The inappropriately dilute urine for the given serum osmolality may indicate diabetes insipidus (DI).
Recommend serum calcium measurement and a review of medications for possible causes, and consider
referral for endocrinology review.
Rationale
Diabetes insipidus (DI), the inability to concentrate urine, is an important differential diagnosis in the
investigation of polyuria. A high urine volume coupled with low urine osmolality measured in the setting of a
supervised or overnight water deprivation test is pathognomonic. A low osmolality (<200 mOsm/kg) in a
random urine sample is unhelpful unless the serum osmolality at the same time is >295 mOsm/kg, indicating
relative dehydration. By contrast a high urine osmolality >700 mOsm/kg in a random urine sample can be
useful to exclude DI, often suggesting urinary frequency rather than true polyuria.
Psychogenic polydipsia, the main differential diagnosis for polyuria typically presents with low or low-normal
plasma osmolality, as the primary problem here is excessive water intake. Causes of DI may be central
(antidiuretic hormone (ADH) deficiency) e.g. head trauma, pituitary disease, or nephrogenic (ADH resistance)
e.g. hypercalcaemia, lithium therapy. Response to exogenous ADH distinguishes these two major forms.
References
1. Bichet DG. Diagnosis of polyuria and diabetes insipidus. In: UpToDate, Sterns RH and Emmett M
(Eds), UpToDate, Waltham, MA; 2013.
2. Makaryus AN, McFarlane SI. Diabetes insipidus: diagnosis and treatment of a complex disease. Cleve
Clin J Med 2006;73:65-71.
PREFERRED KEY WORDS
High normal serum osmolality
Low urine osmolality
Inappropriate dilute urine
Inappropriate diuresis
Urine osmolality consistent with water
diuresis
Excessive water loss
?Diabetes insipidus (DI)
Suggest serum/plasma calcium
Review drugs/medications
History of polyuria noted
?Lithium therapy
?Cranial/central or nephrogenic DI
?Medication related
Suggest (endocrine) referral
Dehydration
?Hypercalcaemia
Discuss fluid intake with patient
LESS RELEVANT KEY
WORDS
Suggest water deprivation test
Borderline hypernatraemia
24 h urine volume to confirm polyuria
Decreased eGFR
?Diuretic therapy related
Suggest urinalysis
Desmopressin to differentiate
cranial/nephrogenic DI
Suggest ADH
Mildly elevated creatinine
Mildly elevated bicarbonate
Mild (metabolic) alkalosis
Urine osmolality < serum osmolality
?Osmotic diuresis
?Renal ADH resistance
Phone GP
?Metabolic
?Iatrogenic
?Urinary tract infection
Review family history
Suggest referral to nephrologist
NOT SUPPORTED KEY
WORDS
?Primary polydipsia
Creatinine consistent with stage 3 renal
disease
?Chronic kidney disease (CKD)/ renal
disease
Suggest imaging
?Cushing’s/Conn’s
Repeat early morning urine osmolality
Suggest aldosterone/renin ratio
?Diabetes mellitus
Suggest serum electrophoresis
?Thyrotoxicosis/suggest thyroid function
tests
Normal osmolar gap
Suggest serum cortisol
?Liquorice use
Suggest 24 h urine cortisol
Suggest comprehensive biochem profile
Suggest full blood count
Suggest hospital referral
?High water intake
Suggest HbA1C
Primary water deficit
Suggest ACTH
Suggest microalbumin
Stage II CKD
MISLEADING KEY WORDS
?Age related
Suggest repeat in 3–6 months
Inappropriate ADH secretion
Suggest renal biopsy
Case 12-04
Acid-Base
Patient ID
75-year-old man
Patient Location
Emergency Department
Clinical Notes on Request Form
Renal failure. Dizzy.
Case Details
Blood Gases
Sample type: venous
pH
7.22
(7.35–7.45)
pCO2
54 mmHg
(35–45)
pO2
75 mmHg
(80–110)
HCO3
22 mmol/L
(22–30)
Base Excess -6 mmol/L
(-3/+3)
O2 Sat.
92 %
Na
121 mmol/L (135–148)
K
7.3 mmol/L (3.5–5.3)
iCa
1.05 mmol/L (1.13–1.32)
Note: all reference intervals shown are for arterial blood only.
Suggested Comment
Critical electrolyte and calcium results, while consistent with renal failure, require confirmation by repeat
sample before initiating treatment. Serum creatinine and urea should also be measured urgently. Provided the
patient is not in circulatory shock, the pH reliably indicates an acidaemia; most likely metabolic but also with a
respiratory component given the high pCO2. Assessment of oxygenation is unreliable and if required, should
be assessed by pulse oximetry and arterial blood gas analysis.
Rationale
There is growing evidence that, in certain situations, venous blood gas analysis provides adequate and reliable
information to allow one to avoid the more painful and potentially dangerous practice of arterial blood
sampling. Venous sampling is particularly useful in diabetic ketoacidosis and uraemic patients. There is
currently insufficient evidence to support its use in compromised circulatory states (e.g. arrest/ shock) and
mixed acid-base disorders as the relationship between arterial and venous blood gas parameters may change in
these situations.
pH and bicarbonate results are virtually interchangeable with arterial values, thus allowing for acid-base
assessment. pCO2 values are generally 6 mmHg higher in venous blood. At the diagnostic cut-off of 45 mmHg
(i.e. the arterial blood cut-off), venous pCO2 has high sensitivity (nearly 100%) but poor specificity (56%) as a
screening test for arterial hypercarbia. pO2 values are less than half of those in arterial blood and consequently
no comment regarding hypoxia should be made from venous samples. Hyperkalaemia due to haemolysis is
always possible but less of a concern here given the pattern of abnormalities and the clinical context.
Communication should focus clinical staff on immediate confirmation of these abnormal results before urgent
medical treatment.
References
1. Kelly AM. Review article: can venous blood gas analysis replace arterial in emergency medical care.
Emerg Med Australas 2010;22:493-8.
2. Verma AK, Roach P. The interpretation of arterial blood gases. Aust Prescr 2010;33:124-9.
PREFERRED KEY WORDS
Venous blood gas sample
Critical hyperkalaemia and
hyponatraemia
Severe hyperkalaemia
Severe hyponatraemia
Acidaemia
?Low Ca 2° to renal failure
Consistent with mixed respiratory and
metabolic acidosis
Consistent with renal failure
Na, K, Ca consistent with renal failure
Suggest repeat electrolytes/Ca
Suggest renal function tests
?Respiratory disease/pneumonia/
pulmonary oedema
Venous pO2 not reliable
Urgent medical attention required
LESS RELEVANT KEY
WORDS
NOT SUPPORTED KEY
WORDS
Recollect arterial blood sample
Metabolic acidosis
Suggest glucose
?High potassium due to haemolysis,
excess cold
?Drugs/medications
?Diabetic ketoacidosis
Suggest test for adrenocortical
insufficiency
?Respiratory failure
Respiratory acidosis
Reference intervals are for arterial blood
Low pH and high pCO2
Suggest urine K, Na, protein
?Diuretics/spironolactone
Suggest anion gap
Suggest serum and urine osmolality
?O2 therapy
Bicarbonate loss
?Adrenal insufficiency
Suggest CXR
Hypercarbia
?Spurious hyperkalaemia
Hyperkalaemia caused by acidosis
Acidosis
Acute respiratory acidosis
High K, low Ca
Mild hypocalcaemia
Suggest cardiac monitoring
?Clinical symptoms due to low Na
?Lactic acidosis
No respiratory compensation
Suggest nephrology review
Hyponatraemia due to renal failure
Severe acidosis
No comment
?Dizziness due to hypocalcaemia
Mixed acid base disorder
Consistent with arterial sample
Suggest dialysis
Respiratory compensation
Suggest PTH/Ca/Mg/Vitamin D/ALP
?Specimen contamination
Suggest urine pH
?Renal tubular acidosis
Difficult to assess respiratory component
Lack of renal composition
Low Ca due to critical illness
Venous blood gas difficult to comment
Suggest renal ultrasound
SIADH
?Chronic respiratory acidosis
Acute on chronic respiratory acidosis
Metabolic compensation
Mixed metabolic acidosis and respiratory
alkalosis
Suggest Cl for RTA
MISLEADING KEY WORDS
?Cerebral disease
O2 status acceptable for elderly
Hypoxia and cyanosis
Case 13-06
Acid-Base
Patient ID
52-year-old woman
Patient Location
Emergency Department
Clinical Notes on Request Form
No information provided
Case Details
Sodium
Potassium
Chloride
Bicarbonate
Urea
Creatinine
150 mmol/L
3.0 mmol/L
127 mmol/L
13 mmol/L
24.9 mmol/L
102 µmol/L
(137–145)
(3.5–4.9)
(100–109)
(22–32)
(2.7–8.0)
(50–100)
Previous renal surgery
Venous blood gas analysis
pH
7.22
pO2
31 mmHg
pCO2
29 mmHg
Calc. Bicarb 12 mmol/L
(7.34–7.45)
(25–40)
(40–50)
(22–31)
Suggested Comment
Significant metabolic acidosis with hypokalaemia and normal anion gap. Hypernatraemia suggests
dehydration. Possible causes include: renal tubular acidosis, severe diarrhoea, or ureteric diversion. If the cause
is unknown, suggest check urinary pH and electrolytes. If considering parenteral bicarbonate, it is important to
normalise potassium beforehand.
Rationale
Normal anion gap metabolic acidosis can occur due to bicarbonate loss (e.g. severe diarrhoea, ureteric
diversion to the colon, or proximal renal tubular acidosis, also known as type II RTA); failure of acidification
in the distal renal tubule (type I RTA); or excess ammonium retention (e.g. ureteric diversion to the colon). If
the ureter is diverted to the ileum, acidosis is unlikely unless there is obstruction and prolonged urine-gut
contact. Loss of bicarbonate and fluid volume leads to compensatory retention of sodium and chloride, and
hyperchloraemia.
Urine pH should be <5.3 in acidosis, if renal acidification is normal. However, severe volume depletion may
increase urine pH. High urine pH can be followed up by loading tests with bicarbonate or ammonium if
necessary to localise the site of RTA. Increasing blood pH shifts potassium into cells so it is important to
correct the hypokalaemia, then monitor potassium closely during treatment of any acidosis.
References
1. Walmsley RN, White GH. Normal “anion gap” (hyperchloremic) acidosis. Clin Chem 1985;31:309-13.
2. Cruz DN, Huot SJ. Metabolic consequences of urinary diversions: an overview. Am J Med
1997;102:477-84.
3. Rodríguez Soriano J. Renal tubular acidosis: the clinical entity. J Am Soc Nephrol 2002;13:2160-70.
PREFERRED KEY WORDS
Normal anion gap metabolic acidosis
Hypernatraemia/dehydration/increased
urea:creatinine
?Renal tubular acidosis
Type II (proximal) renal tubular acidosis
Type I (distal) renal tubular acidosis
Metabolic acidosis
?Diarrhoea/gastrointestinal loss
Suggest urinary electrolytes
Suggest urine pH
?Urinary diversion
?Obstructed ileal bladder
?Vesicocolic fistula
LESS RELEVANT KEY
WORDS
Hypokalaemia
History of renal surgery
(Partial) respiratory compensation
?(Acute) renal impairment
?Carbonic anhydrase inhibitors
Suggest serum/urine osmolality
Suggest glucose/insulin
Low bicarbonate
Suggest albumin/total
protein/phosphate/Ca/Mg
Suggest urine ammonia
?Renal or gastrointestinal disease
Further clinical info needed
Loss of bicarbonate
Low pH/ acidaemia
Low pCO2
Venous blood gas sample noted
Suggest specialist referral
Phone result
?Dietary intake
?Chronic laxative abuse
?Villous adenoma
Suggest lactate/citrate
Suggest repeat testing
Diuretic phase following acute tubular
necrosis
Suggest troponin/cardiac markers/ECG
?Nephrolithiasis
Suggest ammonia chloride loading
?Increased osmotic diuresis
NOT SUPPORTED KEY
WORDS
?(Resolved) diabetic ketoacidosis
Suggest serum electrolytes, urea,
creatinine
Uncompensated metabolic acidosis
?Vomiting/ infection
Treat with IV potassium and insulin
?Salt ingestion/water depletion
Ingestion of ammonium chloride
?Drug reaction
UNACCEPTABLE KEY
WORDS
?Saline/IV contamination
?Diuretics
?Mineralocorticoid excess
Fluid status unknown
?Cholestyramine
Case 6-06
Renal
Patient ID
58-year-old woman
Patient Location
General Practice
Clinical Notes on Request Form
Newly diagnosed with type II diabetes mellitus.
?Microalbuminuria
Case Details
Sample: morning spot urine
Albumin
Creatinine
ACR
12 mg/L
3.9 mmol/L
3.1 mg/mmol (<3.5)
Additional Information
No previous urine results.
Suggested Comment
Normal results. Suggest repeat in one year.
Rationale
Patients with type II diabetes mellitus should be tested for albuminuria annually starting from the time of
diagnosis. The albumin:creatinine ratio (ACR) is the preferred method for assessment of albuminuria in both
diabetic and non-diabetic individuals. The ACR should be measured using a first morning spot urine specimen;
however, a random urine sample is also acceptable when the former is not possible or practical. Alternately,
the albumin excretion rate (AER) may be measured using 24 h urine collection.
Microalbuminuria is indicated by
ACR 2.5–25 mg/mmol in males
ACR 3.5–35 mg/mmol in females
OR
AER 30–300 mg/24 h in either gender
Macroalbuminuria is indicated by:
ACR >25 mg/mmol in males
ACR >35 mg/mmol in females
OR
AER >300 mg/24 h in either gender
If the ACR or AER is positive for microalbuminuria, the person should have one to two additional ACR/AER
within three months. Microalbuminuria is confirmed if two of three tests are positive. If the ACR or AER
showed macroalbuminuria, total protein should be measured in a 24 h urine collection. If the ACR is normal,
as it is in this patient, testing should be repeated annually.
Reference
1. Johnson DW, Jones GR, Mathew TH, et al.; Australasian Proteinuria Consensus Working Group.
Chronic kidney disease and measurement of albuminuria or proteinuria: a position statement. Med J
Aust 2012;197:224-5.
PREFERRED KEY WORDS
Normal result
Recheck 6–12 months
Further monitoring suggested
LESS RELEVANT KEY
WORDS
Results exclude microalbuminuria
Suggest AER
Criteria 2/3 in 6 weeks =
microalbuminuria
Repeat ACR
Suggest HbA1c
No comment
ACR borderline
Morning urine preferred
Suggest plasma creatinine
Monitor cardiovascular risk factors
Suggest three timed overnight urines
Microalbuminuria may develop later
Albumin fragments may not be detected
Suggest creatinine clearance
Suggest lipids
UNACCEPTABLE KEY
WORDS
No nephropathy
Urine creatinine low, ?diluted
Microalbuminuria present
Exclude urinary tract infection, exercise,
cystic fibrosis
AER >20 x2 = microalbuminuria
AER <20 not microalbuminuria
ADS recommends 12 month monitor
Urine albumin and creatinine slightly
high
Incomplete and inappropriate sample
Check urine glucose level
ACR unreliable, dilute urine
Case 6-10
Renal
Patient ID
32-year-old male
Patient Location
Medical Ward
Clinical Notes on Request Form
Paroxysmal nocturnal haemoglobinuria
Case Details
Sodium
Potassium
Bicarbonate
Urea
Creatinine
136 mmol/L
4.0 mmol/L
21 mmol/L
26.7 mmol/L
552 µmol/L
(134–146)
(3.4–5.0)
(22–32)
(3.0–8.0)
(60–120)
Additional Information
Total Protein 63 g/L
(60–80)
Albumin
34 g/L
(35–50)
Bilirubin
55 µmol/L
(<20)
Haemoglobin 94 g/L
(135–180)
Reticulocytes 4.1%
(0.2–2.0)
Blood film features suggest active haemolysis.
Haem Index 470
(<100)
Suggested Comment
This patient has renal failure. Elevated serum haemoglobin indicates haemolysis. Clinical note of paroxysmal
nocturnal haemoglobinuria suggests in vivo haemolysis, in which case the measured potassium reflects
circulating concentration. Repeat measurement of potassium is recommended for confirmation.
Rationale
Paroxysmal nocturnal haemoglobinuria is a condition characterised by the clonal expansion of one (usually the
red blood cell line) or more haemopoietic stem cells that are deficient in certain surface proteins. These are
acquired through somatic mutations, which predispose the blood cells to complement-mediated haemolysis,
particularly in acidic conditions. This may lead to haemoglobinuria/haemosiderinuria that can damage the
kidneys. Traditionally, the haemolysis is thought to occur in paroxysms during sleep-induced acidosis. More
recent evidence suggests it can occur throughout the day but the haematuria is more notable in the concentrated
overnight urine.
The clinical note given with the biochemistry results should assist with the interpretation of those specific
results. The patient is clearly in renal failure. The evidence for intravascular haemolysis is the raised serum
haemoglobin as measured by the haem index, bilirubin and reticulocytes, and blood film features.
Other supportive features of this intravascular haemolytic condition include elevated serum lactate
dehydrogenase, low/absent haptoglobin, haemoglobinuria/haemosiderinuria. Complications of paroxysmal
nocturnal haemoglobinuria include iron deficiency anaemia, hyoplastic bone marrow and thrombosis, which
may involve the liver, brain, abdominal and lower limb venous systems, and can be fatal.
Reference
1. Krauss JS. Laboratory diagnosis of paroxysmal nocturnal hemoglobinuria. Ann Clin Lab Sci
2003;33:401-6.
PREFERRED KEY WORDS
Renal failure
Intravascular haemolysis
Consistent with paroxysmal nocturnal
haemoglobinuria
Suggest haptoglobin
Sample haemolysed
In vivo potassium okay
?In vitro or in vivo haemolysis
Bilirubin affected by haemolysis
Repeat potassium on nonhaemolysed
sample
LESS RELEVANT KEY
WORDS
UNACCEPTABLE KEY
WORDS
Expected higher potassium
Suggest cell markers by flow cytometry
Hyperbilirubinaemia
Haemolytic anaemia
?Renal haemosiderosis
Active red blood cell production/
increased reticulocytes
Anaemia/low haemoglobin
?Renal failure due to haemoglobinuria
Suggest LDH
Elevated urea and creatinine
Suggest complete haematology
Anaemia multifactorial
Renal failure 2° to paroxysmal nocturnal
haemoglobinuria
Suggest liver function tests and
direct/indirect bilirubin
Suggest iron studies/erythrocyte folate
Suggest direct Coombs’ Test
?Other concurrent disease
Suggest haematology consultation
?Thrombotic thrombocytopaenic purpura
and haemolytic uraemic syndrome
Suggest urine protein/casts
Consult with renal physician
Suggest monitor renal failure
?Creatinine underestimated
?Clinical information
?Disseminated intravascular coagulation
Haemolytic jaundice
?Clinical dehydration
Suggest renal biopsy
?Paroxysmal cold haemoglobinuria
Suggest urine haemoglobin and
haemosiderin
Suggest Ham test
?Renal failure 2° to thrombosis
No comment
Monitor serum potassium
?Previous renal function history
?Non-paroxysmal nocturnal
haemoglobinuria cause of renal failure
Recommend dialysis/exchange
transfusion
?Blood transfusion
?Soluble transferrin receptor assay
?Erythrocytes protoporphyrin high
?Sepsis or thrombotic thrombocytopaenic
purpura
Suggest MRI/CT scans
Suggest neutrophil ALP score
Categorise paroxysmal nocturnal
haemoglobinuria by CLST
Suggest cardiac markers
Suggest Donath-Landsteiner test
Liver function okay
Suggest B-haem streptococcus culture
Suggest fine needle aspiration
Renal recovery as potassium normal
Transfusions and iron therapies
Manage complications of paroxysmal
nocturnal haemoglobinuria
Low albumin due to proteinuria
Suggest eGFR
Case 7-04
Renal
Patient ID
45-year-old female
Patient Location
General Practice
Clinical Notes on Request Form
Routine health check. Fit and well.
Case Details
Urea
Creatinine
eGFR
Glucose (random)
Bilirubin
ALP
GGT
ALT
6.1 mmol/L (2.7–7.8)
120 µmol/L (50–110)
45 mL/min/1.73m2 (80–120)
4.7 mmol/L (3.5–7.8)
6 μmol/L
(3–21)
54 U/L
(30–120)
17 U/L
(10–35)
50 U/L
(0–30)
Additional Information
No previous laboratory results.
Suggested Comment
Mildly elevated creatinine and decreased eGFR suggests possible renal impairment, along with a minor
increase in ALT. eGFR is not reliable in the presence of extreme lean body mass, high protein, vegetarian diets
or creatine supplements. Consider urine protein, mid-stream urine sample and formal creatinine clearance if
there is suspicion that eGFR may be incorrect and review patient for causes of increased ALT.
Rationale
The estimated Glomerular Filtration Rate (eGFR) is derived from serum creatinine, age and sex using the
CKD-EPI equation and it should be automatically reported with every creatinine requested in subjects >18
years of age (1). eGFR results may be unreliable or misleading in subjects with the following: acute changes in
kidney function, on dialysis, exceptional dietary intake (e.g. vegetarian diet, high protein diet, creatine
supplements), extremes of body size, skeletal muscle diseases, and severe liver disease.
An elevated ALT in an asymptomatic subject may be due to alcohol-related liver injury, chronic hepatitis B or
C, autoimmune hepatitis, fatty liver disease, haemochromatosis, Wilson’s disease, or alpha-1-antitrypsin
deficiency (2). Non-hepatic causes include coeliac sprue, muscle diseases, and strenuous exercise.
References
1. Kidney Health Australia. http://www.kidney.org.au
2. Pratt DS, Kaplan MM. Evaluation of abnormal liver-enzyme results in asymptomatic patients. N Engl J
Med 2000;342:1266-71.
PREFERRED KEY WORDS
Check urine albumin/protein
Increased creatinine
Reduced eGFR
Increased ALT
Renal impairment/disease
Repeat creatinine/eGFR
?Medication/drugs
Suggest viral/hepatitis serology
Suggest creatinine clearance
?Hepatitis
Suggest mid-stream urine sample
?Increased muscle mass
eGFR not valid in some situations
Extreme body composition eGFR not
validated
?Diet
Suggest clinical review
LESS RELEVANT KEY
WORDS
Moderate chronic kidney disease
Assess cardiovascular risk factors
Mild liver disease
Renal referral recommended
Repeat liver function tests
Monitor results
Follow-up required
Suggest full liver function tests
C-reactive protein / lipids
?Fatty liver
?Alcohol
Refer to Kidney Foundation website
?Creatinine method interference
Full blood count
Normal urea
Suggest Cockcroft-Gault/body surface
area calculation
Check glomerular haematuria
Review renal risk factors
UNACCEPTABLE KEY
WORDS
?Immunological
Measure blood pressure
Check for diabetes
PTH/calcium studies
Measure calcium/phosphate
Fasting glucose
No dehydration
HS/iron studies
?Acute renal failure
No comment
Consider starting renal therapy
Consider liver imaging
Suggest other miscellaneous tests
Case 10-01
Renal
Patient ID
72-year-old man
Patient Location
General Practice
Clinical Notes on Request Form
?Renal impairment. Blood pressure normal
Case Details
Sodium
Potassium
Glucose
Bicarbonate
Urea
Serum creatinine
eGFR
139 mmol/L (134–146)
4.6 mmol/L (3.4–5.0)
5.1 mmol/L (3.0–5.4)
28 mmol/L
(22–32)
8.2 mmol/L (3.0–8.0)
133 μmol/L (60–110)
49 mL/min/1.73m2 (>60)
Additional Information
Previous serum creatinine 129 μmol/L 6 months ago
Suggested Comment
eGFR between 30 and 60 mL/min/1.73m2 on two occasions 6 months apart indicates stage 3 chronic kidney
disease (CKD). Minimal change in serum creatinine indicates stable kidney function. Suggest assess coronary
vascular disease (CVD) risk and check for albuminuria (urine albumin:creatinine ratio, ACR) and haematuria
(dipstick). If ACR/dipstick is normal, re-assess in 12 months. If albuminuria and/or haematuria present, repeat
ACR/dipstick tests (considered present if two of three repeat tests are positive for ≥3 months) and consider
further investigation including renal U/S and referral to a nephrologist.
Rationale and References
eGFR should be estimated by applying CKD-EPI equation on isotope dilution mass spectrometry-traceable
creatinine assays. Age-related decision points for eGFR in adults are not recommended, as an eGFR
< 60 mL/min/1.73 m2 carries significantly increased risks of adverse clinical outcomes without consistent age
association. Hence, whilst it is very common in older people, it should not be considered a normal part of
ageing.
Initial further investigations suggested include (preferably first morning void) urine albumin:creatinine ratio
(ACR) and dipstick (for haematuria), monitoring of serum creatinine and cardiovascular risk assessment. If no
abnormalities are found, a wait-and-watch approach is appropriate with yearly reviews. Note the reduced GFR
should be taken into account when making drug-dosing decisions, and attention paid to cardiovascular risk
reduction. However, if the investigations show abnormalities, the need for further investigations and specialist
referral should be considered.
References
1. Johnson DW, Jones GR, Mathew TH, et al. Chronic kidney disease and automatic reporting of
estimated glomerular filtration rate: new developments and revised recommendations. Med J Aust
2012;197:224-5.
2. Kidney Health Australia – Health Professionals section. http://www.kidney.org.au
PREFERRED KEY WORDS
eGFR suggests moderate CKD (stage 3)
Consistent with renal impairment
Stable creatinine result
eGFR declines with age
Interpret eGFR with caution in elderly
Typical for age if stable and no other
abnormalities
If stable, no other abnormalities,
unlikely CKD
Assess CVD risk factors
Suggest ACR
Suggest test for proteinuria
Suggest test for haematuria
Suggest fasting lipid profile
Monitor eGFR 3–6 monthly
Beware renal excretion for drugs with
low GFR
Investigate further
Refer to nephrologist if indicated
LESS RELEVANT KEY
WORDS
Slightly raised creatinine and urea
Low eGFR
Mild renal impairment
?Lifestyle habits/underlying causes
?Normal GFR for age
Suggest spot protein/creatinine ratio
Suggest 24 h urine protein excretion
Suggest Ca/PO4/PTH/vit D
?Renal impairment
Apply eGFR as recommended
No comment
?Muscle mass and dietary intake
Due to postrenal or ?intrarenal cause
Clinical correlations required
Suggest cystatin C
Suggest mid-stream urine examinations
eGFR by MDRD uses creatinine, sex and
age
?Drugs that interfere with eGFR
Consider patient age and creatinine
uncertainty of measurement
Electrolytes normal
Review patient (Kidney Health Plan)
?Hydration status
eGFR 10th percentile for age & sex
Use surface area corrected eGFR
eGFR use in stable medical conditions
eGFR inappropriate for drug dosing
Suggest iron studies
Suggest imaging
UNACCEPTABLE KEY
WORDS
Suggest creatinine clearance
Deterioration of renal impairment
Suggest glucose
Suggest PSA
Suggest full blood count and INR
Repeat after overnight fast
Unlikely to be CKD
Suggest renin, aldosterone
Suggest protein electrophoresis
?Diabetic
eGFR suggests severe CKD (stage 4)
Case 11-03
Renal
Patient ID
3 years & 10 months old girl
Patient Location
General Practice
Clinical Notes on Request Form
Febrile. Nephrectomy for Wilm’s tumour 2.5 years ago.
Case Details
Serum
Sodium
Potassium
Glucose
Bicarbonate
Urea
Creatinine
139 mmol/L
5.0 mmol/L
4.6 mmol/L
17 mmol/L
6.9 mmol/L
45 μmol/L
(132–143)
(3.5–5.0)
(3.0–5.5)
(17–30)
(1.1–5.7)
(23–37)
Additional Information
C-reactive protein 110 mg/L (< 8.0)
Previous serum creatinine 27 μmol/L 18 months ago (reference interval 16–31 μmol/L)
Suggested Comment
Results indicate renal impairment. An acute infective or inflammatory process is likely. Causes of acute
deterioration in renal function including dehydration, urinary tract infection or drug toxicity should be
excluded. Interference in the creatinine result (for creatinine assays based on Jaffe reaction), especially from
cephalosporins, should be excluded. In a child of this age, age-specific creatinine is a guide only, and an
estimation of glomerular filtration rate (eGFR) will improve assessment of renal function.
Rationale
The important finding here is of renal impairment in a child already at risk with a history of solitary kidney and
Wilm’s tumour. The rise relative to the reference interval since the previous test suggests this is not due to the
previous nephrectomy, but represents a new disease process. The clinical notes and markedly elevated Creactive protein (CRP) suggest an acute infective/inflammatory process is likely. Thus in the first instance,
causes of acute renal deterioration including dehydration, urinary tract infection or drug toxicity should be
excluded. It is important to recognise that in a child with a history of previous illness may still be small for age.
For these patients, age-based creatinine reference intervals are only a guide and the GFR may be lower than the
creatinine suggests.
There is limited data in estimating GFR in children with solitary kidney, using either the modified Schwartz
formula or cystatin C-based formulas; however the relationship appears no worse than in the general paediatric
population. The KDOQI guidelines for evaluating kidney disease in children and adolescents recommend
estimation of GFR using a creatinine-based formula (1). The guidelines also recommend a first morning urine
specimen for protein:creatinine ratio, although random specimens are acceptable, for assessment of proteinuria.
Some medications may interfere with creatinine result obtained from assays based on Jaffe reaction, and
should be enquired for, especially cephalosporins. Specific to children with a history of Wilm’s tumour, a
survey suggests renal failure is a rare (<0.26%) complication in children with unilateral nephrectomy (2); the
more frequent causes of kidney failure in that group include Denys-Drash syndrome (in males;
pseudohermaphroditism and diffuse mesangial sclerosis), progressive tumour in the remaining kidney,
radiation nephritis and other causes, including hyperfiltration injury.
References
1. Hogg RJ, Furth S, Lemley KV, et al. National Kidney Foundation's Kidney Disease Outcomes Quality
Initiative clinical practice guidelines for chronic kidney disease in children and adolescents: evaluation,
classification and stratification. Pediatrics 2003:111:1416-21.
2. Ritchey ML, Green DM, Thomas PR et al. Renal failure in Wilms’ tumor patients: a report from the
National Wilms’ Tumor Study Group. Med Pediatr Oncol 1996;26:75-80.
PREFERRED KEY WORDS
LESS RELEVANT KEY
WORDS
Renal failure ?acute ?chronic
Deterioration renal function compared to
previous urea and electrolytes
Impaired renal function
?Pre-renal renal impairment
?Dehydration ?secondary to infection
?Urinary tract infection
?Infection/sepsis
?Cephalosporin interference - Jaffe
Suggest urine microscopy, culture and
sensitivity/casts/crystals
Suggest urine creatinine/protein/
albumin/microalbumin
Suggest ultrasound of kidney/lower
abdomen
Suggest monitor renal function test,
calcium and liver function tests
Estimate eGFR - Schwartz formula
Exclude pyelonephritis
Suggest urine protein/creatinine ratio a.m. sample
?Trimethoprim blocking creatinine
excretion
?Tumour recurrence
Elevated urea and/or creatinine
Elevated C-reactive protein
?Inflammation
Febrile status
Suggest full blood examination/ESR
Further investigation required
Low to low normal bicarbonate
Suggest referral to specialist
Acidosis
Suggest timed urine protein/creatinine
clearance
Suggest arterial blood gases
Borderline hyperkalaemia
Metabolic acidosis
High normal potassium
Suggest blood cultures
?Nephritis
?Radiation adverse effects
?Medication history
?Intrinsic renal disease
?Extrinsic renal disease
?Recent meat intake
Low creatinine/urea ratio
Exclude glomerulonephritis
?Chemotherapy adverse effects
Hyperkalaemia
Suggest referral to emergency
department/admission
Reduced eGFR
Suggest urine electrolytes
Wilm’s tumour - urogenital malformation
?Hyperventilating
Suggest loss of heterozygosity
karyotyping for Ip/16q
?Blood pressure status
Consider empiric antibiotics
Unable to calculate eGFR due to age
?Previous tx related impairment renal
failure
Recommend rehydration
Suggest intravenous pyelogram
Exclude tumour lysis
?WAGR (Wilms’ tumour, aniridia,
genitourinary anomalies, and mental
retardation ) syndrome
?Long term complication of one kidney
Suggest CT scan of remaining kidney
If renal failure persists - renal imaging
Aggressive management and phone GP
UNACCEPTABLE KEY
WORDS
Suggest repeat tests when patient well
Suggest renal biopsy
?Denys-Drash syndrome
?Compensatory hypertrophy of kidney
Suggest tumour markers e.g. CEA
Suggest serum am cortisol
Suggest immunoglobulins
Suggest chest CT scan
Suggest IGF-II
Suggest coagulation studies
Case 11-07
Renal
Patient ID
26-year-old man, refugee clinic
Patient Location
General Practice
Clinical Notes on Request Form
Generalised oedema, eosinophilia
Case Details
Fasting serum
Sodium
Potassium
Chloride
Bicarbonate
Glucose
Urea
Creatinine
Urate
Phosphate
Total Calcium
Albumin
Protein
Total Bilirubin
GGT
ALP
ALT
Additional Information
Fasting serum
Cholesterol
Triglyceride
138 mmol/L
4.6 mmol/L
101 mmol/L
27 mmol/L
5.2 mmol/L
5.8 mmol/L
63 μmol/L
0.63 mmol/L
1.62 mmol/L
2.10 mmol/L
7 g/L
37 g/L
2 μmol/L
51 U/L
88 U/L
37 U/L
(137–145)
(3.5–4.9)
(100–109)
(22–32)
(3.2–5.5)
(2.7–8.0)
(50–120)
(0.15–0.45)
(0.65–1.45)
(2.10–2.55)
(34–48)
(65–85)
(2–24)
(<60)
(30–110)
(<55)
23.7 mmol/L (<5.5) (desirable)
7.5 mmol/L (0.3–2.0)
Suggested Comment
Severe hypoalbuminaemia and hyperlipidaemia in the clinical setting of oedema consistent with nephrotic
syndrome. In view of the history of refugee status and the eosinophilia, helminthic parasites are a likely
aetiology. Recommend urine protein/creatinine ratio and faecal microscopy.
Rationale
Severe hypoalbuminaemia and hyperlipidaemia with oedema are typical signs of nephrotic syndrome. The lack
of gastrointestinal symptoms and presence of hyperlipidaemia make a protein-losing enteropathy unlikely.
There are many causes of nephrotic syndrome but the history of refugee status and presence of eosinophilia
raise the possibility of parasitic infestation as the cause.
Helminthic parasites such as strongyloides, filariae, schistosoma and echinococcus are more commonly
associated with eosinophilia than other parasites. Initial investigations should focus on the cause of
hypoalbuminaemia. A spot protein/creatinine and/or albumin/creatinine ratio will confirm the presence of
macroalbuminuria and form the basis for subsequent monitoring. A 24 h urine protein collection will help
clarify nephrotic range proteinuria. Faecal microscopy may help identify a parasitic cause if this is present.
References
1. van Velthuysen ML, Florquin S. Glomerulopathy associated with parasitic infections. Clin Microbiol
Rev 2000;13:55-66.
2. Mawhorter SD. Eosinophilia caused by parasites. Pediatr Ann 1994;23:405,409-13.
PREFERRED KEY WORDS
Severe hypoproteinaemia
Severe hypoalbuminaemia
Marked hyperlipidaemia
Liver enzymes normal
Generalised oedema
Eosinophilia
?Nephrotic syndrome
Suggestion of nephrotic syndrome
?Parasitic infestation
?HBV/HIV/syphilis/hepatitis
?Protein-losing enteropathy
Secondary lipid abnormalities
Suggest referral to specialist
Suggest 24 h urine protein/albumin
Urine protein/creatinine ratio
Suggest stool examination
?Infiltrations
LESS RELEVANT KEY
WORDS
Hyperlipidaemia
Hyperuricaemia
Hypoalbuminaemia
Hypoproteinaemia
Allergies/inflammation/autoimmune
Suggest hepatitis/parasite serology
?Malnutrition/malabsorption
Check full blood examination/ film/ PT/
Coagulation
?Drug related
Suggest serum and urine electrophoresis
Hyperphosphataemia
Suggest autoimmune markers
?Viral/chronic infection
?Haematological malignancy
Urgent medical attention required
Suggest faecal alpha 1 antitrypsin
Suggest ionised calcium
?Diabetes/hypertension
?Glomerular disease
Suggest examine urinary sediment
Increased cell turnover
Associated with renal dysfunction
?Liver disease/failure
Interference from lipaemia
?Renal biopsy if indicated
Suggest thyroid function tests
?Renal disease
Repeat glucose
Suggest urine microscopy and culture
?Hypothyroidism
Adjusted calcium high
Suggest urea/creatinine/creatinine
clearance
?Post-streptococcal glomerulonephritis
Borderline low calcium
Check urine lipiduria
?Dilutional due to oedema
Suggest electrolytes with direct ISE
method
?Sickle cell anaemia
Suggest CRP
Repeat albumin/protein
UNACCEPTABLE KEY
WORDS
Primary lipid disorder not excluded
Suggest PTH
?Inherited analbuminaemia
?Gout / increased salt intake / LeschNyhan syndrome
Suggest cardiac risk analysis
Diabetes mellitus unlikely
?Cushing's disease
?Thyrotoxicosis
?Decreased synthesis
Case 12-10
Renal
Patient ID
40-year-old female
Patient Location
General Practice
Clinical Notes on Request Form
Newly diagnosed with diabetes mellitus
Case Details
Sample: Random spot urine
Time: 2:00 pm
Albumin:creatinine ratio
5.5 mg/mmol creatinine (<3.5)
Additional Information
HbA1c
HbA1c (IFCC)
7.4%
57 mmol/mol
Suggested Comment
Mild elevation of urine albumin:creatinine ratio in a newly diagnosed patient with diabetes. Random collection
noted; first morning void collections are recommended to exclude exercise and posture effects. Sample
collection should also avoid acute heavy exercise within 24 hours, febrile illness, nonsteroidal antiinflammatory drugs, menstruation and vaginal discharge. Two of three positive samples within three months
are required for diagnosis.
Rationale
Urine albumin:creatinine ratio (ACR) is the preferred test for assessing proteinuria in diabetic patients. As this
is a newly diagnosed case at age 40, it is likely that this is type 2 diabetes and it is appropriate to test for ACR
at the time of diagnosis. For a diagnosis of persistent albuminuria there is a requirement for at least two out of
three separate collections to be positive. This is due to the high within-subject biological variation of ACR
(CVi 30–60%) as well as the many causes of false positive results.
The laboratory is in a good position to advise on appropriate sample collection. The preferred specimen is a
first morning collection with avoidance of acute heavy exercise within 24 hours, febrile illness, nonsteroidal
anti-inflammatory drugs, menstruation or vaginal discharge. Other factors which can increase urine albumin
excretion but which may not be able to be controlled are congestive cardiac failure and some drugs, especially
ACE inhibitors and angiotensin II receptor blockers (ARBs).
The use of timed urine collections, either overnight or 24 hour is not required, but it is recommended to use
gender-specific cut points (<3.5 mmol/mg for women and <2.5 mg/mmol for men) to provide an approximate
correction for the increased creatinine output in men. There are many factors that must be managed for a
patient with newly diagnosed diabetes, and it is not possible to cover them all in a comment attached to a urine
ACR. The comment should focus on the interpretation and responses to this specific test, as laboratories are
generally unaware of the other factors that may influence management.
References
1. Martin H. Laboratory measurement of urine albumin and urine total protein in screening for proteinuria
in chronic kidney disease. Clin Biochem Rev 2011;32:97-102.
2. Royal Australian College of General Practitioners (RACGP). Guidelines for preventive activities in
general practice. 6th edition. Melbourne: RACGP; 2005.
PREFERRED KEY WORDS
LESS RELEVANT KEY
WORDS
Borderline raised ACR
Confirm microalbuminuria
Exclude transient microalbuminuria
Suggest repeat spot early morning urine
(1st void) for test
Suggest repeat ACR
Confirm 2/3 urines in 3–6 months
ACR>3.5
Exclude fever/urinary tract
infection/exercise/menses
Exclude diabetic nephropathy
Exclude heart failure
If eGFR >60 then cardiovascular disease
risk low
ADS recommends ACR<3.5
Urine ACR unreliable, collected at 2 pm
Exclude pregnancy
Suggest glycaemic control and review
Suggest serum urea and electrolytes,
creatinine
Suggest eGFR
Results suggest microalbuminuria
Exclude hypertension/check BP
ADS recommends HbA1c ≤7.0
Elevated urinary albumin
HbA1c result confirms diabetes
Elevated ACR
Suggest lipid/HDL profile and
monitoring
Suboptimal glycaemic control
Patient at risk of cardiovascular disease
HbA1c target of <6.0% desirable
?Degree of renal damage
Persistent raised ACR - nephropathy
Assess cardiovascular disease risk factors
Check weight/BMI/diet
Suggest lifestyle modification
Exclude poor glycaemic control
Suggest liver function tests
Suggest thyroid function tests
?Chronic kidney disease
If microalbuminuria - treatment advised
Suggest urine collection – fasting and
supine
Suggest retinal screening
Suggest foot screening
?High ACR due to low muscle mass
?Weight loss
?Polydipsia/polyuria
?Family history of diabetes
Exclude smoking
NOT SUPPORTED KEY
WORDS
Suggest follow up testing annually
Confirm diabetes mellitus
Suggest 24 h urine protein
Suggest fasting glucose monitoring
Microalbuminuria clinically significant
Chronic kidney disease indicated
Suggest repeat HbA1c
Suggest 24 h urine for albumin
Test >2 h postprandial
Hb variants can affect HbA1c value
Suggest endocrine/diabetes review
Suggest overnight urine albumin
collection
Goal for Type II DM HbA1c <6.5%
MISLEADING KEY WORDS
Suggest therapy e.g. ACE inhibitor
Suggest repeat urine albumin when
HbA1c <6.0%
HbA1c shows good diabetic control
Case 14-02
Renal
Patient ID
20-year-old female
Patient Location
General Practice
Clinical Notes on Request Form
Feeling run down; past medical history unremarkable.
Case Details
Sodium
Potassium
Urea
Creatinine
eGFR
138 mmol/L (134–145)
5.1 mmol/L (3.4–5.0)
34.1 mmol/L (3.0–7.0)
256 µmol/L (44–80)
21 mL/min/1.73m2
Additional Information
Thyroid function tests were within normal limits.
Full blood count showed a haemoglobin of 9.7 g/dL (11.5–16).
Red cells were normochromic normocytic.
A repeat electrolytes, urea, creatinine was performed, and was essentially similar.
Suggested Comment
Serum creatinine and eGFR indicates severe reduction in renal function. In view of coexistent anaemia,
chronic kidney disease (CKD) is likely, however, exclusion of causes of acute renal failure is recommended.
Urgent referral to a nephrologist is recommended with additional testing for fasting glucose, lipids,
bicarbonate, calcium, phosphate, parathyroid hormone (PTH) and spot urine albumin. Blood pressure should
be checked and haematuria assessed using a urine dipstick.
Rationale
A persistent eGFR between 15 and 29 mL/min/1.73m2 constitutes stage 4 chronic kidney disease (CKD).
While the requirement for the definition of CKD is two tests greater than three months apart, referral to a
nephrologist should be performed urgently in view of the age of the patient, and lack of information on the rate
of progressive decline in renal function. Further investigations may be performed in the meantime to look for
common related conditions, which may be the cause/consequence of CKD, or interact with CKD to increase
morbidity/mortality. Anaemia of CKD is related to both reduced erythropoietin production by the kidney and
resistance to the action of erythropoietin. High blood pressure is both a cause and an effect of CKD, and should
be detected early and treated. Diabetes is a common cause of CKD and, if present, should be treated rigorously,
but would also impact on the management of other aspects of CKD.
CKD constitutes a significant risk factor for cardiovascular events and death; an absolute risk approach is
recommended. Changes in metabolism of calcium, phosphate, parathyroid hormone (PTH) and vitamin D
typically start to occur once GFR is <60 mL/min/1.73m2, and are associated with an increased risk of fracture
and increased cardiovascular mortality.
Haematuria, if present, could be consistent with glomerulonephritis. Renal ultrasound may be considered to
exclude structural abnormalities as a cause of chronic kidney damage.
Reference
1. Kidney Health Australia. http://www.kidney.org.au
PREFERRED KEY WORDS
Severe renal function impairment
Coexistent anaemia
Chronic kidney disease is likely
Consistent with stage 4 chronic renal
failure
?Proteinuria/suggest spot urine albumin
Suggest calcium/phosphate
(PO4)/parathyroid hormone (PTH)
Suggest microscopy and culture
Suggest urine albumin/creatinine ratio
Suggest fasting glucose
Review medications/drugs
Check blood pressure
?Urinary tract infection/ pyelonephritis/
polycystic kidney disease/ drug toxic
Mild hyperkalaemia
Urgent referral to nephrologist
recommended
Assess haematuria
?Diabetes
Suggest electrolytes, urea, creatinine,
bicarbonate
?Hypertension
Suggest HbA1c
Suggest lipids
High potassium secondary to reduced
renal function
Avoid nephrotoxic agents
eGFR needs to be elevated >3 months
?Glomerulonephritis/vasculitis/acute
tubular necrosis
LESS RELEVANT KEY
WORDS
Suggest renal imaging
?Acute renal injury/failure
Raised urea and creatinine
Low eGFR
Suggest protein/albumin/liver function
tests
Monitor kidney function
Exclusion of causes of acute kidney
injury recommended
Confirm with repeat sample
Suggest clinical assessment
?Family history kidney disease
Repeat tests twice in next 3 months
Suggest iron studies
Suggest repeat urea/creatinine
Low haemoglobin
Suggest 24h protein
Assess volume status
Confirm eGFR
Suggest C-reactive protein
NOT SUPPORTED KEY
WORDS
Suggest 24h urine electrolytes/ urea/
creatine/calcium/PO4
?Systemic lupus erythematosus
Suggest serum and urine osmolality
?Dehydration
?eGFR unreliable due to body size/diet
Suggest hCG to exclude pregnancy
?Cardiac muscle damage
Suggest urine myoglobin
?Synthetic cannabis use
Suggest renal biopsy
Suggest CK/CKMB
?Pre-renal cause
?Acute blood loss
Suggest erythropoietin levels
Screen for ANA/ANCA/GBM antibodies
MISLEADING KEY WORDS
?Stage 3 chronic kidney disease
Suggest bone marrow studies
Case 14-03
Renal
Patient ID
72-year-old male
Patient Location
General Practice
Clinical Notes on Request Form
Presents with malaise and generalised muscle pain. No medications.
Case Details
Na
K
Cl
HCO3
Urea
Creatinine
Glucose (random)
136 mmol/L
6.0 mmol/L
104 mmol/L
25 mmol/L
16.5 mmol/L
180 µmol/L
9.5 mmol/L
(135–145)
(3.5–4.5)
(98–108)
(23–33)
(3.0–8.0)
(60–120)
(3.0–5.5)
Suggested Comment
Disproportionately raised potassium with chronic kidney disease (CKD) stage 3b (eGFR: 32 mL/min/1.73m 2)
in an elderly person with hyperglycaemia is suggestive of hyporeninaemic hypoaldosteronism. Please repeat
urgently on a plasma sample to confirm potassium and renal status. Recommend further investigation with
fasting glucose, creatine kinase, morning cortisol, renin and aldosterone and urine albumin (first morning
void). NB acute causes for reduced GFR should be excluded.
Rationale
Effective renal excretion of potassium is typically maintained until the eGFR falls below ~15 mL/min, i.e.
stage 5 chronic kidney disease (CKD). Hence this elevation in potassium is higher than expected for this
degree of renal impairment. Pseudohyperkalaemia should be excluded by urgent repeat testing. Syndrome of
hyporeninaemic hypoaldosteronism is a relatively common but often overlooked cause of hyperkalaemia with
some authors suggesting it may account for 10% of all cases of hyperkalaemia. The typical patient is an elderly
with diabetes mellitus; often the syndrome is triggered by stressors such as dehydration, acute illness or the
introduction of medications such as non-steroidal anti-inflammatory drugs or angiotensin converting enzyme
inhibitors (although this patient was stated not to be on medications).
Once true hyperkalaemia is confirmed, adrenal insufficiency should be excluded and renin/aldosterone
evaluated. A reduced GFR at any age is indication for investigation and follow-up. Acute reduction in GFR
must always be excluded. The CKD management guidelines (2012) are available at www.kidney.org.au and
provide guidance on response to a first abnormal result (repeat with albumin:creatinine ratio within 2 weeks) as
well as guidance for further testing and treatment based on both the eGFR and the albumin:creatinine ratio.
References
1. Williams GH. Hyporeninemic hypoaldosteronism. N Engl J Med 1986;314:1041-2.
2. Hollander-Rodriguez JC, Calvert JF Jr. Hyperkalemia. Am Fam Physician 2006;73:283-90.
PREFERRED KEY WORDS
LESS RELEVANT KEY
WORDS
NOT SUPPORTED KEY
WORDS
Critical hyperkalaemia
Potassium is higher than expected
Renal impairment and with
hyperkalaemia
High urea, creatinine and potassium
consistent with CKD
eGFR 32 mL/min consistent with stage
3b CKD if present for 3 months
eGFR suggest stage 3 CKD
Moderate renal dysfunction
?Acute renal failure
?Acute/chronic kidney disease
Exclude pseudohyperkalaemia
Suggest repeat electrolytes/bicarbonate
Check potassium on both heparin and
serum samples
Suggest eGFR
Moderate hyperglycaemia
?Hyporeninaemic hypoaldosteronism
Increased risk of diabetes
Suggest fasting glucose/HbA1c
Suggest GTT
Suggest creatine kinase to assess muscle
pain
Suggest urine albumin:creatinine ratio/
protein/ microalbumin
?Diabetes mellitus
Suggest repeat serum creatinine, urea and
glucose
?Adrenal insufficiency/Addison's
Suggest morning cortisol
Suggest aldosterone/renin
Suggest synacthen stimulation test
Suggest full blood count and platelets
Urgent clinical review
Kidney injury secondary to type 2
diabetes mellitus (DM)
Check for previous results
Refer to www.kidney.org.au
?RTA type 4
Exclude acute causes for reduced GFR
Suggest urinalysis/culture and sensitivity
Increased random glucose
Check and monitor blood pressure
Increased potassium, urea and creatinine
Suggest calcium, phosphate, vitamin D,
parathyroid hormone, magnesium, iron
Suggest renal ultrasound/imaging
Suggest fasting lipids
Suggest thyroid function tests
?Rhabdomyolysis
Suggest liver function tests
Suggest electrocardiogram
Low sodium /high potassium
?Dehydration
Assess cardiovascular disease risk factors
Suggest CK, CKMB, myoglobin, AST,
troponin I
Check specimen for haemolysis
Suggest C-reactive protein/autoimmune
testing
?Metabolic/inflammatory/malignancy
Review of medications
Normal sodium/chloride
?Family history of kidney disease
HbA1c not Medicare rebatable
?Low insulin
?Renal failure
?Glomerulonephritis
?Lupus
Offer advice on lifestyle change
Suggest arterial blood gas
Calculated osmolarity: 310 mmol/kg
Renal failure risk factors:
>60y/obese/smoker
Prerenal cause of renal dysfunction
Mild hyperkalaemia
Hyperkalaemia due to renal impairment
Refer nephrologist/urologist
Suggest plasma and urine electrophoresis
CARA guide-31 eGFR is stage 4/5
eGFR indicates advanced CKD
Suggest serum/urine osmolality
Suggest viral serology
MISLEADING KEY WORDS
Check creatinine after 6–12 weeks
?Aging kidney
Case 6-04
Ca/Mg/Phos
Patient ID
43-year-old male
Patient Location
General Practice
Clinical Notes on Request Form
Cramps
Case Details
Sodium
Potassium
Bicarbonate
Urea
Creatinine
Total Protein
Albumin
Bilirubin
ALT
Gamma GT
Alk Phos
Calcium, adjusted
Phosphate
142 mmol/L
4.3 mmol/L
29 mmol/L
5.6 mmol/L
0.1 mmol/L
75 g/L
45 g/L
15 μmol/L
21 U/L
14 U/L
67 U/L
1.55 mmol/L
1.8 nmol/L
(136–145)
(3.5–5.1)
(21–32)
(2.0–8.5)
(0.06–0.11)
(60–80)
(35–52)
(<21)
(0–40)
(12–64)
(<110)
(2.10–2.60)
(0.8-1.5)
Suggested Comment
The severe hypocalcaemia and the mild hyperphosphataemia in the presence of normal renal function suggest
hypoparathyroidism. Suggest repeat calcium together with PTH, and also exclude hypomagnesaemia.
Rationale
The low calcium and high phosphate in the absence of renal failure suggest hypoparathyroidism. To exclude
other causes of hypocalcaemia, the measurement of 25-OH vitamin D, magnesium and a clinical history would
be useful. The most likely causes for hypoparathyroidism are post neck surgery and autoimmune disease.
Reference
1. Marx SJ. Hyperparathyroid and hypoparathyroid disorders. N Engl J Med 2000;343:1863-75.
PREFERRED KEY WORDS
?Hypoparathyroidism
PTH deficiency
?Clinical history
Suggest PTH
Suggest magnesium
?Hypomagnesaemia
LESS RELEVANT KEY
WORDS
Hypocalcaemia
Raised phosphate
Suggest vitamin D
Check ionised calcium
Normal renal function
?Vitamin D deficiency
?Other causes
?Pseudohypoparathyroidism
?Drugs/medications
Suggest thyroid function tests
Low corrected calcium
Phone to discuss with doctor
Normal ALP
Consider calcium replacement
Suggest other test
Contamination unlikely
Suggest parathyroid antibodies
Normal albumin
UNACCEPTABLE KEY
WORDS
Suggest 24h urine calcium/phosphate
Suggest amylase/lipase
?Acute pancreatitis
Tetany suggested
?Artefactual causes
Suggest CK
Suggest acute referral
Suggest creatinine clearance
?Hyperparathyroidism
?Renal disease
Hypercalcaemia
?Vitamin D intoxication
Suggest calcitonin
Suggest ALP isoenzymes
Suggest osmolality
Case 7-03
Ca/Mg/Phos
Patient ID
58-year-old female
Patient Location
Surgical Ward
Clinical Notes on Request Form
Post-colectomy
Case Details
Sodium
Potassium
Urea
Creatinine
Calcium
Phosphate
Albumin
146 mmol/L
3.6 mmol/L
4.0 mmol/L
50 µmol/L
2.1 mmol/L
0.2 mmol/L
31 g/L
(136–146)
(3.5–5.0)
(2.7–8.0)
(60–110)
(2.2–2.6)
(0.8–1.4)
(35–50)
Additional Information
Pre-operative biochemistry unremarkable.
Suggested Comment
Severe hypophosphataemia may indicate phosphate depletion, possibly accentuated by refeeding. Consider
urgent parenteral supplementation.
Rationale
Phosphate is the most abundant intracellular anion and only <0.1% is found in the extracellular fluid. Serum
measurement of phosphate is not a sensitive reflection of intracellular stores. Hypophosphataemia is a
relatively common abnormal biochemistry finding among hospitalised patients. These are usually mild and
self-limiting.
Hypophosphataemia can be caused by intracellular shifts driven by insulin, catecholamine or alkalosis, such as
refeeding syndrome, recovery from diabetic ketoacidosis, respiratory/ metabolic alkalosis, and glucose or
insulin administration. It may also be due to decreased intestinal absorption, such as vomiting, diarrhoea,
malabsorption/malnutrition, and vitamin D deficiency. Lastly, an increase in renal excretion may precipitate
hypophosphataemia, for example in primary/secondary hyperparathyroidism or disorders of renal handling
(e.g. Fanconi’s syndrome, hypophosphataemic rickets, oncogenic osteomalacia). In acute care setting, severe
hypophosphataemia is associated with alcoholism and refeeding without phosphate replacement.
In this case there is no evidence of long-standing hypovitaminosis D or renal phosphate wasting and an acute
mechanism should be suspected. Severe hypophosphataemia leads to central nervous system effects
(irritability, somnolence), cardiac dysfunction and rhabdomyolysis. In the presence of rhabdomyolysis the
phosphate level increases again.
References
1. Singhal PC, Kumar A, Desroches L, et al. Prevalence and predictors of rhabdomyolysis in patients with
hypophosphatemia. Am J Med 1992;92:458-64.
2. Troup S. Phosphate monograph. http://www.acb.org.uk/Nat%20Lab%20Med%20Hbk/Phosphate.pdf
PREFERRED KEY WORDS
LESS RELEVANT KEY
WORDS
Marked hypophosphataemia
Phone results
?On total parenteral nutrition
?Refeeding syndrome
?Malabsorption post-colectomy
?Gastrointestinal tract loss
?Respiratory alkalosis
?Phosphate loss/sequestration
Suggest repeat/monitor levels
Suggest phosphate replacement
Suggest other electrolytes/ vitamins/
protein tests
Suggest magnesium
Risk of myopathy and associated
problems
Risk of rhabdomyolysis
Suggest assess acid-base
?IV glucose
?Intracellular shift with glucose load
?Parenteral nutrition without phosphate
?Malnutrition
Corrected calcium okay
Hypoalbuminaemia
?Renal losses
?Drug effect
Suggest calcium/phosphate excretion
studies
?Alcoholism
Suggest ionised calcium / corrected
calcium
Suggest PTH
Check glucose
Refer to pathologist
Low albumin ?due to acute response
?Artefact
?Hyperparathyroidism
?Acid-base imbalance
Suggest nutritional review
Suggest clinical review
?Magnesium deficiency
Low normal potassium
History of post-colectomy
Low creatinine
May not reflect cellular phosphate
High normal sodium
?Aldosterone activation
UNACCEPTABLE KEY
WORDS
Hypocalcaemia
?Sepsis
?Due to pre-op preparation
?IV overhydration
?Treated diabetic ketoacidosis
Suggest creatine kinase
?Liver disease
Suggest troponin/BNP/ECG
?Magnesium supplementation
?Neuroleptic malignant syndrome
Generally unremarkable
No comment on this case
?Recovery from burns
Suggest calcium replacement
?On respiratory ventilation
?Cardiac event
Case 9-08
Ca/Mg/Phos
Patient ID
57-year-old male
Patient Location
Medical Outpatients
Clinical Notes on Request Form
Family history of hypercalcaemia.
Case Details
Serum
Na
139 mmol/L (135–145)
K
4.5 mmol/L (3.5–5.0)
Cl
103 mmol/L (96–109)
Bicarbonate 31 mmol/L
(23–32)
Urea
5.8 mmol/L (3.5–7.5)
Creatinine
95 μmol/L
(40–120)
Calcium
2.64 mmol/L (2.10–2.60)
Phosphate
0.7 mmol/L (0.8–1.5)
Albumin
48 g/L
(35–50)
PTH
5.0 pmol/L
(1.6–6.9)
Fasting Spot Urine
Urine Calcium
<0.30 mmol/L
Urine Creatinine
1.5 mmol/L
Calcium/Creat. Ratio <0.20 mol/mol
Suggested Comment
Suggest check ionise calcium, repeat plasma phosphate (fasting) and consider 24 hour urine calcium/creatinine
clearance ratio to investigate for possible familial hypocalciuric hypercalcaemia.
Rationale
Familial hypocalciuric hypercalcaemia (FHH) is a rare (1:70,000) autosomal dominant inherited condition
caused by heterogeneously distributed inactivating mutations within the calcium sensing receptor (CASR)
gene. It is characterised by persistent asymptomatic hypercalcaemia, relatively low urine calcium and
inappropriately normal or elevated parathyroid hormone. It is important to differentiate FHH from primary
hyperparathyroidism as the former requires no specific treatment while the latter requires surgery.
A variety of methods have been advocated to adjust serum total calcium to estimate ionised calcium but all
have their limitations. It is advisable to measure ionised calcium directly to clarify the diagnosis in this case. In
the setting of hypercalcaemia, either a 24 h urine calcium:creatinine clearance ratio [Ca/CrCl, (24-h Ucalcium/P- calcium, total)/(24-h U-creatinine/P-creatinine)] of <0.01 or a fasting urine calcium excretion (CaE)
of <30 μmol/L GFR are consistent with the diagnosis of FHH. The use of either a spot urine calcium or spot
urine calcium/creatinine ratio is insufficiently robust for diagnostic purposes.
The mutations associated with FHH are widely distributed in the extracellular calcium sensing or the signal
transduction domain of the CASR gene, so targeted genetic testing is insufficient for diagnosis. The relatively
low serum phosphate appears incongruous with FHH and assessment of a repeat serum/plasma phosphate and
urine TmPi is advisable, as mild primary hyperparathyroidism is also possible in this case and urine Ca/CrCl or
CaE can overlap in both conditions. Ultimately, in some cases, family studies of serum ionised calcium and
urine Ca/CrCl may still be required.
References
1. Glendenning P. Diagnosis of primary hyperparathyroidism: controversies, practical issues and the
need for Australian guidelines. Intern Med J 2003;33:598-603.
2. Christensen SE, Nissen PH, Vestergaard P, et al. Discriminative power of three indices of renal
calcium excretion for the distinction between familial hypocalciuric hypercalcaemia and primary
hyperparathyroidism: a follow-up study on methods. Clin Endocrinol (Oxf) 2008;69:713-20.
PREFERRED KEY WORDS
Consistent with familial hypocalciuric
hypercalcaemia (FHH)
?Familial hypocalciuric hypercalcaemia
Consistent with familial hypercalcaemic
syndrome
Suggest ionised calcium
Repeat phosphate
Suggest 24h urine calcium
LESS RELEVANT KEY
WORDS
UNACCEPTABLE KEY
WORDS
Mild hypercalcaemia
Urinary calcium low
Hypophosphataemia
Family history of hypercalcaemia
?Primary hyperparathyroidism
Normal PTH level
Inappropriately normal PTH
Corrected calcium normal
FHH asymptomatic, benign condition
Suggest magnesium
Due to calcium sensing receptor (CASR)
gene mutation
Suggest repeat testing
Autosomal dominant condition
Suggest vitamin D
?Multiple endocrine neoplasia (MEN) 1
or 2a
PTH top end reference range
Urine dilute, low creatinine
Suggest thyroid function tests
Suggest family screening
Suggest ALP
Normal renal function
?Drug induced hypercalcaemia
Urine Ca/Cr diagnosis of FHH or l°
hyperparathyroidism
Ca/Cr ratio difficult to interpret
Low calcium/creatinine clearance
Repeat non-fast, no tourniquet
?Malabsorption, renal loss, alkalosis
?Familial hyperparathyroidism
?Excess vitamin D
Parathyroidectomy contraindicated
Low PO4 ? intake/antacids/non-fasting
Repeat PTH
No comment
Need clinical history
FHH requires no treatment
Inappropriate PTH or PHrP secretion
Suggest endocrinology review
?l° hyperPTH + vitamin D deficiency
?Fasting sample
Suggest genetic testing for CASR
?Neoplasm
Suggest parathyroid scan
FHH may lead to hyperCa crisis
Hyperparathyroidism unlikely
Suggest succinic dehydrogenase mutation
Consistent with hypoparathyroidism
?Milk-alkali syndrome
?Chrondrocalcinosis, gall stones
Suggest calcitonin
?Hypocalciuria 2° to PTH
Renal impairment not excluded
Suggest PHrP
Suggest 1,25 OH vitamin D
Case 10-08
Ca/Mg/Phos
Patient ID
31-year-old female
Patient Location
Endocrine Clinic
Clinical Notes on Request Form
Muscular dystrophy. ? Disuse bone loss
Case Details
Fasting morning spot urine
Creatinine
3.8 mmol/L
Calcium
3.37 mmol/L
N-Telopeptide
404 nmol BCE/L
Calcium/Creatinine
0.89 mol/mol (0.10–0.58)
N-Telopeptide/Creat.
106 nmol BCE/mmol (<50)
(BCE = Bone Collagen Equivalents)
Additional Information
Plasma
Calcium, Adjusted
Phosphate
Creatinine
Alkaline Phosphatase
PTH
25-OH Vitamin D
2.51 mmol/L
1.00 mmol/L
29 μmol/L
84 U/L
3.3 pmol/L
107 nmol/L
(2.15–2.60)
(0.80–1.50)
(45–90)
(35–135)
(0.7–7.0)
(>50)
Suggested Comment
Low muscle mass and the resultant low creatinine production makes interpretation of the urine creatinine ratios
unreliable. A 24 h urine sample or fractional excretion is suggested to estimate renal calcium losses. A bone
turnover marker in serum or in a 24 h urine sample is recommended in place of N-telopeptide/creatinine ratio.
Rationale
The use of creatinine ratios for analytes in urine is commonly used with the aim of reducing the effect of
hydration on the analyte concentration. This process however introduces an additional variability due to
between-person differences in the rate of creatinine production, which is directly proportional to the muscle
mass of the patient. This process is generally robust but may produce misleading results in patients with
extremes of muscularity.
Subjects with a large muscle mass will produce lower results for urine analytes after creatinine correction and
subjects with low muscle mass will give elevated results. In this case there are three pieces of evidence that the
patient has a low creatinine production and therefore creatinine correction may produce spuriously elevated
results. These factors are the history of muscular dystrophy, the very low serum creatinine and the low urine
creatinine concentration. Of these, the serum creatinine of 27 μmol/L is the strongest indication of the very low
muscle mass.
The key feature in this case is to identify this effect and not proceed with an inappropriate interpretation. This
effect was not recognised in over half the responses with consequent inappropriate interpretations. In order to
obtain the information sought with these requests it is necessary to use tests that are not affected by the
creatinine, such as timed urine collections or serum tests.
PREFERRED KEY WORDS
Low creatinine/muscle wasting
Ratios false 2° to low creatinine
Normal plasma ALP
Suggest 24h urine calcium
Suggest P1NP
Suggest 24h urine NTx
Suggest C-Telopeptide
Suggest serum osteocalcin
Suggest calculate fasting urine Ca
excretion
Suggest serum NTx
Suggest P1CP
LESS RELEVANT KEY
WORDS
Increased bone resorption/loss
High urine NTx/creatinine ratio
Suggest BMD scan
High urine NTx
High calcium/creatinine ratio
Bone resorption/osteoporosis due to
immobilisation
?Osteoporosis
Hypercalciuria
Suggest TSH/TFT
Normal vitamin D/adequate
supplementation.
NTx marker of increased bone resorption
Normal PTH
?Glucocorticoid Rx
?Hyperthyroidism
Normal plasma calcium
Normal plasma phosphate
?Antiresorptive Rx
?Calcium/vitamin D supplements
?Increased NTx 2° circadian
variation/exercise
Suggest sequential bone mineral markers
Results consistent with muscular
dystrophy
?Menstrual variation/menopause
Does not support disuse bone loss
No comment
?Preanalytical/sample collection factors
?Drug effect
Check dietary calcium intake
Collect fasting 2nd void urine NTx
?Hypercalciuria
NTx not useful for osteoporosis diagnosis
?Endocrine disorder
Hyperparathyroidism excluded
Paget’s disease excluded
Bone demineralisation not increased
turnover
Normal calcium homeostasis
Suggest CK
Elevated vitamin D
Low plasma albumin
Suggest bone formation markers
Suggest tests to confirm bone loss
Suggest TRACP5b
UNACCEPTABLE KEY
WORDS
?Hyperparathyroidism
?Metastatic bone disease
Consider increase antiresorptive therapy
Consider hypogonadism if very unwell
Increased risk fractures
Consider genetic cause/lifestyle
?GIT disease
?Pregnancy
Suggest 24h UFC/?Cushing’s
Exclude multiple myeloma
?Paget’s disease
Suggest BALP, ICTP
Refer to endocrine clinic
Case 11-01
Ca/Mg/Phos
Patient ID
79-year-old woman
Patient Location
General Practice
Clinical Notes on Request Form
Osteoporosis. No medication.
Case Details
Fasting serum
Sodium
Potassium
Chloride
Bicarbonate
Urea
Creatinine
Total Bilirubin
ALP
GGT
ALT
Total Protein
Albumin
Globulin
Calcium
Phosphate
Calcium, Adjusted
25OH Vitamin D
PTH
139 mmol/L
4.2 mmol/L
104 mmol/L
23 mmol/L
5.5 mmol/L
67 μmol/L
5 μmol/L
124 U/L
31 U/L
23 U/L
77 g/L
48 g/L
29 g/L
2.8 mmol/L
1.2 mmol/L
2.73 mmol/L
36 nmol/L
4.1 pmol/L
(135–145)
(3.5–5.5)
(95–110)
(20–32)
(3.5–9.5)
(45–90)
(3–15)
(30–115)
(5–35)
(5–40)
(63–80)
(36–47)
(23–39)
(2.15–2.55)
(0.80–1.50)
(2.20–2.60)
(75–250)
(1.6–6.9)
Suggested Comment
Mild hypercalcaemia and non-suppressed PTH is most likely due to primary hyperparathyroidism (PHPT).
Familial hypocalciuric hypercalcaemia (FHH) and lithium therapy can also present with mild parathyroiddependent hypercalcaemia. FHH should be excluded by reviewing previous results and estimation of either
fasting urine calcium excretion or 24 h urine calcium creatinine clearance ratio. Mild vitamin D deficiency is
common with PHPT and may exacerbate bone disease.
Rationale
The degree of hypercalcaemia and the non-suppressed PTH in the context of investigation of osteoporosis
indicate primary hyperparathyroidism is likely. A review of past results would be helpful in this case. Familial
hypocalciuric hypercalcaemia (FHH) is a rare (1:70,000), autosomal dominant inherited condition
characterised by persistent, asymptomatic hypercalcaemia, low urine calcium excretion and inappropriately
normal/ elevated parathyroid hormone due to a mutation of the calcium sensing receptor (CASR) gene.
In the setting of parathyroid dependent hypercalcaemia two tests of urine calcium are helpful. A 24 h urine
calcium clearance ratio involves assessment of serum calcium, serum creatinine, urine calcium and urine
creatinine. A clearance ratio of <0.01 is typical in FHH and >0.02 is common in primary hyperparathyroidism.
Note the clearance ratio [(24-h U-calcium/P- calcium, total)/(24-h U-creatinine/P-creatinine)] is not the same
test as a urine calcium/creatinine ratio. Alternatively, a fasting urine calcium excretion (urinary calcium/
urinary creatinine × serum creatinine) of <30 μmol/L GFR is also consistent with FHH. The differentiation of
FHH is important to prevent unnecessary surgical neck exploration. Since there are many mutations within the
CASR gene, no single simple genetic test is available to exclude this diagnosis.
PREFERRED KEY WORDS
Mild hypercalcaemia
Inadequately suppressed PTH
Mild vitamin D deficiency
?Primary hyperparathyroidism
?Hyperparathyroidism
?FHH
?Drug history (Li or thiazides)
Review previous results
Suggest 24h Ca, Ca/creatinine ratio
LESS RELEVANT KEY
WORDS
Low vitamin D
Hypercalcaemia
Normal PTH
Suggest ionised calcium
Mildly increased ALP
Suggest vitamin D replacement
Repeat calcium
Suggest bone turnover/formation markers
Suggest thyroid function tests
Raised ALP due to bone formation
Repeat PTH
Repeat tests after vitamin D supplement
Repeat collection with no stasis
Increased bone turnover
Suggest protein electrophoresis-serum,
urine
Consistent with osteoporosis
?Ca supplementation
Suggest 1,25 dihydroxy vitamin D
Suggest PTHrP
Suggest bone mineral density scan
?Immobilisation
?Paget’s disease
Moderate vitamin D deficiency
Suggest serum magnesium
?Hyperthyroidism
?Myeloma
Repeat testing
Repeat 25-OH vitamin D
Severe vitamin D deficiency
Contribute to osteoporosis
Normal phosphate
Suggest full blood examination
Not typical vitamin D deficiency
Raised ALP 2° to vitamin D deficiency
?Healing bone fractures
Normal renal function
Albumin elevated
Low vitamin D due to inadequate
sunlight
Repeat test with adequate hydration
?Vitamin D dietary shortage
Suggest parathyroid ultrasound scan
Low vitamin D due to primary PHPT
?Magnesium deficiency
?Transient hypercalcaemia
Suggest genetic studies for FHH
?Osteomalacia
Repeat on fasting specimen
Low vitamin D prevents Ca absorption
?Decreased intestinal absorption vitamin
D
Repeat test before vitamin D supplement
Not consistent with osteoporosis
UNACCEPTABLE KEY
WORDS
?Malignancy
?Sarcoidosis or TB
?Hypercalcaemia age related
Hyperparathyroidism unlikely
PHPT excluded
Suggest renal imaging
Suggest ACE
Suggest chest X-ray
?Acromegaly
Suggest X-rays for bone fractures
?Secondary hyperparathyroidism
?Hyperglycaemia
No vitamin D replacement
?Hypothyroidism
Case 12-02
Ca/Mg/Phos
Patient ID
45-year-old female
Patient Location
General Practice
Clinical Notes on Request Form
No history of fractures or falls.
Strong family history of autoimmune disease and cancer.
Case Details
25OH vitamin D
62 nmol/L
Additional Information
Serum
Creatinine
Calcium
Albumin
Calcium, Adjusted
Parathyroid Hormone
63 μmol/L
2.32 mmol/L
43 g/L
2.30 mmol/L
8.9 pmol/L
(60–105)
(2.15–2.65)
(38–50)
(2.15–2.65)
(1.1–7.2)
Suggested Comment
Borderline raised PTH with intact renal function, unremarkable adjusted total calcium and vitamin D is
unlikely to be pathologically important.
Rationale
Mildly increased parathyroid hormone with unremarkable adjusted total calcium and serum creatinine are
frequent laboratory findings. Current published guidelines by ANZBMS and recent recommendations by the
Institute of Medicine (IOM) both advocate adoption of a target threshold of 50 nmol/L for 25OHD (1, 2). The
IOM committee was concerned about some laboratories reporting higher 25OHD target thresholds and there
was an urgent need for consensus target thresholds reported by laboratories. Whilst a variety of diseases have
been associated with vitamin D deficiency, randomised clinical trial data demonstrating the benefit of vitamin
D supplementation in these groups are lacking (3). Although ionised calcium is more sensitive than adjusted
total calcium in the diagnosis of early primary hyperparathyroidism, the adjusted calcium result makes this
diagnosis unlikely in this case (4).
References
1. Working Group of the Australian and New Zealand Bone and Mineral Society: Endocrine Society of
Australia; Osteoporosis Australia. Vitamin D and adult bone health in Australia and New Zealand: a
position statement. Med J Aust 2005;182:281-5.
2. Slomski A. IOM endorses vitamin D, calcium only for bone health, dispels deficiency claims. JAMA
2011;305:453-6.
3. Manson JE, Mayne ST, Clinton SK. Vitamin D and prevention of cancer – ready for prime time? N
Engl J Med 2011;364:1385-7.
4. Glendenning P. Diagnosis of primary hyperparathyroidism: controversies, practical issues and the need
for Australian guidelines. Intern Med J 2003;33:598-603.
PREFERRED KEY WORDS
Vitamin D normal
Vitamin D deficiency unlikely
Normal calcium level
Mild increase PTH
Normal renal function
?Bisphosphonate therapy
LESS RELEVANT KEY
WORDS
Elevated PTH
Suggest Mg/Phos/ALP
Suggest ionised calcium
Repeat PTH
Suggest 2° hyperparathyroidism
?Malabsorption
?Drug ?anticonvulsants
Repeat serum calcium
?Adequate dietary calcium intake
Suggest repeat Ca/VitD/PTH after
supplement
Repeat vitamin D
Strong family history autoimmune
disease
Suggest coeliac screening
?Early hyperparathyroidism
Mild 2° hyperparathyroidism
Suboptimal vitamin D level
?Normocalcaemic hyperparathyroidism
?Diuretic therapy
?Malnutrition
No reference range given for vitamin D
PTH assay interference to be excluded
Repeat testing 3–6 months
Repeat PTH in 3 months
?Adequate vitamin D
Normal corrected calcium
Suggest renal panel/albumin
?Chronic vitamin D deficiency
Suggest fasting morning PTH
Suggest fasting calcium
Further investigation required
Interpret results with seasons
Consistent with mild physiologic vitamin
D deficiency
Suggest diabetes screening
Suggest liver function tests
Repeat fasting PTH at 8pm
?D2, D3 or both
No formal comment issued
PTH inappropriate for calcium level
Suggest review in 2–3 months
Normal albumin
NOT SUPPORTED KEY
WORDS
Suggest vitamin D supplementation
Increased risk cancer/autoimmune
disease
Vitamin D low
Suggest 24h Ca/PO4 excretion
?Primary hyperparathyroidism
Exclude adenoma/malignancy if
clinically indicated
Due to family history increased vitamin
D beneficial
Suggest thyroid screening
Suggest bone turnover markers
Suggest urinary Ca/creatinine
2° hyperparathyroidism unlikely
Vitamin D indeterminate
?Familial hypocalciuric hypercalcaemia
Exclude MEN I and II
Suggest full blood examination/ESR
Check renal function
Suggest total protein and protein
electrophoresis
Suggest increase sunshine
exposure/dietary intake
Recommend referral to specialist
Suggest uric acid maybe
Suggest calcium supplementation
MISLEADING KEY WORDS
?Bone densitometry
Mildly low PTH
?Patient on vitamin D masking low
vitamin D
Case 13-03
Ca/Mg/Phos
Patient ID
22-year-old male
Patient Location
Hospital in-patient
Clinical Notes on Request Form
In hospital 4 months following head injury.
Case Details
Total Calcium
Albumin
Phosphate
Alkaline Phos
PTH
25OH Vitamin D
2.93 mmol/L
38 g/L
1.46 mmol/L
84 U/L
0.6 pmol/L
111 nmol/L
Additional Information
Sodium
Potassium
Bicarbonate
Urea
Creatinine
eGFR
147 mmol/L (134–146)
3.5 mmol/L (3.4–5.0)
32 mmol/L
(22–32)
8.7 mmol/L (3.0–8.0)
69 μmol/L
(60–110)
>60 mL/min/1.73m2
(2.15–2.60)
(35–50)
(0.8–1.50)
(35–135)
(0.7–7.0)
(>50)
Suggested Comment
Mild hypercalcaemia with suppressed PTH suggests hypercalcaemia of immobilisation given the history and
the age of the patient. Dehydration exacerbates hypercalcaemia. Consider thyroid function tests to exclude
thyrotoxicosis.
Rationale
Immobilisation hypercalcaemia mainly results from rapid bone turnover and may be seen after spinal cord
injury or long bone fracture in children and adolescents with growing bones and after cerebrovascular accident
in the elderly, especially in the presence of high turnover states such as Paget’s disease. The exact mechanisms
of immobilisation hypercalcaemia are unclear, but are not PTH-dependent. The generally accepted explanation
is the loss of mechanical stress (mechanostat theory) leading to increased osteoclastic bone resorption and
decreased bone formation, which are the hallmarks in bone biopsy.
The serum calcium level depends on the rate of bone resorption and the capacity of renal calcium excretion.
The median interval between initiation and immobilisation and onset of hypercalcaemia is 4 weeks, but may be
extended to 16 weeks in patients with normal renal function. In chronic renal insufficiency the interval is
shortened and may range from 3 to 16 days. Hypercalcaemia itself can induce acute impairment of renal
function via glomerular arteriolar vasoconstriction, volume depletion, and nephrocalcinosis and then in turn,
compromise calcium clearance and accentuate the degree of hypercalcaemia. Other possible causes such as
Grave’s disease may be considered, and rarely granulomatous diseases such as sarcoidosis.
Reference
1. Cheng CJ, Chou CH, Lin SH. An unrecognized cause of recurrent hypercalcemia: immobilization.
South Med J 2006;99:371-4.
PREFERRED KEY WORDS
Mild hypercalcaemia
Due to immobilisation
Prolonged hospitalisation
Due to excessive bone turnover
Hyperparathyroidism unlikely
PTH independent/ non-parathyroid
Not vitamin D mediated hypercalcaemia
Slightly increased urea
?Dehydration
Replacement fluid will decrease calcium
?Hyperthyroidism
Suggest thyroid function tests
Suggest ongoing monitoring
Significant increased corrected calcium
LESS RELEVANT KEY
WORDS
Hypercalcaemia
Reduced PTH
Need to exclude malignancy
Raised Na
Suggest 24h urine calcium/ phos
?Due to medications
?Chronic granulomatous disease
Suggest ionised calcium
Suggest 1,25 vitamin D
?Sarcoidosis
Suggest repeat total calcium/PTH
Suggest N-telopeptide/bone markers
?Diabetes insipidus
Suggest urine osmolality/volume/Na
Normal albumin
High/normal phosphate
Normal 25-hydroxyvitamin D
Suggest cortisol
Normal ALP
Suggest ACTH/GH/FSH/LH/prolactin
?Hyperaldosteronism
?Increased renal retention
Low creatinine
Suggest ALP
Suggest serum/urine electrolytes
Elevated serum calcium
Suggest clinical and radiological exam
Suggest magnesium
Renal cause unlikely
Normal creatinine/ eGFR
?Increased 1,25 vitamin D intake
NOT SUPPORTED KEY
WORDS
Suggest PTHrP
?Familial hypocalciuric hypercalcaemia
?Increased vitamin D intake
Suggest ACE
Suggest urine electrophoresis/
immunofixation
25OH vitamin D raised
?Sepsis
Suggest vitamin A
Exclude factitious cause
?Paget’s disease
Suggest water deprivation test
?Rhabdomyolysis
Mild 2° hypoparathyroidism
?Increased intestinal absorption
?Hypothyroidism
?Milk-alkali syndrome
Recommend urgent consultation
Case 14-06
Ca/Mg/Phos
Patient ID
82-year-old female
Patient Location
General Practice
Clinical Notes on Request Form
Presents with back pain.
Case Details
Albumin
Calcium (Total)
Calcium (Adjusted)
Intact PTH
29 g/L
2.60 mmol/L
2.83 mmol/L
3.2 pmol/L
(32–46)
(2.15–2.55)
(2.15–2.55)
(1.3–6.8)
Additional Information
Ionised Calcium
1.27 mmol/L (1.12–1.32)
Sample pH
7.41
(7.35–7.45)
Ion Ca (pH-adjusted) 1.28 mmol/L (1.12–1.32)
Suggested Comment
The significant discrepancy between the ionised calcium measurement and the albumin-adjusted calcium
suggests the possibility of an abnormal globulin binding to calcium; recommend serum and urine protein
electrophoresis to investigate.
Rationale
In normal physiology, approximately half of serum calcium exists in free (ionised) form while the other half is
protein-bound, mainly to albumin. The direct measurement of ionised calcium plays an important role in the
diagnosis of pseudohypercalcaemia. It is an infrequent but important condition in monoclonal gammopathy
(e.g. multiple myeloma and Waldenström macroglobulinaemia), where monoclonal immunoglobulins cause
raised total/albumin-adjusted calcium and normal ionised calcium.
Several method-dependent mechanisms may explain this apparent discrepancy. The paraproteins may act as
carrier proteins in a similar fashion to albumin, causing increased binding of calcium. Paraproteins may also
increase the solid phase of the serum, causing a 'water exclusion' effect and may affect indirect ion-selective
electrode methods. Finally, paraproteins may precipitate during laboratory analysis, affecting photometric
methods.
If ionised calcium measurement is not available, a simple two-time dilution or use of an alternate assay with
different methodology may clarify the situation. Whilst pseudohypercalcaemia is uncommon, the clinical
presentation of back pain and discrepant total calcium compared with ionised calcium justify further
investigation for a possible paraproteinaemia in this case.
References
1. Loh TP, Yang Z, Chong AT, et al. Pseudohypercalcaemia in a patient with newly diagnosed
Waldenstrom macroglobinaemia. Intern Med J 2013;43:950-1.
2. Annesley TM, Burritt MF, Kyle RA. Artifactual hypercalcemia in multiple myeloma. Mayo Clin
Proc 1982;57:572-5.
3. Schwab JD, Strack MA, Hughes LD, et al. Pseudohypercalcemia in an elderly patient with multiple
myeloma: report of a case and review of literature. Endocr Pract 1995;1:390-2.
PREFERRED KEY WORDS
Ionised and adjusted calcium discrepant
Poor correlation between adjusted and
ionised calcium
Increased protein (non-albumin) bound
calcium
?Pseudohypercalcaemia
?Multiple myeloma
?Paraproteinaemia
Suggest serum electrophoresis
Suggest urine Bence-Jones
protein/electrophoresis
Suggest serum free light chains
Suggest immunoglobulins
LESS RELEVANT KEY
WORDS
Normal ionised calcium
Normal parathyroid hormone (PTH)
Hypercalcaemia
Increased serum total/adjusted calcium
?Malignancy
Suggest repeat calcium/PTH for
confirmation
Hypoalbuminaemia
Clinical status/back pain
Suggest full blood count
Suggest renal function tests
Suggest urine calcium
Suggest serum total protein
Suggest electrolytes, urea, creatinine
Normocalcaemia
Suggest erythrocyte sedimentation rate
?Inflammation
Suggest bone marrow aspirate
Suggest beta-2 microglobulin
If confirmed patient should be referred
Check total calcium with different assay
NOT SUPPORTED KEY
WORDS
?Hyperparathyroidism
Suggest phosphate
Suggest vitamin D
Suggest alkaline phosphatase (ALP)
?Drug/medication induced
?Secreting PTH-related peptide
?Exogenous vitamin D
Suggest liver function tests
Consider renal failure
Suggest review clinical history/status
?Familial hypocalciuric hypercalcaemia
?Breast/lung/kidney cancer
Suggest magnesium
?Increased calcium due to tourniquet
time
Suggest bone markers
?Liver disease
Suggest TSH
Suggest fasting calcium
?Chronic disease
Suggest urine protein/albumin
Euparathyroidism
Monitoring of patient calcium level
required
Suggest vitamin D supplementation
Albumin should be corrected too
Hyperparathyroidism unlikely
Exclude recent blood transfusion
MISLEADING KEY WORDS
Suggest bone scan/CT/imaging
Sarcoidosis
?Hyperthyroidism
?Malnutrition
?Renal/GIT loss
?Dehydration
No abnormality
Suggest repeat in 3 months
Not clinically significant hypercalcaemia
Not suggestive of malignancy
GI loss unlikely
Suggest calcitonin
Case 9-04
Urate
Patient ID
55-year-old male
Patient Location
General Practice
Clinical Notes on Request Form
Check-up
Case Details
Serum Urate 0.48 mmol/L (0.20–0.42)
Suggested Comment
Mild hyperuricaemia may be associated with renal failure, high purine diet, ethanol, diuretics and increased
cell turnover states. Hyperuricaemia is associated with an increased risk of gout. Further
investigation/management would be based on context of test request.
Rationale
Factors that increase serum urate include renal failure, high purine diet, ethanol, diuretics and increased cell
turnover states, as well as rare enzyme defects such as phosphoribosylpyrophosphate (PRPP) synthetase
superactivity and hypoxanthine-guanine phosphoribosyltransferase (HPRT) deficiency. A 24-hour urine
collection on a standard diet excluding alcohol and drugs known to affect urate metabolism will allow
differentiation of overproduction of uric acid versus undersecretion, but this is rarely required in routine
practice.
The diagnostic test for gout is the demonstration of urate crystals in joint aspirate. Serum urate is the most
important risk factor for gout and reduction of serum urate in this context is beneficial. However, only a small
minority of people with hyperuricaemia (defined as serum urate >0.42 mmol/L) will develop gout. About 40%
of patients presenting with acute gout have serum urate <0.42 mmol/L, but note that serum urate is a negative
acute phase reactant and may decrease during an acute attack. In absolute terms, at any given level of serum
urate, the risk of gout is the same for men and women.
Hyperuricaemia is associated with insulin resistance, obesity, hypertension and hypertriglyceridaemia. It has
been suggested to be an independent risk factor for cardiovascular disease and for the future development of
hypertension and renal disease. However, the role of serum urate as a marker for cardiovascular risk is not
established and it is not included in risk calculation. There is no conclusive evidence yet to recommend uratereducing therapy in this context.
References
1. Zhang W, Doherty M, Pascual E, et al. EULAR evidence based recommendations for gout. Part I:
Diagnosis. Ann Rheum Dis 2006;65:1301-11.
2. Campion EW, Glynn RJ, DeLabry LO. Asymptomatic hyperuricemia. Risks and consequences in
the Normative Aging Study. Am J Med 1987;82:421-6.
3. Johnson RJ, Kang DH, Feig D, et al. Is there a pathogenic role for uric acid in hypertension and
cardiovascular and renal disease? Hypertension 2003;41:1183-90.
PREFERRED KEY WORDS
Mild hyperuricaemia
?Asymptomatic hyperuricaemia
Hyperuricaemia alone not diagnostic for
gout
?Urate underexcretion
?Urate overproduction
?Renal impairment
?Excess alcohol
?Diuretics/aspirin/drugs
?Increased purine intake
Maintain urate <0.36 mmol/L if on
treatment
Suggest full history/clinical examination
Suggest urea/creatinine/electrolytes
Increased risk of gout
Associated/assess metabolic syndrome
Associated/assess cardiovascular disease
Suggest fasting blood sugar
concentrations
Suggest fasting lipids
Suggest full blood count
Suggest liver function tests
Suggest tests for secondary disease
Suggest monitoring
Risk of urolithiasis
?Obesity
?Hypertension
?Hypothyroid
Consider diet/lifestyle changes
?Abnormal lipid profile
Suggest calcium/phosphate
LESS RELEVANT KEY
WORDS
Hyperuricaemia
?Gout
?Diabetes
?Arthritis
?Hyperparathyroidism
?Primary or secondary disorder
Suggest urate pre/post low purine diet
?Prolonged fasting/stress/exercise
No comment issued routinely
?Lead poisoning
Risk of gout if urate >0.42 mmol/L
?Idiopathic
Urate lowering treatment if clinically
indicated
?Dehydration
Treatment dependent on
overproduction/underexcretion
Suggest urine albumin/pH/microscopy
Urate levels higher in males
Suggest electrophoresis
?Genetic cause
Urate antioxidant properties
?Sarcoidosis
Repeat urate with normal diet/exercise
Acute gout not excluded
Genetic causes less likely
Suggest 24 h urine urate/creatinine
?Psoriasis
?Malignancy/haematological
disorder/chemotherapy
UNACCEPTABLE KEY
WORDS
Repeat urate on fasting sample
Consider diurnal/seasonal variation
Advise urate lowering treatment
Risk of gout if urate >0.32 mmol/L
?Fasting
Suggest HbA1c
Risk of gout if urate >0.60 mmol/L
Low risk gout at this level
?Probable insignificant increase in urate
Consistent with gout
Moderate hyperuricaemia
Suggest rheumatoid factor/anti-nuclear
antibodies
?Vitamin B12 deficiency
?Hypoparathyroid
Case 7-07
Glucose
Patient ID
84-year-old male
Patient Location
Emergency Department
Clinical Notes on Request Form
Collapsed
Case Details
Plasma
Sodium
Potassium
Bicarbonate
Urea
Creatinine
Glucose
CK
cTroponin T
122 mmol/L
3.7 mmol/L
27 mmol/L
7.5 mmol/L
99 µmol/L
0.6 mmol/L
13,800 U/L
0.02 μg/L
(134–146)
(3.4–5.0)
(22–32)
(3.0–8.0)
(60–110)
(3.0–5.4)
(30–190)
(see below)
Additional Information
<0.03
Myocardial damage unlikely
0.03–0.09
May suggest increased cardiac risk in the setting of acute coronary syndrome
≥0.10
Consistent with myocardial infarction if associated with ischaemic symptoms or
ECG changes
Suggested Comment
Profound hypoglycaemia – suggest check insulin, C-peptide, and possible inappropriate sulphonylurea
ingestion. Concomitant hyponatraemia suggests possible adreno-cortical insufficiency. Suggest urine
electrolytes, cortisol, thyroid function tests, endocrine review and pituitary investigations as appropriate.
Marked CK elevation is of likely skeletal muscle origin, possibly due to seizure or compartment syndrome,
although hypothyroidism may be contributory.
Rationale
The cause of collapse in this patient is likely due to hypoglycaemia. For investigation of low glucose, other
samples should have ideally been taken prior to the result, especially for insulin, C-peptide, drug screen
(sulphonylurea and others) and cortisol. Non-suppressed insulin and C-peptide would favour inappropriate
sulphonylurea ingestion and be an indication for drug screen. Insulin administration is possible, in which case
C-peptide would be suppressed. Combination of hypoglycaemia with hyponatraemia suggests possible
adrenocortical insufficiency (1, 2). One should elicit evidence of clinical volume depletion and salt wasting
(urine electrolytes). Clinical endocrine review would corroborate clinical features, discern primary from
secondary adrenal insufficiency, and determine the need for a synacthen test and pituitary investigations. Lownormal potassium and normal bicarbonate favour secondary adrenal insufficiency. Hypothyroidism may be
associated with, and contributory to, hyponatraemia and CK elevation. Elevated CK is disproportionate to the
just-detectable troponin and suggests skeletal muscle aetiology, possibly secondary to compartment syndrome
(if immobile for a long time), seizure (secondary to hypoglycaemia), or hypothyroidism. If clinical probability
of acute coronary syndrome is low, just-detectable troponin should still be regarded as a prognostic factor.
References
1. Frost P, Williams AB. A 57 year old woman admitted to the emergency department with
hyponatraemia and hypoglycaemia. Crit Care Resusc 2000;2:308-9.
2. Diederich S, Franzen NF, Bähr V, et al. Severe hyponatremia due to hypopituitarism with adrenal
insufficiency: report on 28 cases. Eur J Endocrinol 2003;148:609-17.
PREFERRED KEY WORDS
?Exogenous/endogenous insulin
?Oral hypoglycaemic agents
Suggest toxicology screen
Suggest C-peptide and insulin
? Suggest serum and urine sodium and
osmolality
?Adrenal insufficiency/failure
Suggest cortisol/adrenal studies
Suggest thyroid function tests
?Hypopituitarism
Suggest pituitary test(s)
Muscle damage/rhabdomyolysis
At risk of renal failure
LESS RELEVANT KEY
WORDS
Hypoglycaemia
Elevated CK
Hyponatraemia
Review medications / drug history
Suggest repeat/serial troponin T
?Alcohol abuse
Non-cardiac cause
Cardiac injury not excluded
Immediate glucose administration
required
Suggest liver function tests
Suggest clinical endocrine review
?Seizure activity
?Hypothyroidism
Suggest/monitor electrolytes/glucose
Suggest alcohol level
?SIADH
Monitor renal function
?Malignancy
?Liver dysfunction
Excess sodium loss
?Inadequate salt intake
UNACCEPTABLE KEY
WORDS
History of collapse
Non-elevated troponin
Suggest myoglobin
?Artefact
Suggest CK isoenzymes
?Fluid/electrolyte redistribution
?Septicaemia/sepsis
?Physical activity
?Cerebral event
?Diabetic coma
?Renal disease
?Other rare causes
Suggest miscellaneous tests
?Prolonged hypothermia
?Trauma
?Hyperthermia
Normal potassium
Repeat tests to confirm
Suggest muscle autoantibodies
This troponin level uninterpretable
No renal failure
?Congestive cardiac failure
Adequate hydration required
Case 8-01
Glucose
Patient ID
46-year-old woman
Patient Location
General Practice
Clinical Notes on Request Form
Past history of gestational diabetes.
Case Details
75 g oral glucose tolerance test
0 min glucose
5.8 mmol/L
60 min glucose
13.1 mmol/L
120 min glucose
5.6 mmol/L
Additional Information
Previous fasting glucose
6.1 mmol/L
Suggested Comment
Normal glucose tolerance test. Suggest repeat fasting glucose in 12 months’ time.
Rationale
The three main questions on this case are the diagnosis, the tool to use for follow-up (fasting glucose or oral
glucose tolerance test, OGTT) and the timing of any follow-up. With regard to diagnosis there is difficulty
deciding which guidelines to follow. Several professional bodies, including the Royal College of Pathologists
of Australasia, the Australasian Association of Clinical Biochemists and World Health Organization define
impaired fasting glucose (IFG) as a fasting glucose result in the range 6.1 to 6.9 mmol/L. Since 2003, the
American Diabetes Association guidelines have expanded this to 5.6 to 6.9 mmol/L.
Briefly, arguments for the lower cut point include: the higher cut point is associated with lower test sensitivity
for diabetes, the narrower biochemical definition of IFG meant poorer test reproducibility (i.e. patients are
more likely to be differently classified on different occasions) as the within-individual biological variation for
glucose is relatively large, lower IFG cut point may allow more patients with undiagnosed diabetes to be
subjected to OGTT, in certain studies using receiver operating curve analyses, lower IFG cut points of 5.2–5.7
mmol/L are optimal for predicting diabetes risk. Conversely, arguments against the lowering of cut point
include: marked increase in the prevalence of IFG in adult population (30–40% vs 7–10%), poorer risk
prediction for diabetes, poorer risk prediction for all-cause and cardiovascular mortalities and the potentially
enormous public health costs to manage the significantly increased number of patients classified as IFG. The
excellent point-counterpoint arguments can be found in the references and are highly recommended readings.
With this conflict between guidelines laboratories must choose which is more appropriate for use. The
assessment of this case has been made using the local guidelines on the grounds that most clinicians would be
more familiar with them. The local guidelines support following up testing with fasting plasma glucose (rather
than OGTT) at one year for patients at high risk, which is appropriate as this patient has a recent result
indicating IFG as well as a history of gestational diabetes.
References
1. Shaw JE, Zimmet PZ, Alberti KG. Point: impaired fasting glucose: The case for the new American
Diabetes Association criterion. Diabetes Care 2006;29:1170-2.
2. Dekker JM, Balkau B. Counterpoint: impaired fasting glucose: The case against the new American
Diabetes Association guidelines. Diabetes Care 2006;29:1173-5.
PREFERRED KEY WORDS
LESS RELEVANT KEY
WORDS
Normal glucose tolerance
Normal fasting blood glucose
Not diabetic
Previous impaired fasting glucose results
increase risk
History of gestational diabetes increases
risk
Retest glucose at 1 year
Impaired fasting glycaemia
Suggest repeat OGTT 1–2 years
Measure glucose (other than at 1 year)
Fasting blood glucose results increase
risk
High 1h glucose
Normal 2h glucose
Monitor patient
Lifestyle modification suggested
Clinical evaluation/history required
Previous impaired fasting glycaemia
1h glucose result increases risk
Elevated fasting blood glucose
Suggest renal function tests
Indeterminate results
Repeat OGTT <12 months
Suggest assess cardiovascular risk factors
1h glucose not interpretable
Diabetes unlikely
?Rapid gastric emptying
?Hyperthyroidism
?Abnormal glucose metabolism
Early glucose intolerance
Suggest lipids
Equivocal fasting blood glucose
?Patient fasted correctly
Consider insulin resistance
Adequate glucose absorption
UNACCEPTABLE KEY
WORDS
Impaired glucose tolerance
Suggest HbA1c
1h glucose indicates diabetes
Suggest insulin measurement
Suggest repeat OGTT 3 years
Suggest thyroid function tests for
?hyperthyroidism
Case 9-03
Glucose
Patient ID
45-year-old male
Patient Location
Emergency Department
Clinical Notes on Request Form
Dizziness, sweating and confusion. Not on any medication.
Case Details
Serum glucose
2.5 mmol/L
(3.0–5.4)
Additional Information
Sodium
141 mmol/L (134–146)
Potassium
4.0 mmol/L (3.4–5.0)
Bicarbonate
27 mmol/L
(22–32)
Urea
5.2 mmol/L (3.0–8.0)
Creatinine
72 μmol/L
(60–110)
eGFR
>60 mL/min/1.73m2 (>60)
Total protein
83 g/L
(60–80)
Albumin
48 g/L
(35–50)
Calcium
2.35 mmol/L (2.15–2.60)
Bilirubin
10 μmol/L
(<20)
Alk. Phos.
63 U/L
(35–135)
ALT
44 U/L
(< 40)
Gamma GT
38 U/L
(< 60)
Suggested Comment
The symptoms are consistent with the neurological manifestation of hypoglycaemia (neuroglycopaenia), which
is supported by the low serum glucose. Adult hypoglycaemia presentations to emergency departments are most
commonly due to complications of diabetes therapy or alcohol abuse. If aetiology is uncertain, useful tests may
include insulin, C-peptide, cortisol and sulphonylurea screen. The latter three may be added to the original
sample, if sufficient volume remains.
Rationale
The presenting symptoms and low serum glucose in this case are consistent with the neuroglycopaenia. An
important confirmatory factor is the resolution of the symptoms with administration of glucose (Whipple’s
triad). Once confirmed, investigation of the hypoglycaemia will depend on the history and relative likelihood
of possible causes. In the Emergency Department, diabetes therapy (most importantly, insulin and
sulphonylurea) and alcohol account for the majority of hypoglycaemia presentations. However, the history in
this case indicates no medications.
Renal and hepatic failures can cause hypoglycaemia, and are excluded by the normal renal and liver function
tests. Other causes include adrenal failure, insulin-secreting tumour and factitious hypoglycaemia, which are
relatively rare but may also be important. The choice of tests will be directed by the clinical picture. Another
important consideration is the need to obtain appropriate samples at the time of a hypoglycaemic episode for
some tests. It may be useful to take this opportunity to investigate this case further, especially if there have
been previous episodes. Insulin, C-peptide, cortisol and sulphonylurea screen may be the best first-line tests
during an episode, with additional testing later if indicated. Note that sulphonylurea screens are performed on
serum and not urine samples.
PREFERRED KEY WORDS
Symptomatic hypoglycaemia
Marked hypoglycaemia
?Hypoglycaemic agents
Iatrogenic causes most common
?Drug-induced
?Exogenous insulin
?Insulinoma
?Adrenal insufficiency
Test during hypoglycaemic episode
Suggest insulin
Suggest C-peptide
?Alcohol
Suggest insulin if clinically indicated
Suggest C-peptide if clinically indicated
Suggest sulphonylurea screen if clinically
indicated
Suggest sulphonylurea screen
Suggest cortisol
Suggest pituitary and adrenal tests
Suggest further investigation
LESS RELEVANT KEY
WORDS
Hypoglycaemia
Suggest repeat glucose
?Reactive hypoglycaemia
?Fasting or postprandial glucose
?Early diabetes
Other biochemistry normal
?Endocrine disorder
History of no medications noted
Request detailed history
?Prolonged fasting
Suggest glucose replacement
Suggest OGTT (extended) if clinically
indicated
Suggest investigate with prolonged fast
Marginally elevated ALT/total protein
?Functional hypoglycaemia
Fluoride oxalate collection
Suggest OGTT
Exclude artefactual causes
Suggest growth hormone
?Factitious hypoglycaemia
?Symptom recovery with normal blood
sugar level
Suggest drug screen
Suggest TSH
Suggest ACTH
Investigate diet
?Post-gastrectomy
?Pancreatic tumour
?Extrapancreatic tumours
Artefact unlikely
Consider imaging
Insulinomas are rare
Suggest insulin antibodies
?Septicaemia
Differentiate endogenous vs exogenous
insulin
Suggest IGF/IGFII/IGFBP3
No investigation if single episode
?Diarrhoea/vomiting
?Islet cell hyperplasia
?Overdose hypoglycaemics/ ?noncompliant
?Immune hypoglycaemia
Hypoglycaemia 2° to liver disease
unlikely
Suggest short synacthen if clinically
indicated
Specialist opinion may be required
Suggest investigate with 24 h fast
?Endocrine neoplasms
Pituitary and adrenal disease less likely
UNACCEPTABLE KEY
WORDS
Suggest urine hypoglycaemic agent
screen
?Galactosaemia
?Glycogen storage disease
?Dehydration
Suggest hepatitis serology
Suggest catecholamines
Insulinoma is a common cause
?Liver/renal failure
Suggest glucagon measurement
?Leucine/fructose
Suggest DHEAS
Suggest proinsulin
Suggest electrolytes
Nil
Suggest ketones
Mild hypoglycaemia
Suggest lactate
Insulin overdose excluded
Case 10-09
Glucose
Patient ID
2-year-old boy
Patient Location
Emergency Department
Clinical Notes on Request Form
History of convulsions soon after awakening
Case Details
Results from the Emergency Department
Plasma glucose
1.7 mmol/L (Fasting: 3.5–5.5)
Urine ketones
4+
Calcium
3.37 mmol/L
Suggested Comment
These results are consistent with a diagnosis of idiopathic ketotic hypoglycaemia. Other endocrine, metabolic
and toxicological causes should be excluded with tests collected whilst the patient is hypoglycaemic. These
should include electrolytes, creatinine and liver function tests, cortisol, growth hormone, insulin, lactate, betahydroxybutyrate, free fatty acids, blood gases, ammonia, organic acids and acylcarnitines. The raised ketones
make insulin excess and fatty acid oxidation defects less likely.
Rationale
Idiopathic ketotic hypoglycaemia (IKH) is a condition characterised by fasting hypoglycaemia and increased
concentrations of ketones. By contrast, ketones are usually low in children with fatty acid oxidation defects or
those who have insulin-secreting tumours or those who have ingested sulphonylureas. IKH is the most likely
cause of the findings in this case, although careful history and testing are needed to exclude other causes. IKH
usually occurs after 12 months of age and resolves over several years.
In a recent case series, IKH is the most common cause of hypoglycaemia in previously healthy children
presenting to the Emergency Department. They usually first present before the age of 5 years with
symptomatic hypoglycaemia, which resolves after administration of glucose, and ketonuria during the morning
hours after a moderate fast. They are more likely to be Caucasian, male gender, and have a low body weight.
The underlying pathophysiology of IKH is yet to be fully understood. It is often associated with low plasma
concentrations of alanine, an important substrate for gluconeogenesis, and may involve impaired ketone body
metabolism or transport. It is sometimes called ‘accelerated starvation’ and is more common in babies that
were small for dates and had neonatal hypoglycaemia.
References
1. Daly LP, Osterhoudt KC, Weinzimer SA. Presenting features of idiopathic ketotic hypoglycemia. J
Emerg Med 2003;25:39-43.
2. Marcus C, Alkén J, Eriksson J, et al. Insufficient ketone body use is the cause of ketotic
hypoglycemia in one of a pair of homozygotic twins. J Clin Endocrinol Metab 2007;92:4080-4.
PREFERRED KEY WORDS
LESS RELEVANT KEY
WORDS
Idiopathic ketotic hypoglycaemia
Ketotic hypoglycaemia
Exclude inborn errors of metabolism
?Drug ingestion ?medications ?overdose
?reaction
?Adrenal insufficiency ?Addison’s
?Hypopituitarism
?Hypocortisolism
Ketosis, fatty acid oxidation disorders
less likely
Collect samples when hypoglycaemic
Suggest insulin/C-peptide
Suggest cortisol/ACTH
Suggest growth hormone
Suggest lactate
Suggest urine/plasma organic acids
Suggest free fatty acids - exclude fatty
acid oxidation disorders
Suggest amino acid screen
Suggest carnitine/acylcarnitine screen
Suggest electrolytes/renal function
tests/liver function tests/glucose/Ca/Mg
Suggest β-hydroxybutyrate/urine ketones
Suggest blood gas/pH
Suggest urine metabolic screen
Suggest ammonia
?Growth hormone deficiency
Suggest urine amino acids
Urgent oral/IV glucose/admit hospital
Repeat glucose regularly
Marked hypoglycaemia
Ketosis/ketonuria
Exclude endogenous/exogenous insulin
excess
?Malnutrition ?starvation ?long fast
Hypoglycaemia
Suggest thyroid function tests
?Glycogen storage disease
?Salicylate/paracetamol/alcohol overdose
Refer to endocrinologist/specialist
?Gluconeogenic defect
?Organic acidaemia
?Liver disease ?hepatic enzyme defect
Suggest supervised diagnostic fast
?Sepsis – complete blood
picture/blood/urine cultures
Ketosis - hyperinsulinaemia less likely
Exclude MCAD
?Family history
Ketosis excludes hyperinsulinaemia
Assume correct specimen collection
Review original Guthrie screen
Exclude respiratory chain disorders
?Galactosaemia
?Fructose intolerance
?Leucine sensitivity
?Reye’s syndrome
UNACCEPTABLE KEY
WORDS
?Intercurrent illness
?Hormone deficiency
?Insulin dependent diabetes mellitus
Suggest urine reducing substances
Convulsion from hypoglycaemia
?Pyruvate metabolism disorder
Suggest IGF-1
?Tissue biopsy for inborn errors of
metabolism investigation
Suggest glucagon
Suggest HbA1c
Suggest urine microalbumin
?Adrenoleukodystrophy or congenital
adrenal hyperplasia
Suggest 17OHP
Suggest genetic test if amino acid
positive
Suggest G6PD
Risk of mental retardation/seizures
?Low weight for height
?Abdominal mass
?Neoplasm
Ketosis excludes fatty acid oxidation
disorders
Case 11-10
Glucose
Patient ID
31-year-old female
Patient Location
General Practice
Clinical Notes on Request Form
28/40 pregnant.
OGTT (75 g oral glucose load)
Case Details
Fasting glucose
1 h glucose
2 h glucose
5.1 mmol/L
10.3 mmol/L
5.3 mmol/L
Suggested Comment
The latest Australian Diabetes in Pregnancy Society definitions for gestational diabetes (GD) using 75 g oral
glucose tolerance test (OGTT) are: fasting glucose <5.1 mmol/L, 1 hour <10.0 mmol/L and 2 hour <8.5
mmol/L. Under these guidelines, this patient should be managed as for GD, including an OGTT 6–8 weeks
post-partum.
Rationale
Previously, the lack of international consensus on the diagnosis of gestational diabetes (GD) makes it difficult
to be sure what criteria to apply. In 1998 the Australian Diabetes in Pregnancy Society re-affirmed 1991
recommendations of the Australian Diabetes Society to define GD as a fasting blood sugar level (BSL) of ≥5.5
mmol/L or 2 hour post 75 g load BSL of ≥8.0 mmol/L; the 2 hour threshold was derived by rounding the WHO
level of 7.8. In 1992 the New Zealand Society for the Study of Diabetes raised the 2 hour threshold to 9.0
mmol/L for New Zealand. The American Diabetes Association favours the 3 hour 100 g OGTT but also use
two abnormal readings [≥5.3 mmol/L (fasting), ≥10.0 mmol/L (1 hour) and ≥8.6 mmol/L (2 hour)] post 75 g
load.
After publication of the Hyperglycemia and Adverse Pregnancy Outcome (HAPO) Study, which conclusively
links multiple adverse pregnancy outcomes with degrees of hyperglycaemia less severe than those diagnostic
of diabetes, the International Association of Diabetes and Pregnancy Study Groups recommended a uniform
approach to GD diagnosis.
The proposal includes:
1. Use of random/fasting BSL or HbA1c in 1st trimester to exclude undiagnosed overt diabetes.
2. Diagnosis of GD based on either BSL ≥5.1 mmol/L (fasting), 10.0 mmol/L (1 hour) or 8.5 mmol/L (2 hour)
post 75 g load at 24–28 weeks.
These figures represent the average BSL exceeded in HAPO where the odds ratio for adverse outcomes
reached 1.75. While not directly addressed, the non-fasting ‘screening’ test will be redundant. These criteria
are now adopted by the Australian Diabetes in Pregnancy Society and World Health Organisation.
Reference
1. International Association of Diabetes and Pregnancy Study Groups Consensus Panel, et al.
International Association of Diabetes and Pregnancy Study Groups recommendations on the
diagnosis and classification of hyperglycemia in pregnancy. Diabetes Care 2010;33:676-82.
PREFERRED KEY WORDS
LESS RELEVANT KEY
WORDS
UNACCEPTABLE KEY
WORDS
Raised 1h result
Gestational diabetes by IADPSG criteria
Gestational diabetes
Suggest manage patient as gestational
diabetes mellitus
Pregnancy OGTT guidelines differ
Continue to monitor post-partum
Repeat OGTT post-partum
75 g glucose load
Repeat GTT if clinically indicated
Glucose tolerance can deteriorate in later
pregnancy
Suggest further tests if risk factors
Check specimen integrity and order
Guidelines suggest fasting plasma
glucose, HbA1c 1st trimester
Increased risk of diabetes mellitus
Interpret with caution during pregnancy
Review of gestational diabetes mellitus
criteria from HAPO
?Patient fasting
Check pre-test conditions
1h suggests normal absorption of glucose
Repeat GTT 3rd trimester if clinically
indicated
If non-fasting, positive 1h result
Raised 1h result ?rapid absorption
Confirm gestational diabetes mellitus
diagnosis
Suggest polycose screen (GCT)
Gestational diabetes mellitus indicated by
ADA criteria
GTT protocol not followed
Gestational diabetes mellitus using
ADIPS 1998
Normal GTT
Normal GTT using ADIPS criteria
Not suggestive of gestational diabetes
mellitus
Normal GTT using WHO criteria
No gestational diabetes mellitus using
ADA 1999
Case 12-07
Glucose
Patient ID
42-year-old male
Patient Location
General Practice
Clinical Notes on Request Form
Diabetes monitoring
Case Details
HbA1c
HbA1c (IFCC)
6.8%
51 mmol/mol
Additional Information
Type 2 diabetes for 5 years. On metformin.
Suggested Comment
As the general target for HbA1c in a patient with type 2 diabetes for 5 years on metformin therapy alone is
6.0% (42 mmol/mol), the management of this patient should be reviewed. If the patient has clinical
cardiovascular disease a higher target may be appropriate.
Rationale
The Australian Diabetes Society has produced guidelines for the individualisation of HbA1c targets based on a
range of factors (1, 2). Factors to consider with these individualised targets include the type of diabetes (type I
or type II), the duration of diabetes, treatment, presence of cardiovascular disease, pregnancy, recurrent
hypoglycaemia and hypoglycaemia unawareness.
In the supplied case, a number of these factors were specified (type II diabetes mellitus, duration five years,
metformin only) for which the target HbA1c is <6.0% (<42 mmol/mol). A higher value may be appropriate in
this patient if there is clinical cardiovascular disease. It is difficult for the laboratory to be aware of all these
factors when issuing an interpretive comment, but care should be taken not to place a comment which conflicts
with these guidelines. Participants in countries other than Australia should refer to relevant guidelines in their
own country.
Responses suggesting a specific course of action were generally marked as ‘Less Relevant’. From the
laboratory, it is not possible to determine whether changes in lifestyle, metformin dose or drug treatment is the
preferred option.
References
1. Cheung NW, Conn JJ, d’Emden MC, et al. Position statement of the Australian Diabetes Society:
individualisation of glycated haemoglobin targets for adults with diabetes mellitus. Med J Aust
2009;191:339-44.
2. Jones GR, Barker G, Goodall I, et al. Change of HbA1c reporting to the new SI units. Med J Aust
2011;195:45-6.
PREFERRED KEY WORDS
LESS RELEVANT KEY
WORDS
Results above limits 6% or 42 mmol/mol
Short term diabetes mellitus without
cardiovascular disease target <6%, 42
mmol/mol
Short term DM without CVD target 6–
6.5%
HbA1c target 6–6.5% (45–49 mmol/mol)
If cardiac risk target <6.5%
If long-term or cardiovascular disease,
target <7%, 53 mmol/mol
Targets should be individualised
Need to lower HbA1c levels
Recommend lifestyle changes ±
metformin target <6%
Suggest repeat HbA1c in three months
Refer Aust. Diabetes Society guidelines
Refer Med J Aust 2011 position
statement
Refer Med J Aust 2009 position
statement
Refer NZ guidelines Type 2 DM (2011)
HbA1c general target <7% or 53
mmol/mol
Check lipids, urine albumin/creatinine,
renal function
Dual reporting of units
Check other cardiovascular risk factors
Recommend annual urine
albumin/creatinine, lipids
Early control lowers microvascular/
macrovascular complications
ADA, BPAC NZDA, IDF target 6.5%
Recommend lifestyle changes
HbA1c 5–6% reduces cardiovascular
disease
?Variant Hb produce artefacts
?Compliance
Repeat HbA1c in six months
Consider hypoglycaemic episodes
?Modify metformin dose
Recommend lifestyle changes (<6% or
<42 mmol/mol)
?Lower target with new diabetics
?Consider lower target no cardiovascular
disease
Reporting IFCC units
UKPDS early treatment of DM beneficial
Check fasting glucose
HbA1c <1% above reference limit 4–6%
Suggest microalbumin
Check for hypertension
?Suggest sulfonylurea or insulin
Not using IFCC units
Care required when interpret increased
red blood turnover
Consider patient symptoms
?Medication to reduce symptoms
Patient risk of retinopathy
Consider oral anti-diabetic therapy
Follow-up 3–6 months
Refer Clin Chem guidelines 2011
Review of diabetes management
suggested if HbA1c =>7.0%
Target for type 2 DM 50–55 mmol/mol
For younger people tighter control
required
NOT SUPPORTED KEY
WORDS
Good glycaemic control
Poor diabetic control
Targets balanced against risk of
hypoglycaemia
?Risk hypoglycaemia 2° to treatment
Change treatment if >8.0% (64
mmol/mol)
Should achieve <6% accord to UKPDS
MISLEADING KEY WORDS
Very good glycaemic control
No change medication/dosage require
Case 14-01
Glucose
Patient ID
66-year-old female
Patient Location
General Practice
Clinical Notes on Request Form
Previous impaired fasting plasma glucose (FPG). Normal body mass index (BMI) and no family
history of diabetes mellitus.
Case Details
HbA1c (IFCC)
Fasting plasma glucose
48 mmol/mol
6.4 mmol/L
Additional Information
Previous FPG as part of a screening for being a possible live kidney donor showed a FPG of 6.1
mmol/L. This was followed up by an HbA1c and an oral glucose tolerance test (OGTT):
HbA1c (IFCC)
46 mmol/mol
OGTT 0 hour glucose
5.2 mmol/L
2 hour glucose
5.5 mmol/L
All haematology was normal.
Suggested Comment
(Australia) Single HbA1c result above the diagnostic threshold for diabetes mellitus together with a fasting
blood glucose in the range for impaired fasting glucose. A repeat HbA1c test in 3 months is recommended for
confirmation of diagnosis of diabetes. (New Zealand) HbA1c result in the pre-diabetic range. Repeat testing in
6–12 months is recommended to assess progression.
Rationale
HbA1c is now approved for the diagnosis of diabetes mellitus (DM) in Australia (1) and New Zealand (2),
providing it is measured in an accredited laboratory using a National Glycohemoglobin Standardization
Program (NGSP)-certified method. Briefly, in Australia, the WHO criterion (3) of two results ≥48 mmol/mol
(6.5%) in an asymptomatic patient has been adopted. In New Zealand the criterion is two results ≥50
mmol/mol (6.7%) for asymptomatic patients. Either venous plasma glucose or HbA1c are valid for the
diagnosis of DM and if either testing protocol is positive the diagnosis is made. It is not recommended that
both glucose and HbA1c testing protocols be used in the same patient at the same time. Doctors may choose
HbA1c to test patients who are not fasting.
HbA1c should not be used in patients with factors that make testing unreliable including iron deficiency, stage
4 or 5 chronic kidney diseases, haemolytic anaemia or other cause of reduced red blood cell survival and
haemoglobinopathies that affect the laboratory method in use. HbA1c should not be used for diagnosis of acute
onset DM (e.g. Type 1) or for gestational DM. While HbA1c is a valid test for diabetes in Australia, details on
recommended testing protocols (e.g. retesting frequency) are still under development. In contrast, in New
Zealand HbA1c is recommended as the preferred test unless there are specific contraindications.
In this case, where results are available for fasting plasma glucose, oral glucose tolerance test and HbA1c,
unless there are specific contraindications it would seem that HbA1c is likely to be the more sensitive test for
diagnosis and should be used for further assessment.
References
1. d'Emden MC, Shaw JE, Colman PG, et al. The role of HbA1c in the diagnosis of diabetes mellitus in
Australia. Med J Aust 2012;197:220-1.
2. Braatvedt GD, Cundy T, Crooke M, et al. Understanding the new HbA1c units for the diagnosis of
Type 2 diabetes. NZ Med J 2012;125:70-80.
3. World Health Organization. Use of Glycated Haemoglobin (HbA1c) in the Diagnosis of Diabetes
Mellitus. 2011. http://www.who.int/diabetes/publications/report-hba1c_2011.pdf
PREFERRED KEY WORDS
Borderline HbA1c
Not diagnostic of diabetes
(Aus) DM cut-off HbA1c ≥48 mmol/mol
Oral glucose tolerance test (OGTT) and
HbA1c discordant
NZSSD position statement prediabetic
Consistent with pre-diabetes
Fasting plasma glucose (FPG) consistent
with impaired fasting glucose
HbA1c above diabetes mellitus (DM)
cut-off
High risk of developing diabetes
Recommend lifestyle review
Recommend cardiovascular disease risk
assessment
Use repeat HbA1c to confirm DM
Repeat screening HbA1c in future
Suggest lipid profile
Confirm DM if asymptomatic
Re-screen DM in future
OGTT or HbA1c results diagnose DM
HbA1c - WHO criteria for Type II DM
Assuming valid HbA1c
LESS RELEVANT KEY
WORDS
Normal OGTT
Clinical/previous history noted
Re-screen in future with OGTT
Re-screen in future with FPG
Australian guidelines confirm DM
FPG and HbA1c discordant still
diagnostic
HbA1c not funded for diagnosis of DM
Check urine albumin/creatinine ratio
?Technical problem with OGTT
Suggest fasting glucose
Australian Diabetes Society HbA1c
targets for diabetics
Impaired fasting glucose consistent with
insulin resistance
HbA1c better diagnostic tool
Confirm DM with OGTT
NOT SUPPORTED KEY
WORDS
Impaired glucose tolerance
HbA1c - impaired glucose tolerance
HbA1c not supported in Australia for
diagnosis
MISLEADING KEY WORDS
Normal HbA1c level
Repeat HbA1c to monitor DM
Case 6-09
Liver
Patient ID
29-year-old man
Patient Location
General Practice
Clinical Notes on Request Form
Acutely ill with nausea and vomiting
Case Details
Plasma Liver Function Tests
Albumin
28 g/L
Bilirubin
61 µmol/L
ALT
2627 U/L
AST
2196 U/L
Alk Phos
69 U/L
Gamma GT 198 U/L
(35–50)
(<20)
(<46)
(<33)
(35–135)
(<44)
Additional Information
Other laboratory results: Urea and electrolytes unremarkable
Suggested Comment
Results indicate acute hepatitis. Consider viral hepatitis serology, possible adverse drug reactions, and exclude
paracetamol or other toxic substances.
Rationale
It is a good general rule to go for the obvious and gross when looking at results. In this case the transaminases
are very high indicating acute hepatitis, and other minor abnormalities are likely to be secondary to this.
The causes of ALT >1000 are few - viral hepatitis, toxins such as paracetamol, adverse drug reactions,
ischaemic hepatitis secondary to hypotension, and ante-mortem with severe heat stroke. In a man of this age
and with the history given, the latter two can probably be ruled out. In this patient, viral hepatitis and adverse
drug reactions of one sort or another are highly likely. Of note, an early and rapid rise of LDH, AST:LDH ratio
of <1.5 and rapid fall in AST after the initial rise are features suggestive of ischaemic liver injury rather than
viral hepatitis.
Remember that drug screens and paracetamol concentration may be difficult to interpret, as it may be some
time since ingestion and the drug may be nearly absent or at low concentration.
PREFERRED KEY WORDS
Acute hepatitis
Hepatocellular pathology
?Drugs/chemicals
?Viral infection
Suggest viral serology
Suggest paracetamol levels
LESS RELEVANT KEY
WORDS
Acute insult
Elevated AST/ALT
?Ethanol
?History
Increased GGT/bilirubin
Abnormal liver function tests
Suggest INR/clotting studies
?Other causes
Mild jaundice
Suggest ethanol level
?Autoimmune disease
Suggestive of hepatitis A
Suggest toxicology screen
No obstruction
AST/ALT indicate non-alcoholic
Suggest glucose
Hospitalisation/management
recommended
UNACCEPTABLE KEY
WORDS
Acute biliary obstruction
?Hypoxic insult
Suggest other tests
Suggest radiography
Suggest serial liver function tests
Suggest lipase/amylase
?Haemolysis
?Gilberts
?Hepatic failure
?Extensive trauma
?Cholecystitis
Viral hepatitis unlikely
Screen contacts
Case 7-02
Liver
Patient ID
49-year-old man
Patient Location
General Practice
Clinical Notes on Request Form
History of gout. Family history of Type II diabetes and cardiovascular disease.
Case Details
Total protein
Albumin
Bilirubin
ALT
AST
ALP
GGT
70 g/L
44 g/L
11 µmol/L
48 U/L
22 U/L
89 U/L
79 U/L
Additional Information
Sodium
143 mmol/L
Potassium
4.2 mmol/L
Urea
4.2 mmol/L
Creatinine
80 µmol/L
Glucose
5.8 mmol/L
Urate
0.66 mmol/L
Cholesterol 7.9 mmol/L
Triglyceride 4.5 mmol/L
(60–80)
(35–50)
(<20)
(<40)
(<43)
(35–135)
(<60)
(134–146)
(3.4–5.0)
(3.0–8.0)
(50–110)
(3.0–5.4)
(0.12–0.45)
(<5.5)
(<2.01)
Suggested Comment
Mild elevations of ALT and GGT in the setting of mixed hyperlipidaemia and impaired fasting glycaemia
suggest possible fatty liver disease. Alcohol and drugs should be considered as possible contributory factors. If
history is supportive, hepatitis serology may be indicated.
Rationale
Non-alcoholic fatty liver disease (NAFLD) should be considered in patients with asymptomatic elevated
aminotransaminases. Liver enzyme elevations are absent in 78% of NAFLD patients, and when present are
modest, with AST:ALT ratio <1. GGT when raised is less than in alcohol-induced liver injury. The diagnosis
of NAFLD requires 1) imaging or histologic evidence of fatty infiltration of the liver, 2) the absence of
excessive alcohol ingestion (men: <21 drinks/week; women <14 drinks/week), 3) no competing aetiologies for
hepatic steatosis, and 4) no co-existing cause of chronic liver disease. Obesity, type 2 diabetes mellitus,
hypertension, hypertriglyceridaemia, low HDL-cholesterol and metabolic syndrome are risk factors associated
with NAFLD. Competing causes of hepatic steatosis, such as hepatitis C, medications, parenteral nutrition,
Wilson’s disease, severe malnutrition as well as other causes of chronic liver disease should be excluded by
clinical evaluation and serological testing. Confirmation of diagnosis is by imaging studies, although liver
biopsy should be considered in patients with metabolic syndrome, high NAFLD Fibrosis Score, and in whom
competing aetiologies or chronic liver disease cannot be excluded without a biopsy.
Reference
1. Chalasani N, Younossi Z, Lavine JE, et al. The diagnosis and management of non-alcoholic fatty
liver disease: practice Guideline by the American Association for the Study of Liver Diseases,
American College of Gastroenterology, and the American Gastroenterological Association.
Hepatology 2012;55:2005-23.
PREFERRED KEY WORDS
Mild liver enzyme abnormalities
?Alcohol and/or drugs
?Fatty liver disease
?Metabolic syndrome
Suggest hepatitis serology
LESS RELEVANT KEY
WORDS
Hyperuricaemia
Hyperlipidaemia
Recommend oral glucose tolerance test
Recommend repeat fasting lipids & HDL
Hyperglycaemia
Consistent with gout
?Fasting
Suggest repeat fasting glucose
Suggest lifestyle changes
?Viral hepatitis
Suggest repeat tests later
Suggest further liver investigation
?Hepatocellular injury
Suggest clinical review
At cardiovascular risk
?Obesity
Suggest urinalysis/ urine microalbumin
?Insulin resistance
Review cardiac risk factors
Recommend HbA1c
Suggest other tests
Normal renal function tests
Suggest insulin levels
Suggest further lipid tests
Recommend liver imaging
UNACCEPTABLE KEY
WORDS
?Diabetes mellitus
Suggest statin/pharmacotherapy
Suggest thyroid function tests
Familial hypercholesterolaemia
?Increased cell turnover/malignancy
?Pancreatitis
Hepatic impairment
No comment
Case 9-06
Liver
Patient ID
25-year-old male
Patient Location
General Practice
Clinical Notes on Request Form
135 kg, bipolar on Epilim (sodium valproate) 2 x 200 mg BD
Case Details
Bilirubin
ALP
AST
ALT
GGT
LD
12 µmol/L
81 U/L
193 U/L
124 U/L
21 U/L
238 U/L
Additional Information
Epilim
22 mg/L
(3–20)
(30–115)
(5–40)
(5–40)
(5–65)
(100–225)
(50–100)
Suggested Comment
Hepatocellular dysfunction may be related to valproate toxicity even though the valproate concentration is
below the usual therapeutic range. Other causes of liver disease should be investigated. The disproportionate
increase in AST compared with ALT may be seen in ethanol-associated liver disease or with concurrent
muscle injury. Measurement of serum CK may be helpful.
Rationale
Initial and chronic ingestion of valproate may cause dose-dependent, reversible minor elevation of hepatic
transaminases. On the other hand, valproate can cause fatal idiosyncratic hepatotoxic reactions. Anyone can be
affected but children under 2 years old with organic brain disease, congenital metabolic disorders,
developmental delay and taking multiple medications may be at increased risk. The idiosyncratic
hepatotoxicity may not be related to increased serum valproate concentration. Therapeutic drug monitoring of
valproate is less useful than other anticonvulsants like phenytoin because of a weaker correlation between
serum concentrations and therapeutic effect or toxicity.
Reference
1. Björnsson E, Olsson R. Suspected drug-induced liver fatalities reported to the WHO database. Dig
Liver Dis 2006;38:33-8.
PREFERRED KEY WORDS
Consistent with hepatocellular damage
Epilim hepatotoxicity
Sub-therapeutic Epilim level
Exclude other causes of liver disease
Assess patient clinically
?Alcohol
?Muscle involvement
Suggest CK/CK-MB
Epilim may cause acute liver failure
Review pre-Epilim liver function tests
LESS RELEVANT KEY
WORDS
Raised liver transaminases noted
Monitor liver function tests
?Concurrent drug therapy
?Viral hepatitis
?Non-alcoholic fatty liver disease
Review Epilim dose
Obesity noted
Suggest PT/fibrinogen levels
Slightly raised LDH
?Non-compliance
Normal GGT/ALP/TBil
Suggest hepatitis serology
?Transient
Suggest albumin /total protein
AST>ALT
?Recent commencement
Clinical symptoms better than Epilim
level
Consider alternative drug for bipolar
disorder
If no other cause stop Epilim
Restrict salicylate/same metabolic path
?Hepatocellular disease
?Manic episodes or fitting
Details of last dose not stated
?Autoimmune disorder
?Cirrhosis
Monitor Epilim levels
Suggest ammonia ?hepatic
encephalopathy
Valproate highly protein bound
?Cardiac involvement
Epilim doses lower in bipolar disorder
Suggest renal function tests
Epilim can cause obesity
Suggest fasting glucose/lipids
?Risk of pancreatitis
AST from other tissues
?Liver disease with haematologic
disturbance
Suggest urine myoglobin
?Neurometabolic disease
Suggest liver ultrasound
In liver disease ALT>AST
Suggest full blood examination
No comment
Suggest troponin T, CK and BNP
?Early hepatic failure
Suggest free valproic assay
?Acute episode
No evidence of cholestasis
In non-alcoholic fatty liver disease,
AST<ALT
?Urea cycle disorder
UNACCEPTABLE KEY
WORDS
Not likely hepatic dysfunction due to
Epilim
?2° to metabolic syndrome
?Renal damage
No evidence hepatotoxicity
Suggest hormone levels
Suggest thyroid function tests
?End-stage AIDS
Suggest C-Peptide
Deaths seen in patients on Epilim
Case 9-10
Liver
Patient ID
23-day-old baby
Patient Location
Paediatric Ward
Clinical Notes on Request Form
Increasing jaundice.
Case Details
Bilirubin
367 μmol/L
Conjugated bilirubin 10 μmol/L
Suggested Comment
Prolonged, marked, unconjugated hyperbilirubinaemia. Breast milk jaundice may be considered after
excluding hypothyroidism, sepsis and haemolytic disorders. Suggest thyroid function tests, full blood
examination with blood film, direct Coomb’s test, blood group subtyping of mother and baby, glucose-6phosphate dehydrogenase (G6PD) screen, and urine culture. Clinical advice regarding management is
recommended.
Rationale
Hyperbilirubinaemia beyond two weeks of age (three weeks if premature) is considered ‘prolonged’. The
exclusion of conjugated hyperbilirubinaemia is important, as they are always pathological, requiring urgent
attention. Unconjugated hyperbilirubinaemia is most commonly due to breast-feeding but this cannot be
assumed without excluding hypothyroidism, haemolysis (from red blood cell abnormalities and atypically late
presentations of blood group incompatibility) and infection. It is also important to ensure that routine metabolic
screening (including congenital hypothyroidism screening) has been performed. At this level of bilirubin,
treatment advice should be sought. It is worthwhile for laboratories to be familiar with local paediatric hospital
guidelines, which usually have treatment thresholds relating the levels of bilirubin with the baby’s age and
clinical state.
References
1. Evans N. Neonatal jaundice. Australian Doctor 2008;8th Feb:21-28.
2. NICE clinical guideline 98: Neonatal jaundice. http://www.nice.org.uk/guidance/cg98
PREFERRED KEY WORDS
LESS RELEVANT KEY
WORDS
Unconjugated hyperbilirubinaemia
Marked hyperbilirubinaemia
Prolonged hyperbilirubinaemia/jaundice
?Breast milk jaundice
?Sepsis
?G6PD deficiency/other red cell enzyme
defect
?Haemolytic condition
?Hypothyroidism
Review clinical picture/history
Suggest full blood examination
Suggest thyroid function tests
Suggest haemolytic screen
Suggest blood group and rhesus and
antibody status of mother and baby
Suggest sepsis investigations
Suggest red cell G6PD assay
Suggest blood film
Check newborn screening done
Suggest urine microscopy, culture and
sensitivity
?Crigler-Najjar
Suggest liver function tests
?Pathological impaired liver conjugation
Risk of kernicterus
Suggest further investigation
?Baby premature
?Galactosaemia
Suggest reducing substances
Monitor
Consider phototherapy
?Gilbert’s syndrome
Hyperbilirubinaemia
Outside timeframe for physiological
jaundice
Suggest TORCH(S) screen
?Inborn error of metabolism
?Congenital infection
?Hereditary red cell membrane defect
Refer to paediatrician
?Inadequate caloric intake
Cease breastfeed to assess
No comment
Consider treatment
?Thalassaemia/haemoglobinopathy
?Prolonged physiological jaundice
Treat to reduce bilirubin if increase
continues
?Drug effect
Suggest hepatitis screen
Repeat total and direct bilirubin in one
day
?Excessive red blood cell turnover
Suggest glucose
Suggest C-reactive protein
?Lucy Driscoll syndrome
?Secondary to acute illness
Suggest serum albumin
UNACCEPTABLE KEY
WORDS
Immediate treatment required
Consider exchange transfusion
?Obstructive liver disease
No intervention necessary
Bilirubin level excludes Gilbert’s
Bilirubin level excludes haemolytic
disorder
?Hypopituitarism
?Hypoxia
Case 10-04
Liver
Patient ID
60-year-old male
Patient Location
Emergency Department
Clinical Notes on Request Form
Decreased conscious state.
Case Details
Plasma ammonia
375 μmol/L (10–50)
Additional Information
Na
142 mmol/L (134–146)
K
3.8 mmol/L (3.4–5.0)
Bicarb.
23 mmol/L
(22–32)
Urea
6.4 mmol/L (3.0–8.0)
S. Creat.
110 μmol/L (60–110)
eGFR
>60 mL/min/1.73m2 (>60)
Glucose
6.9 mmol/L (3.0–5.4)
Protein
67 g/L
(65–85)
Albumin
30 g/L
(34–47)
T. Bilirubin 26 μmol/L
(<22)
ALP
106 U/L
(30–120)
GGT
147 U/L
(10–71)
ALT
12 U/L
(5–40)
Lipase
27 IU/L
(13–60)
cTnI
0.04 μg/L
ABG
pH
7.47
pCO2 31 mmHg
pO2 97 mmHg
O2 sat. 98 %
Lactate 1.8 mmol/L
(<0.04)
(7.35–7.45)
(35–45)
(80–100)
(>95)
(<1.3)
Suggested Comment
Marked hyperammonaemia is most likely the cause of the decreased conscious state and respiratory alkalosis.
Further investigation for liver and non-liver causes of ammonia toxicity such as drugs (valproate), infective
agents, or urea cycle defects is recommended. Suggest hepatitis screen, INR, sepsis screen, urine organic acids,
orotic acid and plasma and urine amino acids.
Rationale
Mild hyperammonaemia is frequently caused by poor specimen integrity and/or collection technique. Marked
hyperammonaemia in the clinical setting of decreased conscious state and respiratory alkalosis makes a
spurious result unlikely. Such high ammonia values are most frequently seen in patients with liver failure. The
relatively preserved albumin, mildly raised bilirubin and mildly abnormal liver function tests are not consistent
with end stage liver failure, as synthetic capacity seems to be at least partly preserved. An INR will be helpful
to confirm this.
Non-liver causes of hyperammonaemia, many of which are reversible, should be investigated. Urea cycle (e.g.
ornithine transcarbamylase deficiency), fatty and organic acid defects need to be considered as they may
present at any age. Valproate can cause hyperammonaemia by N-acetyl-glutamate depletion; although
valproate and liver function tests are usually within therapeutic/reference limits. Excessive amino acid load
(e.g. gastrointestinal bleeding; unlikely here given the normal urea concentration) and haematologic
malignancies are other possibilities. The mildly raised troponin in this setting is difficult to interpret and might
be due to a cardiac condition (e.g. cardiac failure) or severe illness (e.g. sepsis). In this particular patient the
final explanation was severe right heart failure impeding the ability of the liver to metabolise ammonia loads
caused by increased protein intake.
Reference
1. Walker V. Severe hyperammonaemia in adults not explained by liver disease. Ann Clin Biochem
2012;49:214-28.
PREFERRED KEY WORDS
?Chronic hepatic disease
Marked hyperammonaemia
Respiratory alkalosis
Compensated respiratory alkalosis
Hyperammonaemia coupled with loss of
consciousness
?Hepatic encephalopathy
Suggest further investigation of
hyperammonaemia
?Drug/toxin effect
?Sepsis
?Valproate aetiology
?Urea cycle metabolic defect
?Inborn error of metabolism
Late onset ornithine transcarbamylase
deficiency
Suggest plasma/urine amino acids
Suggest urine organic acids
?Hepatotoxicity/acute hepatic injury
Suggest clotting profile
Suggest plasma acylcarnitine
Suggest valproate level
?Hyperalimentation
LESS RELEVANT KEY
WORDS
Hyperammonaemia
?Ethanol toxicity/alcoholic liver disease
Elevated GGT
?Sample integrity/preanalytical
interference
Hypoalbuminaemia
Suggest recollection/repeat ammonia
?Salicylate aetiology/level
Hyperbilirubinaemia
Correlate with clinical findings
Mildly abnormal liver function tests
Liver function tests not significantly
abnormal
?Acetaminophen aetiology/level
?Surgery/portal shunt
?Chemotherapy effect
Near normal albumin
Suggest urine/blood drug/toxin screen
Suggest ethanol levels
Suggest urine microscopy, culture and
sensitivity
Monitor ammonia
Urea and electrolytes not significantly
abnormal
Unlikely metabolic defect due to age
Normal ALT
?Viral hepatitis/serology
Decreased protein
Consistent with severe liver disease
?Muscle ischaemia/prolonged
immobilisation
Suggest CRP
Suggest repeat cTnI in 6–12 h
?Primary/secondary malignancy
Suggest full blood examination
Monitor liver function tests
?Hashimoto's encephalopathy
Check liver function tests / urea and
electrolytes
Monitor blood gas
cTnI suggests myocardial injury
Suggest chloride level
?Random glucose result
Lactate not significantly elevated
Do not monitor therapy with ammonia
levels
Suggest anion gap
Normal bilirubin
Borderline cTnI
Elevated cTnI due to stress response
Refer to clinical pathologist
Suggest phosphate
Mild renal failure
UNACCEPTABLE KEY
WORDS
Hyperlactataemia
?Gastrointestinal haemorrhage
Unlikely chronic liver disease
?Urinary tract infection
Metabolic acidosis/compensated
Hyperglycaemia
?Carbamazepine aetiology
Suggest BNP/?heart failure
Markedly increased lipase
?Tumour of prostate
Portal hypertension
Not consistent with hepatic
encephalopathy
?Exogenous ammonia
Case 11-08
Liver
Patient ID
59-year-old female
Patient Location
General Practice
Clinical Notes on Request Form
Itchy
Case Details
Total Protein
Albumin
ALP
Total Bilirubin
GGT
AST
ALT
70 g/L
33 g/L
163 U/L
8 μmol/L
143 U/L
37 U/L
47 U/L
(60–81)
(35–50)
(40–140)
(<25)
(<51)
(<41)
(<41)
Additional Information
Similar results one month ago.
Suggested Comment
The liver enzyme pattern is consistent with focal cholestasis. Persistent increase in ALP and GGT in a woman
with a history of itch suggests a diagnosis of primary biliary cirrhosis. Measurement of antimitochondrial
antibodies (AMA) would be useful. Other causes of abnormal liver function tests such as ethanol and
medications should be considered. Liver ultrasound may be helpful.
Rationale
The combination of itch with raised cholestatic enzymes in a middle-aged woman raises the possibility of
primary biliary cirrhosis (PBC), an autoimmune liver disease, which is characterised by the presence of
antimitochondrial antibodies in the serum. Other autoantibodies such as anti-nuclear antibodies (ANA) may
also be present in these patients. Serum immunoglobulin M, cholesterol and bile acids may be increased in
advanced disease but these are not useful in early diagnosis. Many other conditions such as drug reactions,
alcohol and primary sclerosing cholangitis (PSC) may be accompanied by increased cholestatic enzymes.
Clinical suspicion for PBC and PSC are increased by the presence of other autoimmune disease and
inflammatory bowel disease, respectively, The fact that the results were similar one month ago is reassuring
but liver ultrasound may be useful to exclude a focal lesion in the liver such as a malignant tumour.
References
1. Lindor KD, Gershwin ME, Poupon R, et al. Primary biliary cirrhosis. Hepatology 2009;50:291-308.
2. Giannini EG, Testa R, Savarino V. Liver enzyme alteration: a guide for clinicians. CMAJ
2005;172:367-79.
PREFERRED KEY WORDS
Liver function tests suggest cholestatic
liver disease
Primary biliary cirrhosis
?Primary biliary cirrhosis
?Primary sclerosing cholangitis
?Alcoholic liver disease
?Neoplastic liver disease
?Autoimmune disease
?Pruritus due to cholestasis
?Intrahepatic cholestasis
?Infiltrative liver disease
Suggest anti-mitochondrial antibodies
Exclude biliary obstruction/gall stones
Suggest hepatobiliary imaging
Suggest autoantibodies
Request drug history
LESS RELEVANT KEY
WORDS
Hypoalbuminaemia
Suggest immunoglobulins
Suggest lipid studies
Elevated ALP and GGT
Mildly elevated fiver function tests
Suggest viral serology
Suggest full blood examination
Persistent itch/middle aged woman
Low albumin suggests chronic liver
disease
Normal bilirubin level
Persistent moderately raised GGT
?Obesity/diabetes mellitus/metabolic
syndrome
Suggest renal function test and anion gap
?Hodgkin's disease/lymphoma
Suggest bile acids
Suggest AFP
Suggest bone markers
?Cirrhosis
Request clinical history
Suggest repeat/monitor liver function
tests
?Chronic renal failure
?Polycythaemia/haematological
Suggest glucose
?Fatty liver disease
Suggest INR/APTT
Mildly elevated ALP
Abnormal liver function tests
?Pancreatitis/suggest amylase/lipase
Hepatobiliary disease
Mildly elevated ALT
?Thyroid disorder/suggest thyroid
function tests
Suggest protein electrophoresis
?Early Paget's disease
Consistent with chronic obstructive
jaundice
Suggest gastroenterologist consultation
Low alb/poor nutrition/increased loss
Normal AST and ALT
Suggest fat-soluble vitamins
?Reference interval correct
?Inflammatory bowel disease
UNACCEPTABLE KEY
WORDS
? Chronic viral hepatitis
Exclude other causes of pruritus
?Iron overload/suggest iron studies
Suggest liver biopsy
?Infection
?Hepatocellular injury
?Primary biliary sclerosis
Hepatitis unlikely
Suggest serum/urine osmolality
?Cause not related to itch
?Chronic infection
?Wilson's disease
Suggest serum copper/α1-antitrypsin
Suggest ACE to exclude sarcoidosis
Case 12-09
Liver
Patient ID
49-year-old female
Patient Location
General Practice
Clinical Notes on Request Form
Recent weight gain despite reduction in alcohol consumption.
Case Details
Total Bilirubin
Total Protein
Albumin
ALP
GGT
ALT
AST
17 μmol/L
76 g/L
44 g/L
63 U/L
136 U/L
59 U/L
31 U/L
(<21)
(62–80)
(38–48)
(30–130)
(≤60)
(5–55)
(5–55)
Additional Information
Setting: General practice review as a component of Medicare incentive for chronic disease prevention.
Suggested Comment
Mild increases in GGT and ALT are consistent with non-alcoholic fatty liver disease or non-alcoholic
steatohepatitis if alcohol abuse is excluded. Consider obesity, insulin resistance, dyslipidaemia, drugs and
continued alcohol use. Exclude common causes of hepatocellular liver damage. A small proportion of patients
with this profile may progress to non-alcoholic steatosis and cirrhosis.
Rationale
Mild increases in GGT and ALT are consistent with non-alcoholic fatty liver disease. Obesity, insulin
resistance and dyslipidaemia are often associated with this condition, which may be confirmed by hepatic
imaging. Drugs, continued alcohol use and common causes of hepatocellular liver damage, such as
hypothyroidism should be excluded. A small proportion of patients may progress to non-alcoholic steatosis,
fibrosis and cirrhosis but liver biopsy is not indicated unless the problem is severe or sustained. Weight loss,
alcohol avoidance and review of medications are the most likely means by which results may be improved.
Reference
1. Vuppalanchi R, Chalasani N. Nonalcoholic fatty liver disease and nonalcoholic steatohepatitis:
selected practical issues in their evaluation and management. Hepatology 2009;49:306-17.
PREFERRED KEY WORDS
?Metabolic syndrome
?Fatty liver disease
?Non-alcoholic fatty liver disease
?Non-alcoholic steatohepatitis
?Continued alcohol intake
?Drugs/medication
?Iron storage disease/haemochromatosis
?Hepatic steatosis
?Insulin resistance
Suggest lose weight, avoid alcohol, drugs
Suggest hepatitis serology
Suggest diabetes screen
Suggest lipid profile
Suggest thyroid function tests
LESS RELEVANT KEY
WORDS
?Elevated GGT due to alcohol
Liver enzyme induction
?Alcohol
Suggest repeat in 1–3 months
Mildly elevated ALT
Mild hepatocellular damage
Elevated GGT
Suggest hepatic imaging
Weight gain
Weight gain, increased GGT and slight
increase in ALT
Mild elevation of GGT
?Chronic viral hepatitis
?Autoimmune disease
Mildly elevated GGT and ALT
?Obesity
Suggest cardiovascular risk assessment
?Cirrhosis
?Diabetes mellitus
?Toxins
Suggest carbohydrate-deficient
transferrin
Suggest healthy active lifestyle
?Other cause chronic liver disease
?Dyslipidaemia
Suggest albumin:creatinine ratio
GGT may be raised for 1 month
No routine comment
?Herbal remedies
Suggest urate
Repeat after no alcohol and weight loss
AST/ALT ratio <1
Suggest BMI
NOT SUPPORTED KEY
WORDS
Mild non-specific hepatopathy
?Infection
?Increased appetite
?Hepatobiliary disease
Suggest morning cortisol
?Advanced fibrosis
Suggest full blood examination
Suggest liver biopsy
Not hepatobiliary disease
Lowering of bilirubin
MISLEADING KEY WORDS
ALT, AST not significantly elevated
Case 13-01
Liver
Patient ID
33-year-old male
Patient Location
General Practice
Clinical Notes on Request Form
Recent travel to the Middle East. Tiredness, lethargy, nausea and anorexia.
Case Details
Total Protein
Albumin
ALP
Total Bilirubin
GGT
AST
ALT
LD
74 g/L
33 g/L
135 U/L
96 μmol/L
376 U/L
947 U/L
3117 U/L
463 U/L
(60–82)
(35–50)
(30–120)
(<25)
(<50)
(<41)
(<51)
(50–280)
Suggested Comment
Markedly raised transaminases are consistent with predominant hepatocellular damage. The enzyme pattern
and recent travel history suggest viral hepatitis, particularly hepatitis A, as the most likely cause. Recommend
serology to confirm the diagnosis. Paracetamol toxicity is less likely but should also be considered.
Rationale
Marked elevation in AST and ALT with ratio <<1 suggests acute hepatocellular damage. The highest
transaminase concentrations are seen in ischaemic/toxic liver damage and viral hepatitis. Recent travel to the
Middle East suggests infection as the most likely aetiology and of the many possibilities; hepatitis A is the
most common cause in this setting. Acute hepatitis can also be due to toxins and ischaemia although the
AST/ALT ratio in these cases is usually >1.
Of toxins, paracetamol should be considered urgently due to time constraints for treatment, however in
addition to the low AST/ALT ratio, the relatively low LD makes paracetamol toxicity less likely. Alcohol is
less likely due to the low AST/ALT ratio. The relatively low LD and patient age make ischaemic causes
unlikely. Autoimmune causes are possible but less likely with this degree of transaminase elevation and should
be considered only when the more likely causes have been excluded.
Reference
1. Giannini EG, Testa R, Savarino V. Liver enzyme alteration: a guide for clinicians. CMAJ 2005:172
367-79.
PREFERRED KEY WORDS
LESS RELEVANT KEY
WORDS
NOT SUPPORTED KEY
WORDS
Predominantly hepatocellular pathology
Liver function tests suggest viral hepatitis
Liver function tests suggest hepatitic
picture
Results consistent with parenchymal liver
disease
?Cytotoxic liver damage
Hepatocellular disease
Low AST/ALT ratio-viral origin/acute
Exclude paracetamol toxicity
ALT and AST markedly elevated
Results consistent with acute hepatitis
Consider hepatitis A
?Acute hepatitis
?Infectious hepatitis
Exclude viral hepatitis
Suggest hepatitis serology (A,B,C,D,E)
Exclude drug/toxin/herbal induced
?Drug/toxin/herbal induced
Suggest repeat liver function tests
Phone results to referring doctor
?EBV, CMV, Coxsackie, HSV
Suggest coagulation profile
Suggest full blood
examination/differential
Some degree of cholestasis
Suggest renal function
Suggest HIV serology
Exclude ischaemic liver damage
Cholestasis and reduced synthetic ability
Suggest iron studies
Suggest serum copper and
caeruloplasmin
Suggest α1-antitrypsin, anti-smooth
muscle antibodies
?Acute cholangitis
Raised GGT consistent with cytotoxic
damage
?Accompanying haemolysis
Suggest indirect bilirubin
?Leptospirosis
Ischaemic liver damage less likely
Less common EBV, CMV, Coxsackie,
herpes simplex 1 virus
?Autoimmune hepatitis
Suggest malaria study/thick and thin film
?Alcohol induced
?Wilson's disease
Suggest amoebic serology
?Acute on chronic infection
Low albumin metabolism
Suggest glucose level
Suggest ultrasound of hepatobiliary
system
Raised liver function tests consistent with
alcohol/toxins
Suggest hepatology review
Suggest schistosomiasis
Exclude Katayama fever
Suggest rickettsial studies
Suggest ethanol level
Exclude parasitic infections
Suggest toxoplasmosis
Suggest radiology for lesions
Suggest urine drug screen
?Dengue fever/?Q fever
Suggest blood cultures
Suggest stool examination
Exclude hypoxia
?Secondary to cholestatic jaundice
Exclude fatty acid oxidation disorder suggest acylcarnitine
?Brucellosis
?Yellow fever
?Typhoid
Exclude biliary obstruction
Suggest carbohydrate-deficient
transferrin
Case 14-08
Liver
Patient ID
65-year-old female
Patient Location
Intensive Care Unit
Clinical Notes on Request Form
Gastric ulcer.
Case Details
GGT
ALP
ALT
AST
LD
Bilirubin
Albumin
79 U/L
51 U/L
3349 U/L
2325 U/L
4219 U/L
6 μmol/L
13 g/L
(<60)
(30–110)
(<55)
(<45)
(110–230)
(2–24)
(34–48)
Additional Information
Urea
23.5 mmol/L (2.7–8.0)
Creatinine
83 μmol/L
(50–100)
Hb
63 g/L
(115–155)
Suggested Comment
Marked elevation in transaminases suggests acute hepatitis. Drug effects and hypoxia should be considered.
Viral hepatitis is less likely due to significantly raised lactate dehydrogenase (LD) and normal bilirubin.
Markedly low albumin may include effect of recent gastrointestinal bleed but protein-losing states should be
excluded. Recommend coagulation studies and search for underlying conditions, particularly paracetamol and
other medications, heart failure and sepsis.
Rationale
Hypoxic hepatitis (HH) occurs in up to 1% of ICU admissions. Mortality is over 50% during hospitalisation
and 25% after one year, and is related to underlying medical conditions. The diagnosis of HH requires (1)
clinical setting of acute cardiac, respiratory or circulatory failure; (2) a sharp but transient increase in serum
aminotransferases activity, reaching at least 20 times the upper reference limit; (3) exclusion of other causes of
acute liver cell necrosis, particularly viral or drug-induced hepatitis. Contrary to expectation, shock was absent
in around half of reported HH patients. Severe blood loss alone is not usually sufficient to cause HH. In more
than 90% of cases, the patient has a severe underlying pathology, particularly heart failure and septic shock.
Renal failure is common (15–65% of cases) and is an independent marker of poor prognosis due to organ
hypoperfusion. Of note, the elevated urea is likely the result of catabolism of the haemoglobin from the gastric
ulcer.
Massive elevation in transaminases due to paracetamol poisoning typically occurs 2 to 5 days after ingestion,
when the paracetamol concentration is usually undetectable. A careful history of paracetamol ingestion,
including slow release forms, is the best diagnostic approach.
Reference
1. Henrion J. Hypoxic hepatitis. Liver Int 2012;32:1039-52.
PREFERRED KEY WORDS
Marked elevation of transaminases
Severe anaemia
Acute hypoxic liver injury
Acute hepatitis
Viral hepatitis unlikely
Consistent with upper gastrointestinal
tract (GIT) bleed
?Massive bleed - gastric ulcer
?Ischaemia
?Drugs, alcohol or toxins
Suggest coagulation studies
Review drug history
?Hypoperfusion due to blood loss
Poor prognosis
Investigate low albumin
LESS RELEVANT KEY
WORDS
Elevated urea
Elevated lactate dehydrogenase (LD)
Hypoalbuminaemia
Low haemoglobin
Suggest hepatitis serology
?Viral hepatitis
Hepatocellular damage
Clinical history noted
Normal creatinine
Normal bilirubin
Suggest full blood count
?Severe shock
Normal ALP
?Impaired hepatic synthesis
?Haemorrhagic shock
Liver function tests acute/chronic
impairment
Suggest electrolytes
Elevated urea/creatinine ratio
Mildly elevated GGT
Probable hepatic necrosis
Consistent with hepatitis
Cholestasis unlikely
?Pre renal acute kidney injury
Monitor liver function tests
Rule out helicobacter pylori infection
Dissociation urea and creatinine
Correlate with clinical history
?Hyperlactataemia
?Encephalopathy
Suggest renal function tests
Hepatic liver function tests pattern
Suggest lactate and glucose
Non-obstructive picture
Suggest total protein
NOT SUPPORTED KEY
WORDS
Suggest paracetamol levels
?Acute liver failure
?Autoimmune hepatitis
?Malignancies
Suggest arterial blood gas
Isolate bleed/transfuse
?Haematological disorder
?Perforated gastric ulcer
Suggest endoscopy
Suggest creatine kinase
?Mucosal erosion in GIT
Suggest amylase
?Zollinger-Ellison syndrome
MISLEADING KEY WORDS
?Chronic liver disease
Suggest liver ultrasound
Suggest gastrin
?Muscle injury
Suggest lipase
Suggest haptoglobin
?Nephrotic syndrome
Suggest faecal occult blood
Suggest urine protein
?Hepatic dysfunction
Exclude haemoglobinopathy
Case 8-07
Enzymes
Patient ID
85-year-old female
Patient Location
Cardiology
Clinical Notes on Request Form
Past history of heart valve replacement.
Case Details
Total Bilirubin
ALP
AST
ALT
GGT
LD
12 μmol/L
66 U/L
36 U/L
25 U/L
45 U/L
481 U/L
(3–20)
(30–115)
(<40)
(<40)
(5–65)
(100–225)
Additional Information
Hb
HcT
RCC
Retic
MCV
WCC
134 g/L
0.41
4.5 x 1012
33 x 109
93 fL
8.7 x 109
(110–160)
(0.34–0.47)
(3.7–5.4)
(10–100)
(80–100)
(3.5–10.0)
Suggested Comment
The elevated lactate dehydrogenase (LD) may be due to intravascular haemolysis caused by the prosthetic
heart valve. The normal reticulocyte count, haemoglobin and bilirubin levels indicate that the haemolysis is
mild. Suggest monitor to ensure condition is stable, or consider other causes of raised LD, if clinically
indicated.
Rationale
A mild degree of intravascular haemolysis is common among patients (prevalence 50–90%) with normally
functioning prosthetic heart valves and is attributed to mechanical destruction of red blood cells. Clinically
significant haemolysis is rare and occurs mainly with malfunctioning valves accompanied by paraprosthetic
valvular regurgitation (1). Elevated serum lactate dehydrogenase (LD) activity in patients with prosthetic heart
valves is well documented and correlates with the level of haemolysis (2). Depending on the degree of
haemolysis there may be an increase in the reticulocyte count, serum bilirubin and a reduction in serum
haptoglobin. However, LD is a non-specific marker and can be raised due to delayed separation, myocardial
infarction, liver disease, skeletal muscle disease and malignancy. In this case, a full clinical history and the
appropriate choice of tests should assist with the diagnosis.
References
1. Ismeno G, Renzulli A, Carozza A, et al. Intravascular hemolysis after mitral and aortic valve
replacement with different types of mechanical prostheses. Int J Cardiol 1999;69:179-83.
2. Mansuroğlu D, Omeroğlu SN, Izgi A, et al. LDH levels and left atrial ultrastructural chances in patients
with mitral paraprosthetic regurgitation. J Card Surg 2005;20:229-33.
PREFERRED KEY WORDS
Subclinical haemolysis
?Haemolysis
Consistent with prosthetic valve
replacement
Haemolysis due to heart valve
?Intravascular haemolysis
Suggest cardiac markers (troponin,
creatinine kinase)
LESS RELEVANT KEY
WORDS
Increased LD
Suggest serum haptoglobin
Normal full blood count
Bilirubin normal
?Myocardial injury
?Collection artefact
?Myocardial infarction
Hepatic enzymes normal
Suggest clinical evaluation
?Muscle damage
Suggest blood film
Elevated LD non-specific
Normal reticulocyte count
Monitor test results
Results not typical of haemolysis
Other results normal
?Paravalvular leakage/dysfunction
No anaemia - haemoglobin normal
Check if sample haemolysed
?Pulmonary embolus
?Other haematological disorders
Suggest repeat LD
LD unlikely liver origin
Suggest renal function tests
?Cell injury/inflammation
Suggest potassium
?Macro-LD
?Due to warfarin therapy
Check previous LD results
Check INR
Check haemolysis index
Suggest platelet count
No megaloblastic anaemia
?Myocardial disease
UNACCEPTABLE KEY
WORDS
?Malignancy
Suggest LD isoenzymes
?Kidney or lung damage
Intravascular haemolysis unlikely
Increased AST:ALT/high AST
No comment
Suggest investigations for malignancy
Suggest urine Hb/haemosiderin
?Hypothyroidism
?Liver disease
?Type of valve
Isoenzymes not indicated
Not iron deficiency anaemia
Suggest Schumm test
Not due to liver disease or haemolysis
Due to aspirin therapy
Suggest C-reactive protein
Case 8-08
Enzymes
Patient ID
16-month-old male
Patient Location
General Practice
Clinical Notes on Request Form
Post upper respiratory tract infection
Case Details
Serum
Sodium
Potassium
Chloride
Bicarbonate
Urea
Creatinine
Glucose, random
Protein
Albumin
Globulin
T Bilirubin
ALP
AST
ALT
GGT
LDH
144 mmol/L
4.5 mmol/L
109 mmol/L
19 mmol/L
4.8 mmol/L
<20 µmol/L
4.7 mmol/L
63 g/L
38 g/L
25 g/L
3 µmol/L
5150 U/L
29 U/L
14 U/L
<5 U/L
294 U/L
(135–145)
(3.5–5.0)
(97–107)
(20–28)
(1.8–7.5)
(30–70)
(3.6–7.7)
(60–75)
(35–50)
(15–25)
(3–20)
(60–300)
(25–80)
(5–40)
(5–65)
(100–405)
Suggested Comment
In the absence of liver or bone disease the isolated marked elevation of the alkaline phosphatise (ALP) in this
child makes the diagnosis of benign transient hyperphosphatasaemia of infancy and childhood likely. Suggest
ALP isoenzyme testing and repeat ALP in 2–3 months.
Rationale
The clinical history of an isolated marked elevation of alkaline phosphatise (ALP) in a young child after a viral
infection makes the diagnosis of benign transient hyperphosphatasaemia (BTH) of infancy and childhood
likely. The occurrence of BTH is not rare, and is mainly seen in winter. Most patients with BTH present during
the first year of life. It is associated with a wide variety of clinical disorders, including gastrointestinal
diseases, respiratory infections, congenital anomalies/inborn errors of metabolism, anaemia, and malignancies.
The differential diagnosis includes liver (such as biliary obstruction, and liver disease associated with
malignancy) and bone diseases (such as juvenile Paget’s disease or rickets). In patients with BTH, ALPisoenzyme electrophoresis shows increases in bone and liver ALP and ALP typically returns to normal in 2–3
months. In the Australian setting, it was seen in a third of children with increased ALP >1000 U/L in a tertiarycare teaching hospital. In the community setting the likelihood of BTH of infancy would be much higher.
References
1. Stein P, Rosalki SB, Foo AY, et al. Transient hyperphosphatasemia of infancy and early childhood:
clinical and biochemical features of 21 cases and literature review. Clin Chem 1987;33:313-8.
2. Behúlová D, Bzdúch V, Holesová D, et al. Transient hyperphosphatasemia of infancy and childhood:
study of 194 cases. Clin Chem 2000;46:1868-9.
3. Carroll AJ, Coakley JC. Transient hyperphosphatasaemia: an important condition to recognize. J
Paediatr Child Health 2001;37:359-62.
PREFERRED KEY WORDS
Isolated increased ALP
?Benign transient hyperphosphatasaemia
of infancy (BTHI)
BTHI seen in young children
BTHI often post infection
Expect normal ALP <3 months
Expect normal ALP in 3–6 months
Resolves spontaneously
?Familial hyperphosphatasaemia
?Bone disease excluded
?Liver disease excluded
Consider vitamin D deficiency
Suggest repeat ALP ≤3 months
Suggest ALP isoenzymes
Monitor ALP
Suggest follow-up
Consider rickets
LESS RELEVANT KEY
WORDS
Suggest calcium/phosphate
Suggest clinical evaluation
Other liver function tests normal
Suggest vitamin D/PTH
Likely bone origin
Repeat ALP 3–6 months
Low creatinine
?ALP due to viral illness
Repeat ALP
Mild metabolic acidosis
Creatinine consistent with low muscle
mass
Suggest screen family members
?Healing fracture
No further investigation required
Consult with Chemical Pathologist
Suggest consult specialist
Repeat liver function tests
?Malnutrition
?Renal disease
Marginally low bicarbonate
Elevated chloride
?GGT artefactual
UNACCEPTABLE KEY
WORDS
?Malignancy
Not hepatic origin
Suggest magnesium
?Paget’s disease
?Medication-induced
Repeat GGT
?Creatinine interference, suggest repeat
Suggest bone turnover marker
Suggest CT/X-ray
?Chronic respiratory alkalosis
Consider hyperparathyroidism
Likely leukocyte origin
Suggest white cell count/diff
?Osteomyelitis
Suggest liver ultrasound
Reye’s syndrome unlikely
Repeat full blood count
Suggest urinalysis
Suggest blood gases
Suggest serology testing
Intestinal origin
May be due to growth spurt
Consider bone scan
Case 9-01
Enzymes
Patient ID
61-year-old male
Patient Location
Emergency Department
Clinical Notes on Request Form
Abdominal pain.
Case Details
Serum lipase 5169 IU/L
(13–60)
Additional Information
Serum
Protein
77 g/L
Albumin
42 g/L
T Bilirubin
27 µmol/L
ALP
131 U/L
GGT
362 U/L
ALT
46 U/L
Adjusted Ca 3.19 mmol/L
Glucose
19.2 mmol/L
(65–85)
(34–47)
(<22)
(30–120)
(10–71)
(5–40)
(2.10–2.60)
(3.0–7.7)
Suggested Comment
Marked elevation of lipase together with abdominal pain is likely caused by acute pancreatitis. A C-reactive
protein concentration might provide prognostic information. Hyperglycaemia early in the course of the
pancreatitis may resolve. The main causes of pancreatitis are gallstones and alcohol; hypercalcaemia is rarely
the cause of acute pancreatitis and needs to be further investigated.
Rationale
Marked elevation of lipase together with abdominal pain is likely caused by acute pancreatitis. A lipase of >3
times the upper reference limit has a diagnostic specificity of about 98% for pancreatitis. The magnitude of
lipase elevation is a poor prognostic indicator. Severity of the pancreatitis can be assessed using a number of
scoring systems, the most popular being the APACHE II. An APACHE II score of ≥8 is associated with
significant mortality risk. A CRP concentration >150 mg/L provides further prognostic information.
Hyperglycaemia early in the course of the pancreatitis may be due to acute pancreatic dysfunction. Later, it is
a marker for disease severity. The main causes of pancreatitis are gallstones and alcohol; hypercalcaemia is
rarely the cause of acute pancreatitis and needs further investigations. While hypercalcaemia increases the risk
for pancreatitis, additional gene mutations might contribute to the absolute risk. The elevated GGT is not
helpful in diagnosing alcoholic pancreatitis with an AUC of 0.51 for this purpose (3). The ALT is not elevated
enough to indicate a gallstone aetiology (4). Of note, the diagnosis of diabetes mellitus using random fasting
glucose requires 1) glucose of ≥11.0 mmol/L, and 2) classical symptoms of hyperglycaemia or hyperglycaemic
crisis. The diagnosis of diabetes mellitus should not be made in a patient who is systematically ill.
References
1. Lankisch PG, Burchard-Reckert S, Lehnick D. Underestimation of acute pancreatitis: patients with only
a small increase in amylase/lipase levels can also have or develop severe acute pancreatitis. Gut
1999;44:542-4.
2. Working Party of the British Society of Gastroenterology, et al. UK guidelines for the management of
acute pancreatitis. Gut 2005;54:iii1-9.
3. Methuen T, Kylänpää L, Kekäläinen O, et al. Disialotransferrin, determined by capillary
electrophoresis, is an accurate biomarker for alcoholic cause of acute pancreatitis. Pancreas
2007;34:405-9.
4. Tenner S, Dubner H, Steinberg W. Predicting gallstone pancreatitis with laboratory parameters: a metaanalysis. Am J Gastroenterol 1994;89:1863-6.
5. Felderbauer P, Karakas E, Fendrich V, et al. Pancreatitis risk in primary hyperparathyroidism: relation
to mutations in the SPINK1 trypsin inhibitor (N34S) and the cystic fibrosis gene. Am J Gastroenterol
2008;103:368-74.
PREFERRED KEY WORDS
Significantly elevated lipase
Consistent with acute pancreatitis
?Secondary to alcohol
Hypercalcaemia may cause pancreatitis
Glucose - endocrine pancreatic
dysfunction
?Hyperparathyroidism
Suggest PTH
Investigate raised calcium
Suggest fasting lipids
Hypertriglyceridaemia may cause
pancreatitis
?Secondary to gallstones
Suggest CRP
LESS RELEVANT KEY
WORDS
Hypercalcaemia noted
?Cholestatic/biliary disease
Hyperglycaemia
Elevated GGT
Hyperglycaemia due to
stress/inflammation
Liver function tests abnormalities
Suggest fasting plasma glucose/GTT post
resolution
Suggest renal function tests, eGFR
Diabetes mellitus not excluded
Suggest radiology chest/abdomen
?Chronic pancreatitis
?Drugs
Suggest full blood examination, LDH
Acute on chronic pancreatitis
Suggest phosphate
Suggest ionised calcium
Repeat calcium
Liver function tests and plasma glucose
secondary to pancreatitis
?Diabetic ketoacidosis
?Calcium infusion therapy
Liver function tests suggest hepatobiliary
disease
Suggest blood gases for acidaemia
Exclude renal failure
No evidence lipaemia
Repeat liver function tests
Calcium lower after first day
?Antacids cause hypercalcaemia
Possible acute pancreatitis
UNACCEPTABLE KEY
WORDS
Suggest amylase (lipase:amylase)
?Malignancy in pancreas
Pancreatitis may cause hypercalcaemia
?Diabetic nephropathy
Suggest vitamin D
?Raised calcium due to dehydration
?Pseudohyperparathyroidism
Suggest urinary albumin, protein
?Pancreatic glucagonoma
Liver function tests suggest
hepatocellular disease
Myeloma excluded
Glucose suggests diabetes
Malignancy causing hypercalcaemia
Case 12-01
Enzymes
Patient ID
47-year-old female
Patient Location
General Practice
Clinical Notes on Request Form
Previously raised CK, 5 months ago.
Case Details
CK
242 U/L
LD
220 U/L
(<150)
(110–230)
Additional Information
Albumin
42 g/L
Globulins
35 g/L
Protein
77 g/L
Total bilirubin 11 µmol/L
GGT
13 U/L
ALP
41 U/L
ALT
30 U/L
AST
33 U/L
(34–48)
(21–41)
(65–85)
(2–24)
(<60)
(30–110)
(<55)
(<45)
5 months earlier:
CK
465 U/L
LD
246 U/L
(<150)
(110–230)
Suggested Comment
Persistent mild elevation in creatine kinase (CK) suggests muscle damage, probably of skeletal origin.
Troponin, which is a more specific marker for cardiac muscle injury may be considered, if clinically suspected.
Possible causes of a raised CK of this level include medications (especially statins), hypothyroidism, exercise,
myositis, or metabolic conditions. Macro-CK may produce this pattern and should be screened for if other
causes cannot be identified. Recommend TSH and review of medication initially.
Rationale
Mildly (<10× upper reference limit) raised creatine kinase (CK) is a common laboratory finding. Most of these
normalise upon repeat testing, after asking the patient to refrain from exercise and alcohol consumption. Up to
11% of patients on statins have CK in this range; CK increase is dose-dependent and may be exacerbated by
exercise or other medications. To meet prescribing criteria for a statin, patients should have an increased risk
of cardiac disease, so it is not possible to rule out an acute coronary syndrome (ACS) in the absence of clinical
notes. It is important to exclude common clinical problems such as ACS, medication or hypothyroidism before
recommending more extensive investigations such as testing for macro-CK or muscle biopsy. Nonetheless, a
considerable number of patients with persistently elevated CK have subclinical neuromuscular pathology
demonstrable on histology or electromyography, although most of these conditions are benign.
References
1. Prelle A, Tancredi L, Sciacco M, et al. Retrospective study of a large population of patients with
asymptomatic or minimally symptomatic raised serum creatine kinase levels. J Neurol
2002;249:305-11.
2. Dabby R, Sadeh M, Herman O, et al. Asymptomatic or minimally symptomatic hyperCKemia:
histopathologic correlates. Isr Med Assoc J 2006;8:110-3.
PREFERRED KEY WORDS
Persistent elevation of CK
Skeletal muscle origin cannot be
excluded
?Skeletal muscle origin
?Skeletal myopathies
?Myositis
?Myopathy
?Muscle trauma/injury/surgery
Exclude statins
Exclude exercise/excess/?athlete
Exclude hypothyroidism
Review medication, ?drug associated
Exclude thyroid abnormalities/suggest
thyroid function tests
LESS RELEVANT KEY
WORDS
Consider macro-CK
Suggest CK isoenzymes-electrophoresis
Exclude cardiac source
Elevated CK
Normal AST/LD
Isolated CK elevation
Check medical history
?Clinical features of muscle disorder
Exclude alcohol excess
Mild elevation of CK
?Muscular dystrophy carrier
Exclude inflammation/infection
Normal liver function tests
Moderately elevated CK
Suggest ANA/exclude systemic lupus
erythematosus
?Kidney disease/renal function
?Metabolic myopathy
Suggest electrolytes, ?hypokalaemia
Suggests non-cardiac source
Suggest complete blood picture/ESR/film
Suggest monitor CK
?Family history of muscular dystrophy
Suggest calcium/phosphate
?Clinically consistent - phone to discuss
Myocardial injury unlikely
Suggests muscle origin
?Polymyalgia rheumatica
?Periodic paralysis
Suggest transaminase assay
Elevated CK consistent with muscle
damage
Check occupational history
Consider forearm ischaemic test
Suggests non-skeletal muscle origin
NOT SUPPORTED KEY
WORDS
?Intramuscular injection
?Muscular dystrophy
?Rhabdomyolysis/suggest myoglobin
Suggest cortisol
?Seizures
?Brain origin
?Malignancy e.g. colon, kidney
No comment and/or CK <1000 U/L
Suggest specialist referral
?Rheumatoid arthritis
?Guillain-Barre syndrome
Suggest DNA genetic markers
Suggest fasting lipid screen
Suggest vitamin D
Exclude malignant hyperthermia
?Neuroleptic malignant syndrome
?Coeliac disease
MISLEADING KEY WORDS
Consider muscle biopsy
Suggest electromyography
Exclude diabetic ketoacidosis/suggest
blood glucose
Suggest metabolic bone studies
Case 13-02
Enzymes
Patient ID
48-year-old male
Patient Location
General Practice
Clinical Notes on Request Form
Chronic alcoholic
Case Details
Amylase
Lipase
1530 U/L
49 U/L
(<100)
(<60)
Additional Information
Renal and liver function tests requested. Elevated GGT, otherwise no abnormalities.
Suggested Comment
Elevated serum amylase with normal lipase. This may be due to an extra-pancreatic source of amylase such as
parotid disease, which can be clarified by measurement of pancreatic and salivary amylase. Macroamylase
should also be considered. If parotid inflammation is not present clinically then exclusion of macroamylase by
measurement of urine amylase or other technique is recommended.
Rationale
Amylase and lipase are both elevated in cases of pancreatitis, although amylase tends to rise and fall sooner
than lipase. In alcoholic patients, lipase has a higher clinical sensitivity for pancreatitis than amylase. Serum
amylase is made up of two main types. The P-type amylase comes from the pancreas while the S-type amylase
comes from the salivary glands. Causes of elevated amylase without elevation of lipase include secretion from
other organs such as salivary glands, fallopian tubes and cyst fluid, gastrointestinal tract, testes, lungs, thyroid,
tonsils, and some malignant neoplasms. Of note, acute pancreatitis secondary to hypertriglyceridaemia may not
be associated with elevations in serum amylase.
The laboratory can contribute to the management of this patient by excluding the presence of macroamylase,
which are amylase enzymes with autoantibodies attached to them. This may be performed by measuring urine
amylase (normal in macroamylase due to size exclusion by glomerulus, and elevated in other causes),
polyethylene glycol (PEG) precipitation or other separative methods. Pancreatic amylase can be measured, and
when it is not elevated, confirms an extra-pancreatic source. In this patient the pancreatic amylase was within
normal limits, and was consistent with parotitis as the source of elevated amylase.
Reference
1. Harper SJ, Cheslyn-Curtis S. Acute pancreatitis. Ann Clin Biochem 2011;48:23-37.
PREFERRED KEY WORDS
Elevated amylase with normal lipase
Pancreatitis unlikely
Consistent with non-gastrointestinal
cause
?Macroamylasaemia
?Alcohol related salivary amylase
?Salivary amylase
?Parotiditis
?Acute abdominal condition
Suggest amylase isoenzymes
Suggest amylase clearance ratio (ACCR)
Suggest urine amylase
Suggest pancreatic amylase
Suggest PEG precipitation or gel
filtration
LESS RELEVANT KEY
WORDS
?Clinical symptoms
Suggest repeat testing
Suggest imaging
?Active biliary disease
May be seen in acute pancreatitis
Results phoned
Consistent with chronic alcoholism
?Pancreatic pseudocyst
NOT SUPPORTED KEY
WORDS
?Neoplastic disease
Increased GGT consistent with clinical
notes
?Renal failure
?Anti-epileptic medication
Suggest lipid studies
?Alcoholic pancreatitis
?Pancreatitis
?Heterophile antibodies
?Head injury/stroke
?Diabetic ketoacidosis
Lipase inhibited by bile acids
?Alcohol intoxication
Result not phoned
Suggest protein electrophoresis
Consistent with chronic pancreatitis
Suggest reanalysis on different
instrument
Case 14-07
Enzymes
Patient ID
46-year-old female
Patient Location
Pre-admission outpatient clinic
Clinical Notes on Request Form
Awaiting elective surgery for cholecystectomy. Family history of prolonged post-surgical
apnoea.
Case Details
Total Bilirubin
32 µmol/L
Albumin
37 g/L
Total Protein
62 g/L
GGT
158 U/L
ALT
65 U/L
AST
71 U/L
Butyryl-cholinesterase 5 kU/L
(<21)
(38–48)
(62–80)
(<35)
(5–55)
(5–55)
(6–14)
Additional Information
Further inquiry reveals that the family history consisted of a brother who required prolonged
ventilation after receiving an anaesthetic that included suxamethonium.
Suggested Comment
The family history raises the possibility of hereditary butyryl-cholinesterase deficiency. This should be
investigated further by genotyping. Phenotyping of the enzyme with inhibitors such as fluoride and dibucaine
is being phased out because it is less reliable. The possibility of prolonged post-anaesthetic apnoea should be
communicated to the surgical and anaesthetic team. The increase in serum bilirubin, aminotransferases and
glutamyltransferase is consistent with causes of liver dysfunction including cholelithiasis. Borderline reduction
in serum albumin and total protein is consistent with mild impairment of hepatic synthetic capacity, so
measurement of international normalised ratio (INR) is suggested. Cholelithiasis alone is insufficient
explanation for the observed reduction in the activity of butyryl-cholinesterase, but other causes of reduced
hepatic synthesis can cause this degree on butyryl-cholinesterase reduction.
Rationale
Butyryl-cholinesterase (pseudocholinesterase) is an enzyme produced by the liver, circulating in the plasma. Its
activity is a sensitive marker of hepatic synthetic capacity. It is also a specific indicator of the adequacy of the
enzyme that is required for the catabolism of a number of xenobiotic compounds, including suxamethonium.
Suxamethonium, or succinylcholine, is a nicotinic acetylcholine receptor agonist commonly used to induce
muscle relaxation before intubation in anaesthesia. In normal individuals, approximately 90–95% of an
intravenous dose of suxamethonium is metabolised by circulating plasma butyryl-cholinesterase before
reaching the neuromuscular junction. Deficiency of this enzyme can result in high concentration of
suxamethonium reaching the neuromuscular junction. This causes prolonged paralysis of up to 8 hours. The
activity of the enzyme in the plasma is inhibited by organophosphate insecticide toxicity. Conditions such as
uncomplicated cholelithiasis, cholestasis or mild cirrhosis rarely diminish its activity.
Butyryl-cholinesterase can decline following major acute illnesses and in malnutrition, or when hepatic
synthesis declines in the face of acute hepatitis, cirrhosis or hepatic metastasis. Less severe decline can occur
in the presence of muscular dystrophy, chronic renal disease and pregnancy. The family history raises the
possibility of hereditary butyryl-cholinesterase deficiency, which should be investigated further by genotyping
because phenotyping of the enzyme with inhibitors such as fluoride and dibucaine fails to fully characterise the
presence and pattern of hereditary enzyme deficiency.
Reference
1. Yen T, Nightingale BN, Burns JC, et al. Butyrylcholinesterase (BCHE) genotyping for postsuccinylcholine apnea in an Australian population. Clin Chem 2003;49:1297-308.
PREFERRED KEY WORDS
LESS RELEVANT KEY
WORDS
Low cholinesterase level/ activity
Borderline butyryl-cholinesterase
deficiency
Family history of suxamethonium apnoea
Inherited suxamethonium apnoea
?Risk of post suxamethonium/scoline
apnoea
Avoid suxamethonium/ mivacurium/
succinylcholine
Notify anaesthetist/surgeon
?Pseudocholinesterase deficiency
Cholestasis/biliary obstruction
Suggest family genotyping Suggest
repeat butyryl-cholinesterase when
liver function tests normal
Suggest screening of relatives
Suggest genotyping on brother
Suggest phenotyping and genotyping
Suggest genotyping
Suggest screening of relatives
Suggest cholinesterase genotyping
Suggest phenotyping OR genotyping
Avoid curare-derived drugs
Mild impairment hepatic synthetic
capacity
Suggest INR/PT
?Malnutrition
Need pre-op review by surgical team
?Low cholinesterase due to liver disease
Raised serum bilirubin, AST and GGT
Suggest phenotyping
Borderline low cholinesterase
Hypoalbuminaemia
?Biliary stones/ cholelithiasis
Suggest ALP, bilirubin, AST, GGT and
lipase
?Renal disease
Consider drug/alcohol
Hepatocellular damage
?Acquired cholinesterase deficiency
?Genetic variant
?Exposure to organophosphates
Suggest urea, electrolytes, creatinine
Suggest genotyping if required
?Possible suxamethonium/
succinylcholine sensitivity
Suggest medical warning card
?Chronic/acute hepatitis
Abnormal cholinesterase phenotype
?Alcoholic liver disease
Suggest full blood count
Suggest repeat liver function tests post op
?Hypothyroidism
Suggest repeat cholinesterase
Inhibition test maybe inconclusive
NOT SUPPORTED KEY
WORDS
?Uraemia
Steatohepatitis
Autoimmune/viral hepatitis
?Diabetes
Suggest ultrasound
?Liver cirrhosis
?Oestrogen therapy
Low cholinesterase can’t be due to liver
disease
Consistent with cholecystitis
MISLEADING KEY WORDS
Suggest renal function test
?Collagen disease
Case 14-09
Cardiac
Patient ID
63-year-old female
Patient Location
Emergency Department
Clinical Notes on Request Form
Chest pain since morning. To exclude acute myocardial infarction.
Case Details
Troponin I
Collection Time
11:30pm
Result
<2 ng/L
Reference Interval
<16 ng/L
Additional Information
Past medical history: nil of note.
ECG, urea, electrolytes and creatinine and full blood examination normal.
Troponin performed on Abbott Architect by hs-TnI assay; 10% CV at 6 ng/L.
Suggested Comment
An undetectable troponin measured with a high sensitivity assay more than 6 hours after onset of chest pain in
the absence of clinical and/or electrocardiogram (ECG) evidence would indicate a low probability of acute
myocardial infarction (AMI). Suggest repeat measurement in 2–3 hours if clinically indicated.
Rationale
The third universal definition of acute myocardial infarction (AMI) requires the detection of a rise and/or fall
of a cardiac biomarker above the 99th percentile of a reference population (15 ng/L in this case) together with
evidence of ischaemia (clinical symptoms, electrocardiogram changes or imaging evidence for loss of viable
myocardium). The major advantage of high sensitivity-cardiac troponin (hs-Tn) assays is to allow the earlier
exclusion of AMI in emergency departments in low risk patients presenting with chest pain. Diagnostic
sensitivity approaching 100% may be achieved by measuring a second sample within three hours of
presentation and 6 hours of onset of pain.
An accelerated diagnostic protocol suggests that a TIMI (thrombolysis in myocardial infarction) score of ≤1, a
normal ECG and troponin at 0- and 2-hours in patients presenting <12 hours after onset of pain will safely rule
out an AMI. Whilst an undetectable hs-cTnT at presentation has very high negative predictive value, which
may be considered to rule out AMI in low risk patients, this has not been shown with hs-cTnI assays.
Ultimately, the decision for repeat measurement/s has to be guided by clinical presentation. The actual
infarction (luminal occlusion) may occur at any stage from symptom onset. Hence, if this occurred just prior to
presentation (which may have prompted the late presentation), then a single sample at presentation may miss
the rise in troponin.
References
1. Cullen L, Mueller C, Parsonage WA, et al. Validation of high-sensitivity troponin I in a 2-hour
diagnostic strategy to assess 30-day outcomes in emergency department patients with possible acute
coronary syndrome. J Am Coll Cardiol 2013;62:1242-9.
2. Body R, Carley S, McDowell G, et al. Rapid exclusion of acute myocardial infarction in patients
with undetectable troponin using a high-sensitivity assay. J Am Coll Cardiol 2011;58:1332-9.
3. Thygesen K, Mair J, Giannitsis E, et al. How to use high-sensitivity cardiac troponins in acute
cardiac care. Eur Heart J 2012;33:2252-7.
PREFERRED KEY WORDS
No rise in TnI 12 h post chest pain
Result positive if ≥16 ng/L
TnI not elevated within 6 h of symptoms
Low probability of AMI
Acute myocardial infarction (AMI)
unlikely
Myocardial injury is unlikely
Exclude AMI
If TIMI score 0 or 1 repeat at 2 h
Suggest repeat 2–3 h post sample collect
Suggest TnI 2–3 h post presentation
Suggest repeat TnI in 2 h
Suggest repeat TnI after 3 h
Suggest repeat hs-TnI after 3 h Suggest
repeat 4 h post first sample
Suggest repeat troponin I (TnI) in 3–6 h
Clinical correlation recommended
Exclude other causes for chest pain
Rise and/or fall required for diagnosis
If repeat normal, AMI unlikely
For MI TnI >upper reference limit (URL)
at 3 or 6 h and >50% change
Exclude stable coronary ischaemia
LESS RELEVANT KEY
WORDS
Normal TnI
Results provided exclude AMI
Normal ECG
Suggest repeat 6–8 h post chest pain
?Pulmonary infection/inflammation
Results do not exclude AMI
Suggest repeat ECG
Suggest lipid profile
Suggest repeat TnI at 3 and 6 h
Confirm non-ST elevation myocardial
infarction (NSTEMI; rise TnI >50%
URL)
?Clinical suspicion
Follow local guidelines
50% increase in TnI suggests cardiac
Exclude pericarditis
Result negative if <16 ng/L and change
<50%
Exclude other acute coronary syndrome
Suggest serial 6 h TnI, CK or myoglobin
?Heterophilic Ab on original TnI
Suggest re-run sample-use different
method
Suggest further investigations for
unstable angina
Suggest observe patient
Suggest repeat testing at 9–12hrs
?>30% rise at 4 h post first sample
Suggest D-Dimer
?Aortic aneurysm
?Chest/lung pathology
Suggest BNP/NT-proBNP
Suggest exercise tolerance testing
?Stress
Tn may take ≥6 h to rise post pain
A <30% delta Tn is less likely AMI
?Pulmonary embolism
NOT SUPPORTED KEY
WORDS
Exact time of chest pain not given
?Digestive disorder
Suggest repeat with TnT assay within 24
h
?Musculoskeletal chest pain
Repeat TnI not indicated
Reference interval should be <6 ng/L
?Breast pathology
Suggest thoracic scan
Falsely low TnI with high Hb
concentration
Suggest creatine kinase (CK)/CK-MB
Suggest liver function tests/creatinine
Suggest C-reactive protein
Suggest microbial cultures
Case 12-06
Proteins
Patient ID
50-year-old female
Patient Location
General Practice
Clinical Notes on Request Form
Back pain
Case Details
Serum Protein Electrophoresis:
IgG (kappa) monoclonal band detected.
Quantification: 3 g/L
Additional Information
IgG 7.3 g/L
IgA 0.8 g/L
IgM 0.6 g/L
(6.5–14.5)
(0.7–3.5)
(0.6–2.6)
X-ray of spine, full blood count, ESR and liver function tests are unremarkable.
Suggested Comment
Small IgG (kappa) band with probable mild immune paresis is of uncertain significance at this time.
Recommend first morning urine for electrophoresis and serum free light chains measurement. Serum calcium,
renal function tests and skeletal survey should also be done for completeness. While monoclonal gammopathy
of undetermined significance (MGUS) is likely, clinical consideration of oligosecretory conditions e.g.
amyloid is important. Suggest review of serum and urine by electrophoresis initially in 3 to 6 months.
Rationale
The small band size, heavy chain type IgG and the lack of anaemia, and normal x-ray of the spine all point to
MGUS. Mild immune-paresis is not incompatible with this diagnosis, nor is the presence of free light chains in
urine were these to be found. The diagnosis of MGUS can only be made when serum monoclonal protein is
<30 g/L, clonal bone marrow plasma cells <10% (if performed) and there is absence of hypercalcaemia, renal
impairment, anaemia and bone lesions (CRAB) that can be attributed to the plasma cell proliferative disorder.
Therefore, serum calcium, renal function tests and skeletal survey are required. Smouldering multiple myeloma
is defined by serum monoclonal protein ≥30 g/L and/or ≥10 to <60 percent bone marrow clonal plasma cells
and absence of CRAB complications.
While the overall progression rate to myeloma is 1% per year, the monoclonal band size is the most potent
predictor for progression and this band is very small. IgG band types are also less likely to progress than IgA
or IgM types. While bone marrow biopsy is part of the classification, this is probably unnecessary at this band
size unless CRAB are positive, in which case the diagnosis is myeloma regardless of band size. Small
monoclonal bands can sometimes be associated with other lymphoproliferative disease. Of these, systemic
Amyloid Light-chain (AL) amyloidosis is often diagnosed late; clinical suspicion and serum free light chains
are important for timely diagnosis. On available data, this patient appears to be at low risk for progression;
however, indefinite follow up is essential. A short repeat time is prudent in the first instance and initial check
periods of up to 6 months are suggested in various guidelines.
References
1. Mollee P. Current trends in the diagnosis, therapy and monitoring of the monoclonal
gammopathies. Clin Biochem Rev 2009;30:93-103.
2. Cook L, Macdonald DH. Management of paraproteinaemia. Postgrad Med J 2007;83:217-23.
PREFERRED KEY WORDS
LESS RELEVANT KEY
WORDS
Small monoclonal band ?significance
Borderline immuneparesis of
immunoglobulins
Low normal IgG, IgA, IgM
?Suppression of polyclonal
immunoglobulin
No signs of bone marrow failure
?MGUS
?Primary lymphoproliferative disorder
?AL amyloidosis
Results suggest MGUS
Multiple myeloma unlikely
Exclude multiple myeloma
Suggest urine Bence Jones Protein/urine
protein electrophoresis
Suggest serum creatinine, urea &
creatinine
Suggest serum calcium/phosphate levels
Suggest serum free light chains (FLC
ratio)
Suggest follow up testing
Suggest skeletal survey/?lytic lesions
Suggest urine immunofixation
electrophoresis
Continue to monitor ?progression
Clinical correlation required
Use same lab for consistency
Suggest regular clinical review
Suggest beta-2-microglobulin
Suggest serum
electrophoresis/immunofixation
Suggest urine protein/albumin excretion
?History of auto-immune disease e.g.
rheumatoid arthritis
Monoclonal band - benign paraprotein
?Type of clonal gammopathy
IgG discrepancy due to monoclonal
protein
Back pain common in multiple myeloma
Elevated risk of progression to multiple
myeloma
NOT SUPPORTED KEY
WORDS
Suggest bone marrow biopsy
Suggest liver function tests
Suggest complete blood count/ESR
Suggest CRP
Suggest urine microscopy, culture and
sensitivity testing
Exclude nephropathy/urinary tract
infection
Referral to Haematologist
?Multiple myeloma
No immuneparesis
?Smouldering multiple myeloma
?Inflammation/infection
?Solitary plasmacytoma
Suggest phenotype of plasma cells
MISLEADING KEY WORDS
MGUS unlikely
No evidence suggestive of B cell
lymphoma
Results suggest polyclonal globulins
raised
No comment on these results
?Macroglobulinaemia
Exclude free light chains only myeloma
?Waldenstrom’s macroglobulinaemia
Case 13-04
Proteins
Patient ID
39-year-old woman
Patient Location
General Practice
Clinical Notes on Request Form
34 weeks gestation. Hypertension.
Case Details
Protein
47 g/L
Albumin
18 g/L
Total bilirubin 5 µmol/L
ALP
113 U/L
ALT
6 U/L
GGT
<5 U/L
Uric acid
0.50 mmol/L
(64–83)
(36–46)
(<20)
(35–105)
(<33)
(<38)
(0.14–0.34)
Suggested Comment
The presence of hypertension beyond 20 weeks gestation increases the risk of pre-eclampsia, which also
requires evidence of end organ or foetal involvement. Although hyperuricaemia is common in this disorder, it
is not diagnostic. The significant hypoproteinaemia may indicate proteinuria and should be investigated with a
urinary protein:creatinine ratio. Full blood examination and ultrasound assessment of the pregnancy are also
suggested but should not delay urgent clinical review in a hospital setting.
Rationale
Pre-eclampsia is a multi-system disorder characterised by hypertension and the involvement of one or more
organ systems and/or the foetus. Complications may include renal and liver impairment; haematological
manifestations of haemolysis or disseminated intravascular coagulation; seizure and other neurological
disturbances; pulmonary oedema; foetal growth restriction and placental abruption. The condition can arise de
novo or be superimposed on chronic or gestational hypertension.
Findings such as hyperuricaemia and oedema, while common in this disorder, are not diagnostic features.
Similarly, proteinuria is frequently seen but not required for diagnosis. Urgent clinical assessment is essential
as management is influenced by severity of hypertension and clinical findings in mother and foetus. Although
some women will be managed as an outpatient, initial assessment should be performed as an inpatient. In the
long term, women with history of pre-eclampsia or gestational hypertension are at increased cardiovascular
morbidity.
References
1. Brown MA, Lowe SA. Current management of pre-eclampsia. Med J Aust 2009;190:3-4.
2. Society of Obstetric Medicine of Australia and New Zealand. Guidelines for the management of
hypertensive disorders of pregnancy 2008. http://www.somanz.org/pdfs/somanz_guidelines_2008.pdf
PREFERRED KEY WORDS
Hypoproteinaemia
Hypoalbuminaemia
Hypertension associated with gestation
?Pre-eclampsia
?Proteinuria
?Renal loss
Suggest spot urine protein/albumin/
creatinine
Suggest complete blood examinations
including platelet count
Suggest 24h urine protein/albumin/
creatinine
Suggest urine protein
Suggest urgent specialist referral
Suggest clinical review/monitor
Suggest ultrasound/foetal assessment
Phone GP
LESS RELEVANT KEY
WORDS
Hyperuricaemia
ALP consistent with gestation
Hypertension
Hospital (emergency) admission
Suggest coagulation profile
Review to pregnancy reference intervals
Monitor renal function tests
?Nephrotic syndrome
Monitor blood pressure
Monitor haemolysis
Monitor liver function test and lactate
dehydrogenase
?HELLP syndrome
Gestational related oedema
?Protein loss
Significant abnormal results
Monitor platelets
Risk of early delivery
Raised ALT in preeclampsia
?Early onset preeclampsia
Quantification of proteinuria
?Perinatal risk
Exclude HELLP
Pre-eclampsia depends on level of blood
pressure and total protein
Results consistent with gestational
proteinuric hypertension/pre-eclampsia
?Hypertension nephropathy
NOT SUPPORTED KEY
WORDS
Monitor oedema
Monitor urine output
Oliguria
Suggest free T4 and TSH
Suggest repeat sample
Suggest Doppler blood flow studies
Urine microscopy hyaline/granular casts
Monitor proteinuria
MISLEADING KEY WORDS
Protein and albumin consistent with
gestation
Liver function tests normal for gestation
Case 6-07
Lipids
Patient ID
43-year-old male
Patient Location
General Practice
Clinical Notes on Request Form
Nausea and vomiting
Case Details
Total Cholesterol
Triglyceride
13.5 mmol/L (<5.5)
38.5 mmol/L (<1.7)
Additional Information
Albumin
Bilirubin
Alk. Phosphatase
ALT
26 g/L
60 µmol/L
254 U/L
300 U/L
(35–50)
(<20)
(35–135)
(<40)
Suggested Comment
Marked hypertriglyceridaemia and hypercholesterolaemia in a setting of abnormal liver function tests. Suggest
exclude secondary causes such as diabetes, alcohol abuse and obesity. There is an increased risk of pancreatitis
due to high triglyceride concentrations. Suggest measure lipase or amylase levels.
Rationale
Lipid disorders can occur as either a primary event or secondary to some underlying disease. Severe
hypertriglyceridaemia may be due to the rare (<1 in a million) disorder lipoprotein lipase deficiency. Other rare
genetic conditions with a similar presentation include apoC-II and apoA-V deficiency (1).
Secondary causes of severe hypertriglyceridaemia are more common. They are often due to a combined
genetic defect and an acquired condition resulting in an overproduction and delayed clearance of triglyceriderich lipoproteins. Such acquired conditions include diabetes mellitus, alcohol abuse and obesity. The abnormal
liver function tests in this case suggest liver disease secondary to alcohol abuse is a likely cause. Evidence
suggests when triglyceride concentrations are >11 mmol/L, there is an increased risk of chylomicron-induced
pancreatitis and therefore either an amylase or lipase test should be performed (2).
Of note, hypothyroidism, nephrotic syndrome, dysgammaglobulinaemia, cholestatic disorders of the liver,
anabolic steroids and protease inhibitor for HIV treatment are associated with elevated total cholesterol and
LDL. On the other hand, elevated triglycerides and VLDL may be due to underlying chronic renal failure, type
2 diabetes mellitus, obesity, excessive alcohol intake and hypothyroidism, oestrogen and corticosteroid
(endogenous or exogenous) effects.
References
1. Viljoen A, Wierzbicki AS. Diagnosis and treatment of severe hypertriglyceridemia. Expert Rev
Cardiovasc Ther 2012;10:505-14.
2. Ferns G, Keti V, Griffin B. Investigation and management of hypertriglyceridaemia. J Clin Pathol
2008;61:1174-83.
PREFERRED KEY WORDS
Marked hypertriglyceridaemia
Marked hyperlipidaemia
Abnormal liver function tests
?Secondary hyperlipidaemia
?Ethanol
Exclude pancreatitis
Exclude diabetes/suggest glucose
Suggest lipase/amylase
?Clinical history
Phone GP to discuss results
LESS RELEVANT KEY
WORDS
Liver disease
Hypercholesterolaemia
?Familial/primary hyperlipidaemia
?Obstructive liver disease
?Hepatitis/toxin
Cholestasis
Suggest lipoprotein electrophoresis
/ApoE genotype
Urgent refer
Suggest hepatitis serology
Acute pancreatitis
Suggest INR/coagulation studies
Pathologist review
?Chronic liver disease
Suggest viral serology
?Acute illness
No comment
UNACCEPTABLE KEY
WORDS
Repeat on fasting sample
Suggest other liver function tests
Suggest TSH/thyroid function tests
Suggest electrolytes/renal function tests
Suggest HDL/LDL/apolipoproteins
Lipaemic sample - interference
Excess VLDL and ?chylomicrons
Suggest stat urine protein
?Fatty liver
Elevated risk of coronary disease
Suggest dietary/medication treatment
Urgent hepatic ultrasound
?Obesity/metabolic syndrome
Suggest liver biopsy
Repeat tests 6–8 weeks
Suggest full blood examination
Suggest immunoglobulin quantitation
Suggest calcium
Suggest ammonia/lactate/blood gases
Moderate cholesterolaemia
Suggest lipowash/ultracentrifugation
Suggest CK/troponinI/ ECG
?Hepatoma
Suggest anti mitochondrial antibody
?Cushing’s disease/suggest cortisol
Case 7-08
Lipids
Patient ID
33-year-old female
Patient Location
General Practice
Clinical Notes on Request Form
Check-up
Case Details
Plasma lipids (patient was fasting)
Cholesterol
6.0 mmol/L
Triglyceride
0.5 mmol/L
HDL-cholesterol
3.7 mmol/L
LDL-cholesterol
2.1 mmol/L
Additional Information
Cholesterol
Triglyceride
HDL-cholesterol
LDL-cholesterol
Desirable
Target
<5.5
<2.0
>1.0
<3.0
NHF
Target
<4.0
<1.5
>1.0
<2.0
Suggested Comment
The increased total cholesterol is due to the increased HDL-C, which is usually considered to be
cardioprotective, although not necessarily in all cases. The LDL-C is within the desirable target. Oestrogen
therapy, alcohol intake, and genetic causes may be contributory factors. Suggest review in context of absolute
cardiovascular risk status.
Rationale
The reason for the ‘check-up’ in this patient is not stated in the request. Lipid screening is not indicated in a
healthy 33-year-old woman unless she belongs to a high-risk group that includes patients with diabetes
mellitus, previous personal/family history of atherosclerotic disorders, smoking history, hypertension and
obesity (1). Cardiovascular risk management should be based on absolute risk assessment of the patient. Thus
knowledge of the presence or absence of evidence of atherosclerosis or other risk factors is important in
advising action on lipid results.
HDL is more tightly controlled by genetic factors (which determine 40–50% of the variation) than for the other
lipoproteins. In general, plasma HDL concentration is inversely correlated with the prevalence and mortality
rates for coronary heart disease. However, emerging evidence suggests that HDL are heterogeneous particles
and their quality (functional or physical subclasses), rather than simply absolute concentration, may be more
important in their cardioprotective effects. Environmental factors causing a high HDL-C include oral oestrogen
therapy (by ~20%), alcohol consumption (by ~0.1 mmol/L) and extensive aerobic exercise (by 3–9%).
Oestrogens, depending on the type, dose and route of administration, increase synthesis of the principal
apoprotein of HDL, apoA-I. In general, progestagens oppose these effects according to type and dose. Fibrates
increase HDL-C by 10–25%, statins by 5–15%, and niacin by 20–35%. The rare genetic deficiency of CETP is
associated with a high HDL, especially in the Japanese; others include apoA-I mutations. The HDL
concentration in this case may be due to genetic and environmental factors including endogenous oestrogens.
Reassurance is appropriate for a patient with a raised HDL trait who is asymptomatic with no other risk
factors.
Reference
1. U.S. Preventive Services Task Force. Screening for Lipid Disorders in Adults.
http://www.uspreventiveservicestaskforce.org/uspstf/uspschol.htm
PREFERRED KEY WORDS
Total cholesterol predominantly HDL
LDL-C & triglyceride desirable
HDL cardio-protective
Good lipid profile
HDL:LDL/Chol ratio good Low
cardiovascular risk
Review clinical risk factors
?On E2 Rx/pregnant
?Alcohol abuse
LESS RELEVANT KEY
WORDS
Total cholesterol above target
Elevated HDL-C
Suggest repeat tests 2–24 months
?Familial (hyperalphalipoproteinaemia)
?Other drugs
?Family history of cardiovascular disease
?Excessive exercise
Acceptable if not high risk group
?CETP deficiency
Suggest liver function tests
?On lipid medication
Suggest CHD risk calculator
Suggest Apo A &/ B levels
Suggest confirm results
Treatment not indicated
Targets vary with clinical risk
No comment
UNACCEPTABLE KEY
WORDS
Suggest low cholesterol diet/dietician
Suggest thyroid function tests
Suggest lifestyle changes
?Hypothyroidism
Increased cardiovascular risk
Suggest glucose/diabetes lx
Increased cardiovascular risk with very
high HDL
Consider lipid drug Rx
?Diabetes
?Renal disease
?Pre-/analytical error
Suggest renal function tests
?Polycystic ovarian syndrome
Aim for lower total cholesterol on
treatment
Suggest follow-up relatives
Suggest lipoprotein(a)
Suggest CK
?Cholestasis
Case 8-03
Lipids
Patient ID
33-year-old female
Patient Location
General Practice
Clinical Notes on Request Form
Non-alcoholic steatohepatitis (NASH) for a number of years. BMI=38.
Case Details
Cholesterol
Triglycerides
LDL-cholesterol
HDL-cholesterol
7.4 mmol/L
2.0 mmol/L
4.7 mmol/L
1.8 mmol/L
Additional Information
Not on any medication.
Suggested Comment
Raised total and LDL-cholesterol and a history of NASH are both associated with increased risk of
cardiovascular disease. Exclude secondary causes of hypercholesterolaemia and perform a full cardiovascular
risk assessment. Lifestyle interventions to increase exercise and promote weight loss are recommended.
Rationale
Non-alcoholic fatty liver disease (NAFLD) encompasses a spectrum of liver disease from steatosis to NASH,
cirrhosis, and rarely hepatocellular carcinoma may develop as a complication. Prospective studies have shown
that survival with NASH is reduced from cardiovascular- and liver-related causes. NAFLD is strongly
associated with obesity and insulin resistance and other features of the metabolic syndrome. This is a case of a
severely obese lady with documented NASH and lipid studies showing raised total and LDL-cholesterol. The
lipid results in this case are not typical of metabolic syndrome (low HDL and raised triglycerides), making it
important to exclude treatable secondary causes, in addition to doing a full cardiovascular risk assessment.
Although insulin resistance is central to NAFLD, routine insulin measurement is not helpful. Regression of
histological features of NASH has been demonstrated with weight loss.
Reference
1. Riley P, O’Donohue J, Crook M. A growing burden: the pathogenesis, investigation and management
of non-alcoholic fatty liver disease. J Clin Pathol 2007;60:1384-91.
PREFERRED KEY WORDS
Predominantly hypercholesterolaemia
Undesirable LDL-C
Increased risk of cardiovascular disease
Exclude secondary causes
?Hypothyroidism
Suggest fasting glucose
Suggest diet/lifestyle review
Suggest review of cardiovascular risk
factors
Suggest thyroid function tests
LESS RELEVANT KEY
WORDS
Obesity/high BMI
Combined hyperlipidaemia
?Insulin resistance/glucose intolerance
Note history of NASH
Consistent with steatohepatitis/NASH
Suggest liver function tests
Refer to Lipid Guidelines
Repeat fasting lipid profile
Suggest lipid-lowering medication
Hypertriglyceridaemia
Suggest monitor lipids
High risk of Type 2 diabetes
?Patient fasting
Suggest OGTT
Suggestion of metabolic syndrome
?Familial causes
Consider metabolic syndrome
HDL-C level associated with low risk
?Nephrotic syndrome
?Cholestasis
Suggest apolipoproteins
Suggest review by specialist
Risk of cirrhosis
Suggest urine protein
?Pregnancy
?Alcohol intake
?Other metabolic problems
Suggest urea, creatinine & electrolytes
HDL-C due to ethanol
Suggestion of insulin resistance
UNACCEPTABLE KEY
WORDS
Lipid ratio suggests low risk
?Polycystic ovary syndrome
Triglyceride level low risk
Suggest insulin level
Normal LDL-C and HDL-C
Suggest lipid electrophoresis
Cardiac risk ratio - low risk
Suggest androgen levels
?Pancreatic disease
Case 9-02
Lipids
Patient ID
47-year-old male
Patient Location
Emergency Department
Clinical Notes on Request Form
Unwell. Dizzy.
Case Details
Total protein
Albumin
Bilirubin
AST
ALT
ALP
GGT
LD
69 g/L
33 g/L
35 µmol/L
368 U/L
144 U/L
580 U/L
6728 U/L
435 U/L
Additional Information
Na
140 mmol/L
K
4.3 mmol/L
Cl
102 mmol/L
Bicarb
25 mmol/L
Anion gap
13 mmol/L
Creatinine
80 µmol/L
(62–83)
(33–47)
(<20)
(10–45)
(5–45)
(40–110)
(10–70)
(100–200)
(135–145)
(3.2–4.5)
(100–110)
(22–33)
(5–15)
(50–110)
Urea
Urate
Gluc
Lipase
Chol
Trig
3.1 mmol/L (3.0–8.0)
0.45 mmol/L (0.15–0.50)
5.2 mmol/L (3.6–7.7)
164 U/L
(25–300)
30.2 mmol/L (2.6–3.5)
168.4 mmol/L (0.5–2.0)
Suggested Comment
The extremely elevated GGT and increased AST/ALT ratio suggest alcohol-related liver disease. The marked
hypertriglyceridaemia confers an increased risk of acute pancreatitis. A combination of a secondary cause on a
genetic background of dyslipidaemia (lipoprotein lipase and apoC-II deficiency) is a possible cause of the
marked hypertriglyceridaemia. Two secondary causes, diabetes mellitus and renal disease, can be
presumptively excluded by the normal random glucose and creatinine. Other secondary causes of
hyperlipidaemia, including obesity, alcohol and drugs should be sought. Suggest measure the lipid profile after
treatment when the patient is in better health.
Rationale
This patient presented with massively increased triglycerides and liver function test results that suggest alcohol
or drug related liver disease. Although there are many secondary causes of hypertriglyceridaemia that require
exclusion, most do not produce such gross elevations, raising the possibility of a primary dyslipidaemia
aggravated by a secondary insult. Hypertriglyceridaemia (>10 mmol/L) is associated with an increased risk of
pancreatitis, and is the most common cause of acute pancreatitis not due to alcohol or gallstones, accounting
for 7% of all cases. Moreover, hypertriglyceridaemia is a (non-major) risk factor for cardiovascular disease.
Reference
1. Yuan G, Al-Shali KZ, Hegele RA. Hypertriglyceridemia: its etiology, effects and treatment. CMAJ
2007;176:1113-20.
PREFERRED KEY WORDS
Marked elevation of GGT
Mixed cholestatic and hepatitic pattern
Marked hyperlipidaemia
Marked hypertriglyceridaemia
AST/ALT ratio >2
?Alcoholic liver disease
?Alcohol-induced dyslipidaemia
?Primary dyslipidaemia
?Drugs/medications
?Hepatic steatosis
?Lipoprotein lipase/apoCII deficiency
?Dysbetalipoproteinaemia
Consistent with alcoholic hepatitis
Increased risk pancreatitis
Drug/alcohol history
Suggest APOE genotyping
Hyperlipidaemia requires further
investigation
Lipaemia expected to improve with
abstinence
LESS RELEVANT KEY
WORDS
Raised liver function tests
Suggest repeat fasting lipids
Exclude hypothyroidism
?Diabetes mellitus
Suggest lipoprotein electrophoresis
?Increased lipids 2° to liver disease
?Obstructive biliary disease
Suggests chronic liver disease
Suggest imaging/ultrasound
?Cholestatic liver disease
Increased GGT/ALP
?Neoplasm/liver tumour
Predominately cholestatic liver function
tests
Suggest blood ethanol
Exclude nephrotic syndrome
Suggest full blood count/ coagulation
Suggest amylase
Suggest hepatitis serology
Suggest lipoprotein profile
?Cardio/cerebro vascular disease
Suggest clinical review/family history
No evidence pancreatitis
Suggest HDL/LDL cholesterol
measurement
Suggest apolipoprotein A/B
Increased ratio triglyceride/cholesterol
?Increased chylomicrons/VLDL
remnants
?Viral hepatitis
Pancreatitis not excluded
?Familial combined hyperlipidaemia
Suggest other specialised lipid tests
Suggest VLDL/chylomicron
measurement
?Familial combined lipoproteinaemia
Suggest troponin
Suggest iron study
Suggest AFP/HCG/PSA
Suggest carbohydrate-deficient
transferrin
Repeat lipase
Suggest CK
Suggest LD isoenzymes
Raised LD
?Familial hypertriglyceridaemia
?Lipoprotein X
?Obesity-induced dyslipidaemia
Non-alcoholic fatty liver disease
Assess cardiovascular risk factors
Exclude primary biliary cirrhosis
Exclude other causes of liver disease
?Cholangitis
?Pancreatitis
?Hepatosplenomegaly
Increased triglycerides interferes with
lipase measurement
UNACCEPTABLE KEY
WORDS
?Non-fasting
Suggest oral glucose tolerance test
Possible factitious lipids
?Dietary cause for increased lipids
Consistent with acute pancreatitis
Recollect sample
Consider biopsy
No comment
Exposure to petrochemical solvents
Monitor liver function tests
?Gall stones
Repeat liver function tests in 3 days
?Xanthomata
Hyperlipidaemia increases risk of
diabetes mellitus, coronary heart disease
?Zieve’s syndrome
?Infiltrative liver disease
Exclude haemolysis
Case 12-05
Lipids
Patient ID
64-year-old woman
Patient Location
Medical Outpatient
Clinical Notes on Request Form
Diabetes; lipid monitoring
Case Details
Total Cholesterol
Fasting Triglyceride
HDL Cholesterol
LDL Cholesterol
Result
5.9 mmol/L
5.5 mmol/L
1.02 mmol/L
2.1 mmol/L
Target
<4.0
<2.0
>1.0
<2.5
Additional Information
On therapy with Metformin and Sulphonylurea
Suggested Comment
Confirm fasting status and consider secondary causes, especially alcohol use and renal impairment.
Predominant hypertriglyceridaemia consistent with type 2 diabetes. Despite meeting target concentrations, the
HDL-C is low for female gender. LDL-C was measured directly because the Friedewald equation is invalid if
triglycerides exceed 4.5 mmol/L. Although LDL-C meets target for uncomplicated diabetes, it is likely to
underestimate cardiovascular disease (CVD) risk in the presence of raised triglycerides. Consider non-HDLCholesterol (Total – HDL-C), or apoB as preferred CVD risk markers in the presence of raised triglycerides.
Rationale
The moderate predominant hypertriglyceridaemia is adequately explained by the presence of type 2 diabetes,
but alternative causes should be considered and excluded. Exacerbation may cause progression to severe
hypertriglyceridaemia. The HDL target is based on the bottom of the reference interval for men. Women have
concentrations that are about 30% higher than men. Women with diabetes may lose their gender-specific
protection from CVD.
When triglycerides exceed 4.5 mmol/L, the presence of chylomicrons becomes more likely. They differ from
VLDL in terms of composition, so the Friedewald equation no longer applies. Direct measurement of LDL-C
is required. When triglycerides are elevated, cholesterol ester transfer protein acts to reduce HDL-C and
transform LDL into small dense LDL. These small dense LDL have less cholesterol per particle, but do more
damage per particle. In these circumstances, LDL-C will underestimate CVD risk.
There is one apoB per atherogenic particle, so this is the best way to measure harmful lipoproteins when LDL
has been confounded by elevated triglycerides. Non-HDL-Cholesterol (Total – HDL-C) is the next best option.
PREFERRED KEY WORDS
High triglycerides may affect LDL result
High triglycerides may affect HDL result
Consistent with diabetic dyslipidaemia
Confirm fasting specimen
?Poorly controlled diabetic
?Alcohol excess
?Renal involvement / suggest renal
function tests
Consider fibrate therapy
Assumed LDL measured not calculated
Exclude secondary causes
LESS RELEVANT KEY
WORDS
Hypertriglyceridaemia
Suggest modify lifestyle
Clinical history noted
High risk of cardiovascular disease
Suggest HbA1c
Target levels different in diabetes
Suggest lipid lowering therapy
Hypercholesterolaemia
Optimise glucose control
Suggest cardiovascular disease risk
assessment
?Patient obese
?Hypothyroidism / suggest thyroid
function tests
Refer to relevant guidelines
Suggest urine microalbumin
Review medication
Moderate hypertriglyceridaemia
Refer National Heart Foundation
Guidelines
Suggest ApoE phenotyping
Elevated cholesterol/HDL ratio
?Levels secondary to diabetes
HDL target >1 mmol/L
LDL target <2 mmol/L
Triglycerides target <1.5 mmol/L
Mild hypercholesterolaemia
Borderline HDL level
Suggest lipoprotein electrophoresis
Marked hypertriglyceridaemia
Total cholesterol target <4 mmol/L
?Smoking
Request previous results
Suggest glucose
Consider statin therapy
Reduced HDL-C
Hyperlipidaemia
?Liver disease
?Insulin resistance/metabolic disease
Triglycerides target <2 mmol/L
Mildly elevated triglyceride
?Frederickson type IV
Normal LDL/HDL ratio 2.06
LDL target <2.5 mmol/L
LDL target <2 if coronary heart disease
present
Highest risk LDL<2.0 HDL>1.0
triglycerides <1.5
High risk LDL<2.5 HDL>1.0
triglycerides <1.7
Elevated triglycerides/HDL ratio
NOT SUPPORTED KEY
WORDS
Monitor lipids
?Type III hyperlipidaemia
?Patient compliance
Request further clinical history
Suggest baseline CK levels/liver function
tests
?Autoimmune / steroids
LDL above diabetes target value
?Increased intermediate density
lipoprotein
?Commence insulin
Moderate hypercholesterolaemia
Refer to Framingham Risk Tables
?Familial dyslipidaemia
Consider niacin therapy
Consistent with Fredrickson Type IIb
pattern
Suggest insulin/OGTT
Marked cholesterolaemia
Suggest familial defective apoB100
mutation
Normal risk TC<5.5 LDL<3.5 HDL>1
triglycerides <1
MISLEADING KEY WORDS
Normal LDL Cholesterol
Normal HDL Cholesterol
Case 13-09
Lipids
Patient ID
24-year-old female
Patient Location
General Practice
Clinical Notes on Request Form
General check prior to embarking on plans to become pregnant
Case Details
Total Cholesterol
HDL Cholesterol
Fasting Triglyceride
LDL Cholesterol
7.9* mmol/L
1.4 mmol/L
1.6 mmol/L
5.7* mmol/L
(≤5.2)
(1.0–2.5)
(≤2.0)
(≤3.0)
Additional Information
Creatinine
Total bilirubin
Alkaline Phosphatase
AST
ALT
GGT
Creatine Kinase
TSH
69 μmol/L
7 μmol/L
56 U/L
28 U/L
34 U/L
14 U/L
106 U/L
0.65 mU/L
(50–90)
(0–18)
(0–145)
(5–55)
(5–55)
(0–40)
(0–175)
(0.35–5.5)
Suggested Comment
Marked hypercholesterolaemia due to LDL accumulation. Many secondary causes, including hypothyroidism,
anorexia and cholestasis, are not supported by intercurrent results. Assess family history and signs (corneal
arcus, tendon xanthomas), estimate Dutch Lipid Clinic Score and calculate absolute risk of cardiovascular
disease. Consider familial hypercholesterolaemia and genetic testing. Cholesterol concentrations are likely to
increase in the event of pregnancy but pharmacological treatment is contraindicated.
Rationale
Familial hypercholesterolaemia (FH) is a prevalent genetic cause of increased LDL cholesterol concentrations
due to mutations in any of several genes (LDLR, APOB, PCSK9) that affect the LDL receptor pathway. The
condition accelerates the progression of atherosclerosis so that cardiovascular disease (CVD) events occur 20
to 40 years sooner. Affected individuals are recognised by the presence of elevated LDL cholesterol
concentrations, a dominant family history of high cholesterol and premature CVD, and the variable presence of
cholesterol accumulation in other tissues. Total cholesterol is a poor discriminator in later age, but elevated
concentrations in young patients are suggestive. The likelihood of FH is best evaluated by applying the Dutch
Lipid Clinic Score. Cases of probable or definite FH warrant consideration of confirmatory genetic testing with
genetic counselling to facilitate Family Cascade Screening.
Reference
1. Watts GF, Sullivan DR, van Bockxmeer FM, et al. A model of care for familial hypercholesterolaemia:
key role for clinical biochemistry. Clin Biochem Rev 2012;33:25-31.
PREFERRED KEY WORDS
Raised LDL-C
Normal liver/thyroid/renal function tests
?Familial hypercholesterolaemia
Check family history
Risk of coronary heart disease
?Tendon xanthoma
?Corneal arcus
Refer Dutch Lipid Clinic criteria
?Primary hypercholesterolaemia
?Familial Defective ApoB-100
Suggest genetic screening
Exclude anorexia nervosa
Lipids increase in pregnancy
Exclude statins during pregnancy
LESS RELEVANT KEY
WORDS
Suggest monitor/change lifestyle
Exclude nephrotic syndrome
Assess cardiovascular disease risk
Hypercholesterolaemia
Significant hypercholesterolaemia
Suggest repeat lipid studies
Suggest serum/urine total protein,
albumin, globulins
Hypothyroidism unlikely
?Oral contraceptive/oestrogen pills
Consult specialist
Normal triglyceride and HDL
Cholestasis unlikely
?Polycystic ovarian syndrome
Nephrotic syndrome unlikely
Manage with Familial
Hypercholesterolaemia Australasia
Network Consensus
?Isolated hypercholesterolaemia
?Familial combined
hyperlipoproteinaemia
Exclude pregnancy
Exclude nephrotic syndrome and
cholestasis
Consistent with Frederickson Type IIa
Raised cholesterol and HDL ratio
Monitor thyroid function tests during
pregnancy
Check post-partum
Manage with National Vascular Disease
Prevention Alliance Guidelines
Pregnancy not contraindicated
NOT SUPPORTED KEY
WORDS
Exclude diabetes mellitus
Suggest fasting glucose
?Medications/alcohol
Suggest thyroid function tests
Risk of pre-eclampsia
?Hypothyroidism
Suggest urea, creatinine, electrolytes,
liver function tests and CRP
Suggest full lipid studies
Risk of hypertension
Exclude Cushing's syndrome
Risk of preterm birth
Suggest oestrogen, FSH, LH, prolactin,
17-ketosteroids
?Secondary hypothyroidism
Suggest albumin:creatinine ratio
Suggest repeat lipid studies 1–6 months
?Secondary hypercholesterolaemia
?Hypergammaglobulinaemia
Suggest serum electrophoresis
?Dysglobulinaemias
?Metabolic disorder
Exclude insulin resistance syndrome
Suggest serum insulin
Repeat lipid studies in 3 months
Suggest assess pituitary function
Refer to New Zealand Guidelines
MISLEADING KEY WORDS
Suggest lipid lowering drugs
Risk of atherosclerosis in infant
Lower lipids pre-pregnancy
Suggest statins
Case 6-03
Thyroid
Patient ID
55-year-old female
Patient Location
General Practice
Clinical Notes on Request Form
Subclinical hypothyroidism, repeat as suggested.
Case Details
TFTs
Free T4
TSH
Additional Information
Fasting Plasma
Cholesterol
Triglyceride
Glucose
Previous Results:
Free T4
TSH
14 pmol/L (10–23)
3.4 mIU/L (0.40–4.00)
4.5 mmol/L
1.3 mmol/L
4.9 mmol/L
12 pmol/L
4.1 mIU/L
Suggested Comment
Borderline TSH persists. Consider repeating in one year with thyroid autoantibodies (TPO antibodies).
Rationale
TSH has an intra-individual (day-to-day) biological variability of 19.7%. Incorporating this biological
variability, the 95% confidence interval of any TSH result is ±40%, and higher if analytical variability is also
considered. Therefore, both TSH results are borderline with respect to the upper reference limit. Similarly,
population studies have failed to separate TSH levels of >2.5 mIU/L from levels of >4.0 mIU/L when
predicting future development of hypothyroidism. TSH exhibits diurnal variation, peaking at night during sleep
and decreasing by 50% by 8 a.m. and remains relatively constant until late afternoon, when a smaller trough
occurs. The afternoon trough often disappears in disease. The diurnal variation is probably of little relevance to
routine testing as blood sampling occurs mostly during the nadir. Elevated TSH concentrations are often seen
in the elderly, with the 97.5th percentile increasing by 0.3 mIU/L/decade of life after 30–39 years old. In older
population, mild elevation in TSH may represent natural aging process instead of pathology.
Subclinical hypothyroidism is characterised by TSH above upper reference limit and a normal free T4. The
risk of progression to overt hypothyroidism is about 2–3% per year, but this increases significantly when TPO
antibodies are positive. Although increased cardiovascular risks have been described in patients with
subclinical hypothyroidism, the lack of prospective controlled trial means treatment on this basis remains
debatable. However, thyroxine replacement should be considered in patients with TSH >10 mIU/L,
symptomatic for hypothyroidism, positive TPO antibodies, or evidence of atherosclerotic cardiovascular
disease, heart failure, or risk factors associated with these diseases. Guidelines for subclinical hypothyroidism
generally suggest assessment of TPO antibody and yearly monitoring.
Reference
1. Garber JR, Cobin RH, Gharib H, et al. Clinical practice guidelines for hypothyroidism in adults: cosponsored by the American Association of Clinical Endocrinologists and the American Thyroid
Association. Endocr Pract 2012;18:988-1028.
PREFERRED KEY WORDS
Normal thyroid function tests
Previous results borderline
Suggest monitor thyroid function tests
Suggest anti-TPO Abs
LESS RELEVANT KEY
WORDS
TSH high normal
Risk developing hypothyroidism
?Subclinical hypothyroidism
Normal lipids and glucose
No comment
?Thyroiditis
Subclinical hypothyroidism unlikely
?Transient hypothyroidism
Borderline low thyroid function tests
?Previous non-thyroidal illness
Assess cardiovascular disease risk
Normal fasting glucose
?Diurnal variation effect
History noted
Results similar to previous
No treatment required
?Non-thyroidal illness
?Goitre
?Clinical history
Previous results - ? other causes
No further action
Transient compensated hypothyroid
No clinical risk TSH <10
UNACCEPTABLE KEY
WORDS
Suggest clinical exam
Non-steady state thyroid function tests
Aim lower cholesterol target
Suggest T4 replacement
Suggest FT3
T4 replacement effective
?Drug-induced TSH
Repeat glucose and lipids
?Suboptimal replacement
?Age-related
Suggest lipids and cardiac markers
?Reference range of 1st results
Suggest thyrotropin releasing hormone
stimulation test
Assay CV checked
Consider T4 replacement
Case 8-04
Thyroid
Patient ID
31-year-old female
Patient Location
General Practice
Clinical Notes on Request Form
Feeling tired and lethargic.
Case Details
Free T4
TSH
Free T3
20 pmol/L
0.02 mIU/L
5.5 pmol/L
(9–19)
(0.40–4.00)
(3.0–5.5)
Additional Information
Not on any medication.
Suggested Comment
The suppressed TSH and high-normal free T4 and free T3 suggest hyperthyroidism, commonly due to Graves’
disease; less common causes are thyroiditis, nodular thyroid disease and surreptitious thyroxine intake. TSHreceptor antibodies and thyroid scan may be useful. However, low TSH may be found in early pregnancy,
which should be excluded. If the patient is pregnant, repeat thyroid function tests in 6 weeks.
Rationale
The combination of a suppressed TSH (<0.l mIU/L) and borderline/raised free T3 and free T4 concentrations
suggests hyperthyroidism (due to Graves’ disease, thyroiditis, or surreptitious T4 intake). TSH-receptor
antibodies and thyroid uptake scan may be indicated. However in a woman of childbearing age pregnancy
should be considered, as TSH may be lowered in the first trimester of pregnancy, although such extremes of
TSH and free T4 (in response to a significantly increased hCG) are usually seen in the hyperemesis group. If
she was found to be pregnant, the tests should be repeated in 6 weeks’ time. If free T4 and free T3 were clearly
elevated as opposed to borderline in a pregnant woman, then the diagnosis would be gestational thyrotoxicosis,
which warrants treatment.
Reference
1. UK
Guidelines
for
the
Use
http://www.acb.org.uk/docs/TFTguidelinefinal.pdf
of
Thyroid
Function
Tests.
PREFERRED KEY WORDS
LESS RELEVANT KEY
WORDS
Suppressed TSH
Borderline/elevated free T4
Borderline/elevated free T3
Consistent with hyperthyroidism
Consider early/borderline
hyperthyroidism
Consider subclinical hyperthyroidism
?Thyroiditis
?Graves’ disease
?Pregnant
?Recent thyroxine ingestion
?Thyroid nodules
?Ingestion of iodine
Suggest thyroid antibody studies
Repeat thyroid function tests 1-3 months
Suggest thyroid scan
Consider non-thyroid illness
Repeat thyroid function tests
?Goitre
Repeat thyroid function tests 3-6 months
?Thyroid disease
Repeat thyroid function tests <1 month
Suggest follow-up/review
Monitor thyroid function tests regularly
?Non-T4 medication
?Patient post-partum
?Treated thyroid disease
Recommend hCG
TSH not fully suppressed
Symptoms not consistent with
hyperthyroidism
Suggest erythrocyte sedimentation rate
Suggest thyroglobulin
Normal free T4
Consistent with euthyroidism
Suggest endocrine review
Suggest specialist review
?Positive thyroid antibodies
UNACCEPTABLE KEY
WORDS
Suggest clinical examination
?Pituitary dysfunction
Suggest glucose
Suggest thyroid ultrasound scan
Resolved thyrotoxicosis
Suggest FSH/LH/PRL/GH/ACTH
Suggest liver function tests and lipids
?Ingestion of caffeine
Suggest TRH
?Immune-deficient
Case 10-05
Thyroid
Patient ID
21-year-old woman
Patient Location
Emergency Department
Clinical Notes on Request Form
32/40 pregnant, headache.
Case Details
Free T4
TSH
Free T3
21.2 pmol/L (10.0–19.0)
<0.03 mIU/L (0.50–4.00)
4.3 pmol/L
(3.5–6.5)
Additional Information
Serum hCG: 5710 IU/L
Suggested Comment
Assuming the subject is not on thyroxine, the TSH is suppressed and free T4 inappropriately elevated for a
woman of this gestational age. This is not hCG driven and is suggestive of subclinical thyroid pathology such
as Graves’ disease. Suggest measure thyroid antibodies including thyroid-stimulating antibodies and be
vigilant for potential complications of pregnancy.
Rationale
It is tempting to interpret the results given as normal for pregnancy, although this is not the case. During early
pregnancy, the reference interval for thyroid-stimulating hormone (TSH) is lower than in non-pregnant
women, although this returns to the pre-pregnancy range by the third trimester. Human chorionic
gonadotrophin (hCG) concentrations increase during the first trimester, peaking between 7 and 13 weeks. hCG
has mild TSH-like activity, leading to slightly increased free T4 during early pregnancy and in turn, leading to
a feedback decrease in TSH with balanced thyroid function.
When the hCG elevation or activity is exaggerated, it will lead to the syndrome of gestational transient
hyperthyroidism (or thyrotoxicosis, if symptomatic), which is seen in 2.5–11% of pregnant women. This is
characterised by mildly increased free T4 and suppressed TSH. It is usually associated with hyperemesis
gravidarum and spontaneously resolves within 2-3 weeks. It can be distinguished from Graves’ disease by the
absence of thyroid ophthalmopathy, thyroid antibodies (particularly TSH receptor antibodies) and transient
nature of the illness. The TSH suppression and elevated free T4 are abnormal for the present case (at 32 weeks
gestational age), and are suggestive of thyroid pathology. Moreover, the lower hCG concentration seen in this
case (appropriate for gestational age) could not account for the pattern of thyroid function observed. With
Graves' disease there is an increased risk of complications of pregnancy, and small increased risk of neonatal
thyrotoxicosis, although not usually seen with this degree of abnormality.
Anti-thyroperoxidase autoantibodies (TPOAb) are the most sensitive test for thyroid autoimmune disease,
including Graves’ disease. Anti-thyroglobulin autoantibodies (TgAb) test is less sensitive and is generally not
used for this purpose. Measurement of anti-TSH receptor autoantibodies (TRAb) is a quantitative test for
Graves’ disease, whilst thyroid-stimulating immunoglobulin (TSI) is a bioassay test that characterises the
stimulating effects of TRAb. An elevated TRAb/TSI helps differentiate Graves’ disease from gestational
thyrotoxicosis in the first trimester. Women with Graves’ disease and negative TRAb are highly unlikely to
result in foetal hyperthyroidism. By contrast, high titres of TRAb in pregnancy require careful foetal
monitoring.
References
1. Rodien P, Jordan N, Lefèvre A, et al. Abnormal stimulation of the thyrotrophin receptor during
gestation. Hum Reprod Update 2004;10:95-105.
2. http://www.thyroidmanager.org/chapter/thyroid-regulation-and-dysfunction-in-the-pregnant-patient/
3. Barbesino G, Tomer Y. Clinical review: Clinical utility of TSH receptor antibodies. J Clin Endocrinol
Metab 2013;98:2247-55.
PREFERRED KEY WORDS
LESS RELEVANT KEY
WORDS
UNACCEPTABLE KEY
WORDS
Mildly elevated free T4
Suppressed TSH
Not consistent with 3rd trimester thyroid
function tests
Consistent with hyperthyroidism
Suggestive of hyperthyroidism
?Early hyperthyroidism
?Subclinical hyperthyroidism in
pregnancy
Gestational transient hyperthyroidism
unlikely
hCG consistent with gestational age
TSH suppressed in pregnancy
?Graves' disease
?On T4 replacement
Suggest TRAb / TSI testing
Suggest anti TPO/Tg antibodies
Suggest review by specialist
Repeat thyroid function tests postpartum
Use reference range for pregnancy
Suggest thyroid function tests in newborn
?Clinical review/Patient history
Increased risk pre-eclampsia, intrauterine growth retardation
Monitor thyroid function tests
Repeat thyroid function tests
Normal free T3
?Thyroiditis
Risk of neonatal thyrotoxicosis in Graves'
Increased risk hydrops, preterm delivery
Suggest urine creatine/protein
Suggest serum uric acid, liver function
tests
?Autonomous thyroid nodule
Suggest full blood examination/ESR
Thyroid uptake scan contraindicated in
pregnancy
?Anti-thyroid drug
Interpret pregnant TSH with caution
Suggest FSH/LH/prolactin
Suggest ultrasound scan
Not trophoblastic disease
Measure total T4
Phone results
Subclinical hyperthyroidism no adversity
to pregnancy
Graves' can worsen post-partum
High hCG can cause hyperthyroidism 1st
trimester
Increased risk obstetric complications
?Iodine-induced hyperthyroidism
?Transient gestational hyperthyroidism
?Sick euthyroid
?Assay interference
hCG not consistent with gestational age
?Due to elevated hCG
Euthyroid results
?Trophoblastic disease
?Hypopituitarism
Repeat free T4/free T3 by different assay
Repeat hCG
?Corticosteroid excess
Total T4 of greater value than free T4
Check erythrocyte zinc
Not indicative of thyroid disease
?Germ cell tumour
Suggest thyroid uptake scan
?Transient gestational hyperthyroidism
Suggest TBG
Suggest full pre-eclampsia screen
Case 12-03
Thyroid
Patient ID
30-year-old female
Patient Location
General Practice
Clinical Notes on Request Form
Past history of papillary thyroid cancer
Case Details
TSH
Free T4
0.89 mIU/L
15 pmol/L
(0.5–4.0)
(10–25)
Additional Information
Previous total thyroidectomy.
On thyroxine.
Suggested Comment
In a patient with a history of thyroid cancer, aim to suppress thyroid-stimulating hormone (TSH) below 0.1
mIU/L to minimise the risk of recurrence. If the patient has been disease free for at least 5 years, American
Thyroid Association guidelines suggest maintaining TSH between 0.1 and 0.5 mIU/L may be appropriate.
Rationale
Most papillary thyroid cancers express the thyroid-stimulating hormone (TSH) receptor; high dose thyroxine
TSH suppression therapy reduces tumour cell growth and survival. Meta-analysis of initial management of all,
except stage I papillary thyroid cancer supports suppression of serum TSH below 0.1 mIU/L. This
concentration was probably chosen because of inclusion of historical data using earlier generation TSH assays;
some studies support complete TSH suppression. For low risk patients, initial suppression of serum TSH at or
slightly below normal (0.1–0.5 mIU/L) is considered appropriate.
Longer-term management has lower quality evidence with regards to the degree of TSH suppression
recommended. In persistent disease, the consensus is to maintain suppression below 0.1 mIU/L indefinitely. In
patients who have high-risk disease and are disease free, TSH suppressive therapy should be maintained to
achieve serum TSH concentration of 0.1–0.5 mIU/L for 5–10 years. In patients who are at low risk and are
disease free, the consensus is that TSH may be allowed to rise to between 0.3 and 2.0 mIU/L (evidence level
C).
The risk profile of a patient depends on completeness of tumour resection, presence of tumour invasion/
metastasis, histological features of tumour and presence of 131I uptake outside of thyroid bed after thyroid
remnant ablation. Of note, biochemical surveillance of recurrent disease should include concurrent
measurement of thyroglobulin and anti-thyroglobulin (TgAb) antibodies. An elevated TgAb may interfere with
thyroglobulin measurement, rendering the result unreliable. When present, the titre of TgAb may be used as a
surrogate tumour marker.
Reference
1. American Thyroid Association (ATA) Guidelines Taskforce on Thyroid Nodules and Differentiated
Thyroid Cancer, Cooper DS, Doherty GM, et al. Revised American Thyroid Association management
guidelines for patients with thyroid nodules and differentiated thyroid cancer. Thyroid 2009;19:1167214.
PREFERRED KEY WORDS
TSH <0.1 mIU/L for high risk disease
Inadequate suppression of TSH
TSH suppression recommended
Refer to American Thyroid Association
Guidelines
TSH 0.1–0.5 if high risk disease free
Low-normal TSH OK if 5–10 yrs disease
free
Achieve TSH <0.1 mIU/L
TSH <0.1 mIU/L for persistent disease
TSH 0.3–2.0 if disease free and low risk
Suggest monitor
Suggest thyroglobulin
? Likelihood of recurrence
Suggest thyroglobulin antibodies
LESS RELEVANT KEY
WORDS
Clinical history noted
Euthyroid
Review thyroxine dose
Normal reference range does not apply
?Patient compliance
Increase thyroxine dose
?Extent of the disease
TSH 0.1–2.0 mIU/L for low risk disease
TSH 0.1–0.5 mIU/L for low risk
?Residual thyroid disease
Ideal TSH <0.01 mIU/L
?Treatment by specialist
Ideal TSH 0.1–0.5 mIU/L
Tailor thyroxine therapy
Review patient's current clinical detail
TSH total suppression for high risk
disease
Maintain TSH between 0.1–0.01 mIU/L
TSH should be <0.03 mIU/L
NOT SUPPORTED KEY
WORDS
Adequate thyroxine replacement
Refer to Endocrinologist
Suggest 131-I scan if indicated
Suggest calcium and PTH
Suggest TSH stimulation thyroglobulin
TSH 0.05–0.1 mIU/L for low risk
TSH <0.01 mIU/L for high risk
TSH <0.1 mIU/L for low risk disease
Risk of hyperparathyroidism
FT4 level only if ?compliance
Suggest lipids
Low thyroglobulin negative predictor
MISLEADING KEY WORDS
Inadequate medical history
No comment
Suggest free T3
Average TSH inversely associated with
relapse
Case 13-07
Thyroid
Patient ID
32-year-old woman
Patient Location
General Practice Clinic
Clinical Notes on Request Form
Pre-conception testing. Nil Medication.
Case Details
TSH
Free T4
TPO Antibodies
4.5 mU/L
14 pmol/L
42 kIU/L
(0.5–4.0)
(10–25)
(<30)
Suggested Comment
Confirm subclinical hypothyroidism by repeat testing. If confirmed, consider thyroxine replacement. Poor
pregnancy outcomes have been described in women with raised TSH.
Rationale
Thyroid dysfunction during pregnancy can result in serious complications for both mother and foetus. Current
US Endocrine Society Guidelines recommend prenatal measurement of TSH for individuals considered high
risk for thyroid disease. They include women with personal/family history of thyroid disorders, autoimmune
disorders (including type 1 diabetes mellitus), goitre, symptoms or signs of hypothyroidism, positive thyroid
antibodies, > 30 years old, infertility or history of miscarriage and preterm labour.
Individuals with TSH >2.5 mIU/L confirmed by repeat testing prior to conception, should be treated with lowdose thyroxine therapy with an aim to reduce TSH to <2.5 mIU/L. Subclinical hypothyroidism should be
treated irrespective of whether the TPO antibody is positive or negative. The aim of treatment is to maintain
TSH <2.5 mIU/L in the 1st trimester and <3.0 mIU/L in the 2nd and 3rd trimester, or within method- and
trimester-specific TSH reference intervals, where available. Repeat testing should be performed at 6–12 weeks
and 6 months postpartum as there is a higher incidence of postpartum thyroiditis in antibody positive women.
Therapy can be discontinued if pregnancy is not achieved or postpartum. In the absence of an elevated TSH, a
woman with known elevations of TPO antibodies should be screened for thyroid abnormalities before
pregnancy and during the first and second trimesters.
Reference
1. De Groot L, Abalovich M, Alexander EK, et al. Management of thyroid dysfunction during pregnancy
and post partum: an Endocrine Society clinical practice guideline. J Clin Endocrinol Metab
2012;97:2543-65.
PREFERRED KEY WORDS
Subclinical hypothyroidism
Slightly raised TSH
Maintain TSH in trimester specific
reference range
Increased risk pregnancy/foetal
complications
?Risk of overt hypothyroidism
?Subclinical hypothyroidism
?Early/borderline hypothyroidism
Delay pregnancy until diagnosis
Increased risk of postpartum thyroiditis
Autoimmune thyroid disease
Suggest replacement T4
Suggest T4 replacement - aim TSH<2.5
Suggest referral to endocrinologist
Suggest repeat thyroid function tests
Suggest treatment/counselling
Call GP to discuss
LESS RELEVANT KEY
WORDS
Low positive TPO Ab titre
Suggest monitor thyroid function tests
Normal FT4
Consider Hashimoto's thyroiditis
Increased TSH
Exclude non-thyroidal disease
Repeat thyroid function test in 1–3
months
Suggest lipid studies
Repeat thyroid function test in 6 weeks
Consider patient/family history
?Iodine deficiency
?Evolving autoimmune thyroiditis
?Associated with infertility
Suggest check for goitre
Exclude TSH heterophile antibody
Exclude non-toxic goitre
Suggest physical examination
Suggest TSH receptor Abs
Suggest cortisol levels
Repeat thyroid function tests at early 1st
trimester
?Primary hypothyroidism
NOT SUPPORTED KEY
WORDS
Suggest antithyroglobulin Abs
Suggest monitor TPO Ab levels
Suggest thyroid ultrasound
Suggest ANA/dsDNA/AMA
Suggest rheumatoid factor
Exclude multinodular goitre
Exclude thyroid carcinoma
Exclude hypogonadism
Exclude hypoparathyroidism
Suggest 21-hydroxylase Ab
Suggest transglutaminase IgA Ab
Suggest anti-gliadin Ab
Exclude idiopathic myxoedema
Suggest iodine supplementation
Exclude drug interferences
Suggest glucose/HbA1c tests
Suggest ECG
Suggest CK level
Suggest full blood examination
MISLEADING KEY WORDS
Exclude Grave's Disease
Suggest test for FT3
Euthyroid results
Repeat thyroid function tests in 6 months
Case 10-06
Pituitary
Patient ID
18-year-old female
Patient Location
Medical Ward
Clinical Notes on Request Form
Amenorrhoea, nausea and vomiting.
Case Details
Prolactin
3145mU/L
(<500)
Additional Information
Previous results (a few days ago in Emergency Department)
Prolactin
195 mU/L
All reproductive hormones were within reference intervals and hCG negative.
Suggested Comment
Assuming both samples were measured by the same method and laboratory errors have been excluded, the
most likely cause of a rapid increase in prolactin concentration with the given the history would be medications
that block the dopamine receptors e.g. antiemetics, antipsychotics, antidepressant and antihypertensive.
Recommend repeating prolactin after discontinuation of interfering medication. If hyperprolactinaemia
persists, further investigations are warranted.
Rationale
There are many causes of a raised prolactin including pregnancy, nipple stimulation, stress, prolactinomas,
hypothalamic and pituitary disorders, oestrogens, drugs, hypothyroidism, chest wall injury, and chronic renal
failure. Hyperprolactinaemia may also be due to idiopathic causes and macroprolactinaemia.
In this case, there has been a rapid rise in the prolactin concentration over a few days and it is not possible to
know which result is correct. It is important to exclude any error in the total testing process, such as stress of
phlebotomy, sample mix-up or the use of a different analytical method that may be affected by macroprolactin
or hook effect.
This patient has amenorrhoea, which may be due to the elevated prolactin. She also has nausea and vomiting
that could have been treated with an antiemetic drug. Prolactin concentrations of >3,000 mU/L are usually due
to pregnancy, drugs, prolactinomas, hypothalamic and pituitary disorders. In this case pregnancy has been
ruled out, and prolactinomas, hypothalamic and pituitary disorders usually develop over a longer period of
time and are unlikely to present in such an acute manner. Therefore, the most likely cause of the
hyperprolactinaemia would be medications e.g. antiemetics. The prolactin should be remeasured after stopping
any interfering medications for four days. The amenorrhoea is possibly due to another cause and may warrant
further investigations.
References
1. David SR, Taylor CC, Kinon BJ, et al. The effects of olanzapine, risperidone, and haloperidol on
plasma prolactin levels in patients with schizophrenia. Clin Ther 2000;22:1085-96.
2. Dostál C, Marek J, Moszkorzová L, et al. Effects of stress on serum prolactin levels in patients with
systemic lupus erythematosus. Ann N Y Acad Sci 2002;966:247-51.
PREFERRED KEY WORDS
LESS RELEVANT KEY
WORDS
Marked hyperprolactinaemia
Previous normal prolactin level
Acute rise in serum prolactin
Discrepancy between two prolactin
results
Repeat prolactin - no drugs, no stress
Pituitary disease unlikely
Pituitary disease unlikely, consider drugs
Macroprolactin unlikely
Recent negative hCG noted
Pregnancy unlikely
?Drug interaction
?Tricyclics, antihypertensives,
psychotropic
?Antiemetic medication
?Antipsychotic medication
?Dopamine antagonist drugs
?Acute stress e.g. illness
?Different prolactin methods and units
?Previous result high dose hook effect
Suggest medication review
Suggest repeat prolactin
Suggest clinical review
Retest both samples in dilution
?Pituitary tumour/prolactinoma
Hyperprolactinaemia
Suggest screen for macroprolactin
?Stress, anxiety, exercise
Suggest thyroid function tests
?Hypothyroidism
Suggest pituitary MRI
Suggest repeat hCG
?Wrong specimen
Suggest FSH, LH, oestradiol,
progesterone, 17OH progesterone
?Galactorrhoea/lactation
Suggest pituitary function tests
Suggest cortisol
?Epileptic seizure
Suggest renal function tests and liver
function tests
Rerun with PEG precipitation or different
method
?Hypothalamic dysfunction
Note nausea and vomiting
Exclude heterophile antibodies
Secretion pulses and various stimuli
Amenorrhoea consistent with
hyperprolactinaemia
?Subarachnoid haemorrhage
Re-run both samples to confirm prolactin
?Chest wall damage
Reserpine may cause nausea and
vomiting
Idiopathic hypersecretion of prolactin
UNACCEPTABLE KEY
WORDS
?Non-pituitary disease e.g. renal failure,
cirrhosis, polycystic ovaries
Exclude pregnancy
?Pregnancy
Suggest BMI for anorexia nervosa
?Breast manipulation
Suggest TRH stimulation test
Case 9-05
Adrenal
Patient ID
48-year-old female
Patient Location
Medical Outpatients
Clinical Notes on Request Form
Previous microadenoma
Case Details
Dexamethasone Suppression Test
Day Time (h)
Cortisol (nmol/L)
Reference Interval
1
0900
319
(160–650)
1
2300
1.0 mg oral dexamethasone given
2
0900
286
(<50)
Additional Information
Day 1 0900 AM
ACTH
21 ng/mL (9–51)
Suggested Comment
There was inadequate cortisol suppression following dexamethasone suppression test. Failure to suppress is
consistent with Cushing’s syndrome. However, other causes of non-suppression include exogenous oestrogens,
alcoholism, depression, non-compliance or malabsorption of dexamethasone and due to certain drugs e.g.
phenytoin. Given the past history and an ACTH within the reference interval this is suggestive of pituitarydependent Cushing’s syndrome.
Rationale
Cushing’s syndrome is an uncommon disease of cortisol excess, which may be endogenous or exogenous in
origin. The clinical diagnosis is often not straightforward and requires additional tests. The initial screening
tests include urine free cortisol, late night salivary cortisol, 1 mg overnight or 2 mg 48-hour dexamethasone
suppression test. The former two tests require at least two separate measurements. The 1 mg overnight
dexamethasone suppression test (given between 2300 and 0000 h, with serum cortisol measured next morning
at 0800 h), at a cut-off of 50 nmol/L, has a sensitivity >95% and a specificity of 80%.
False positive results occur with variable absorption and metabolism of dexamethasone. Drugs that induce the
hepatic enzymatic clearance through the CYP3A4 pathway, such as phenytoin and alcohol, can have this
effect. Up to 50% of women taking the oral contraceptive pill fail to suppress due to increased levels of cortisol
binding protein. If an initial test returns positive, the finding should be confirmed with an alternate test. The
diagnosis of Cushing’s syndrome is established by concordantly positive results from two different tests. The
underlying cause of the hypercortisolism should be investigated accordingly. Patients with suspected
Cushing’s syndrome should be referred to an endocrinologist for further evaluation.
Reference
1. Nieman LK, Biller BM, Findling JW, et al. The diagnosis of Cushing's syndrome: an Endocrine Society
Clinical Practice Guideline. J Clin Endocrinol Metab 2008;93:1526-40.
PREFERRED KEY WORDS
Inadequate suppression of cortisol
Suggests Cushing’s syndrome
?Stress, illness, infection, alcohol
?Drug therapy e.g. phenytoin
?Dexamethasone non-compliance/
malabsorption
?Depression
?Cushing’s disease
?Oestrogen therapy
?Enhanced dexamethasone metabolism
?Pseudo-Cushing’s syndrome
Suggest extended DST
Suggest high dose DST
Refer to Endocrinologist
Recommend follow up
Adrenal cause unlikely
LESS RELEVANT KEY
WORDS
UNACCEPTABLE KEY
WORDS
?Obesity, eating disorder, weight loss
Normal baseline ACTH
Suggest recurrence of functional
microadenoma
?Ectopic ACTH production
Suggest ACTH-dependent Cushing’s
disease
Normal baseline cortisol
False positive possible
ACTH not suppressed
Change in hypothalamic/pituitary/adrenal
axis
Review clinical presentation
Suggest pituitary and thyroid function
tests
Suggest MRI
Suggest late evening cortisol level
Measure ACTH with dexamethasone
suppression test
?Diabetes
Suggest CRH stimulation test
?Adrenal tumour
?Exogenous steroid
?Pituitary disease
Suggest salivary free cortisol
?Hyperthyroidism/acromegaly
Suggest 24 hour urine free cortisol
Suggest 2 mg dexamethasone
suppression test
Suggest serum potassium
Adrenocortical hyperfunction
Suggest low dose dexamethasone
suppression test
Suggest day 2 dexamethasone level
Hypercortisolism
Suggest petrosal sinus ACTH and
cortisol
Result inconclusive
Normal response
Excludes Cushing’s syndrome
?Metabolic syndrome
?Cyclic Cushing’s syndrome
Pituitary disease unlikely
Suggest insulin hypoglycaemia test
Exclude pituitary-independent Cushing’s
Case 11-05
Adrenal
Patient ID
65-year-old female
Patient Location
Medical Outpatients (Diabetes Unit)
Clinical Notes on Request Form
Type 2 diabetes; to exclude Cushing's syndrome
Case Details
Dexamethasone Suppression of Cortisol
Day Time (h)
Cortisol (nmol/L)
Reference Interval
1
0900
414
(160–650)
1
2300
1.0 mg oral dexamethasone given
2
0900
329
(<50)
Additional Information
Two weeks earlier:
Dexamethasone Suppression of Cortisol
Day Time (h)
Cortisol (nmol/L)
Reference Interval
1
0900
358
(160-650)
1
2300
1.0 mg oral dexamethasone given
2
0900
<35
(<50)
Suggested Comment
Cortisol is not suppressed after 1 mg dexamethasone in contrast to the results two weeks earlier. This could be
due to a lack of compliance in the current test or specimen mix-up in either the previous or this test. Serum
dexamethasone measurement could clarify compliance and specimen identity. Rare possibility of cyclical
Cushing’s could be considered. Recommend assessment of midnight salivary cortisol or 24 hour urine free
cortisol.
Rationale
Poorly controlled diabetes mellitus can be associated with mild hypercortisolism and abnormal dexamethasone
suppression tests. However, a clearly suppressed test a fortnight previously with a clearly unsuppressed result
now should prompt consideration of non-compliance with dexamethasone in the second test or sample mix-up
in either the first or the second test. Measurement of serum dexamethasone concentration may clarify these, but
the test is expensive and not commonly available.
Increased metabolism of dexamethasone by ethanol abuse or glitazone therapy initiated in the interim is
unlikely in view of the short interval. Other causes of false positive dexamethasone suppression test such as
obesity, depression and oestrogen therapy are not considered for the same reason. Further testing with
midnight salivary cortisol or with 24 hour urine free cortisol would be useful if pre-test probability of
Cushing’s syndrome is high.
References
1. Nieman LK, Biller BM, Findling JW, et al. The diagnosis of Cushing's syndrome: an Endocrine Society
Clinical Practice Guideline. J Clin Endocrinol Metab 2008;93:1526-40.
2. Yanovski JA, Cutler GB Jr, Chrousos GP, et al. Corticotropin-releasing hormone stimulation following
low-dose dexamethasone administration. A new test to distinguish Cushing's syndrome from pseudoCushing's states. JAMA 1993;269:2232-8.
3. Vagnucci AH, Evans E. Cushing's disease with intermittent hypercortisolism. Am J Med 1986;80:83-8.
PREFERRED KEY WORDS
LESS RELEVANT KEY
WORDS
Inadequate suppression of cortisol
Discordant dexamethasone suppression
test compared with previous
?Compliance with dexamethasone
?Acute illness/stress
?Cyclical Cushing’s syndrome
?Test protocol followed
?Mislabel of patient or sample
?Malabsorption/rapid metabolism
?Exogenous steroid/glucocorticoid
?Alterations in medication
?Transient Cushing’s syndrome
Review clinical and medication history
Suggest dexamethasone assay
Suggest repeat dexamethasone
suppression test
Suggest 24 hour urinary free cortisol
Suggest midnight salivary cortisol
Suggest 2x24 hour urinary free cortisol
Suggest 2x late night salivary cortisol
Difficult to interpret in diabetics
Results confirmed by repeat analysis
?Anticonvulsant therapy
?Depression
?Alcoholism
Drug-induced rapid liver metabolism
?Glitazone effect
Suggest ACTH
?Cushing's Syndrome
?False positive
?Obesity
?Pseudo-Cushing’s
?Exogenous oestrogen
Suggest late evening cortisol
?Poorly controlled diabetes
Suggest 2 mg dexamethasone
suppression test
?Reason for repeat
?Change in patient’s condition
Suggest dexamethasone-CRH test
Suggest HbA1c
?Cause of discordant dexamethasone
suppression test
Suggest pre/post dexamethasone 24 h
urine cortisol
?Ectopic ACTH production
?Renal status
Results inconclusive
UNACCEPTABLE KEY
WORDS
Suggest high dose dexamethasone
suppression test
Previous normal DST excludes Cushing’s
Suggest imaging of pituitary and adrenal
Suggest cortisol binding globulin
Case 7-05
Reproductive
Patient ID
22-year-old female
Patient Location
General Practice
Clinical Notes on Request Form
None
Case Details
Serum hCG
14 IU/L (<5)
Additional Information
Two weeks earlier:
hCG
21 IU/L
Clinical notes: LMP 15 weeks ago, inconclusive ultrasound
Two months earlier:
hCG
121 IU/L
Clinical notes: ?pregnant
Suggested Comment
Persistent low human β-chorionic gonadotrophin (hCG) suggests likely miscarriage with retained products of
conception. Differential diagnosis includes low-grade gestational trophoblastic neoplasia or false positive hCG
due to laboratory interference. Sample has been forwarded for analysis by another method. Suggest check
urine pregnancy test.
Rationale
Following the termination of normal pregnancy in the first trimester, the human β-chorionic gonadotrophin
(hCG) concentrations fall in two trajectories. In the first two days, the half-life of hCG is approximately 0.63
days and increases to 4 days in the subsequent two weeks. By contrast, in termination of ectopic pregnancy, the
hCG decreases by approximately 25% in the first two days. In trophoblastic neoplasia or persistent trophoblast
following termination of pregnancy, the hCG concentrations may fall at even slower rate or rise.
Therefore, possibilities in this case are the following: failing ectopic pregnancy, incomplete miscarriage,
failing second pregnancy, analytical problem (hook effect, heterophile antibody). The fact that the level is
falling makes the presence of heterophile antibody less likely. The clinical note of normal menstrual period
supports retained products of conception rather than an artefact. It may be prudent to check the hCG against
another assay.
References
1. van der Lugt B, Drogendijk AC. The disappearance of human chorionic gonadotropin from plasma and
urine following induced abortion. Acta Obstet Gynecol Scand 1985;64:547-52.
2. Silva C, Sammel MD, Zhou L, et al. Human chorionic gonadotropin profile for women with ectopic
pregnancy. Obstet Gynecol 2006;107:605-10.
3. Kadar N, Romero R. Further observations on serial human chorionic gonadotropin patterns in ectopic
pregnancies and spontaneous abortions. Fertil Steril 1988;50:367-70.
PREFERRED KEY WORDS
Slow decay in hCG level
?Recent miscarriage
?Retained products of conception
?Ectopic pregnancy
?Non-viable pregnancy
?Incomplete miscarriage
Suggest assay by different method
LESS RELEVANT KEY
WORDS
UNACCEPTABLE KEY
WORDS
?False positive hCG
Suggest repeat/monitor hCG
?Molar pregnancy
Persistent low hCG
Suggest clinical review
Suggest urine pregnancy test
Decreasing hCG level
Suggest ultrasound or imaging
Suggest further review
Suggest heterophile Ab pretreatment
Patient not pregnant
Suggest oestradiol and progesterone
hCG may be present for 6 weeks
hCG half-life is 1-3 days
?Malignancy
Equivocal pregnancy result
?Non-pregnancy source hCG
?hCG assay type and reference range
Suggest rheumatoid factor
Suggest investigation for miscarriage
?New pregnancy
Low levels for gestational age
Suggest beta/hyperglycosylated hCG
assay
Suggest FSH, LH
Malignancy unlikely
Consider D&C
Suggest prolactin
Previous results noted
Suggest group and Rh Ab
Suggest full blood count
Case 7-06
Reproductive
Patient ID
31-year-old male
Patient Location
Endocrinology Outpatients
Clinical Notes on Request Form
Gynaecomastia
Case Details
FSH
LH
Oestradiol
Testosterone
SHBG
3 U/L
3 U/L
179 pmol/L
6.5nmol/L
18 nmol/L
(<10)
(<9)
(55–165)
(11–40)
(10–70)
Additional Information
Alpha Fetoprotein
4 µg/L
hCG
<2 IU/L
Prolactin
426 mIU/L
(<12)
(<2)
(<500)
Suggested Comment
The results indicate hypogonadotrophic hypogonadism, which is a known cause of gynaecomastia, although
other clinical signs of hypogonadism would be expected. Drug causes, hyperthyroidism, renal failure and
cirrhosis should be excluded. Oestrogen-secreting testicular or adrenal tumour cannot be excluded.
Rationale
Gynaecomastia is the abnormal enlargement of breast tissue in males due to an absolute or relative (to
androgen) excess of oestrogen concentration, or sensitivity. It is most commonly physiologic, being associated
with puberty and aging, or idiopathic. Nonetheless, medically important causes must be excluded.
The most common identifiable causes are drug effects, which must be sought by history. Cirrhosis (particularly
alcoholic cirrhosis), renal failure (with reduced testosterone and elevated gonadotropin values) and
hyperthyroidism (due to increased aromatase activity and SHBG) may be accompanied by gynaecomastia.
These causes are likely to have been identified by general pathology testing before the more specific tests are
requested in the endocrine outpatient setting, as indicated in this case. It is however necessary to ensure that
these factors are excluded.
Primary or secondary hypogonadism are important causes of gynaecomastia and the testosterone and LH
results presented in this case are consistent with secondary hypogonadism. HCG-negative feminising tumours
of the testes or adrenals are rare but may also cause this hormonal profile by feedback inhibition of
gonadotrophins.
References
1. Ismail AA, Barth JH. Endocrinology of gynaecomastia. Ann Clin Biochem 2001;38:596-607.
2. Case Records of the Massachusetts General Hospital. Case 12-2000. A 60-year-old man with persistent
gynaecomastia after excision of a pituitary adenoma. N Engl J Med 2000;342:1196-204.
PREFERRED KEY WORDS
Abnormal oestradiol/testosterone ratio
?Drugs/exogenous oestrogen
? Hypopituitarism/hypothalamic
?Hypogonadotrophic hypogonadism
?Testicular neoplasm
?Liver disease
?Renal failure
?Secondary hypogonadism
?Hyperthyroidism
Suggest urea and electrolytes
Suggest testicular imaging
Suggest thyroid function tests
Suggest liver function tests
Suggest pituitary tests/imaging
LESS RELEVANT KEY
WORDS
Low testosterone
Increased oestradiol
LH/FSH low/normal
?Obesity
Consistent with gynaecomastia
Suggest clinical review
?Alcohol
?Cancer (other/not specified)
Suggest adrenal tests/imaging
?Adrenal neoplasm
?Genetic low testosterone
Suggest karyotyping
Normal AFP, hCG and prolactin
Suggest iron studies
Increased conversion testosterone to
oestradiol
?Haemochromatosis
Suggest repeat tests
Oestrogen secreting tumour
Suggest repeat am sample
Normal SHBG
Androgen deficiency syndrome
Low calculated free testosterone
Suggest testicular tests
UNACCEPTABLE KEY
WORDS
?Primary hypogonadism
No evidence testicular tumour
No evidence of tumour
?Secondary to severe illness
No comment
Refer to Endocrinologist
?Hypothyroidism
Reduced SHBG
Measure free testosterone
?Gender correction/reassignment
Normal free androgen index
?Tissue androgen insensitive
No evidence pituitary disease
Refer to Chemical Pathologist
Suggest tumour markers
Case 10-07
Reproductive
Patient ID
53-year-old female
Patient Location
General Practice
Clinical Notes on Request Form
Hirsutism, hoarse voice, increased libido for investigation. On L-thyroxine replacement.
Case Details
Total Testosterone
10.4nmol/L
(<3.2)
Additional Information
Free T4 and TSH within reference interval.
Prolactin
189 mIU/L
(<500)
Suggested Comment
An androgen-secreting tumour must be excluded given the extremely high serum testosterone and signs of
virilisation. Serum androgen profile including DHEA-S, and imaging would assist in distinguishing between
adrenal and ovarian malignancy. Assess drug history as medication effect is possible but other causes of
androgen excess (PCOS, CAH and Cushing's syndrome) are less likely at this testosterone concentration.
Clinical endocrine review is recommended.
Rationale
In terms of frequency, hirsutism is typically idiopathic or associated with polycystic ovarian syndrome
(PCOS). The diagnosis of PCOS requires two of the following: 1) oligo/amenorrhoea, 2) clinical or
biochemical evidence of hyperandrogenism, 3) polycystic ovaries on ultrasound. Less common causes are
hyperprolactinaemia, drugs, congenital adrenal hyperplasia (CAH) and Cushing's syndrome. 17OH
progesterone is a useful screening test for CAH while Cushing’s syndrome can be evaluated by using 24 h
urinary free cortisol, dexamethasone suppression test or midnight salivary cortisol tests.
Androgen-secreting adrenal or ovarian tumours are rare causes of hirsutism, but should be suspected when the
onset of virilisation is rapid, symptoms appear in the third decade of life or later, or when serum testosterone
concentrations are markedly elevated. This has been variously defined as testosterone concentrations greater
than 5.2–6.9 nmol/L or 2.5 times the upper reference limit for any assay. Although such tumours may cause
more modest testosterone elevations (at concentrations usually seen in PCOS, CAH, etc), it is unusual for these
non-malignant conditions to cause very high testosterone concentrations. DHEA-S, because of its almost
exclusive production by the adrenal glands, is an important follow-up investigation. Concentrations greater
than 13.6 μmol/L suggest an adrenal tumour.
Reference
1. Barbieri RL, Ehrmann DA. Evaluation of premenopausal women with hirsutism. In: Basow DS, ed.
UpToDate. Waltham, MA, 2013.
PREFERRED KEY WORDS
Markedly raised testosterone
Hyperandrogenism
Clinically virilising condition
Unlikely PCOS/Cushing’s given degree
of elevation
?Androgen-secreting tumour
?Adrenal neoplasm/hyperplasia
?Ovarian neoplasm
?Exclude drugs/medications
Suggest ovarian/adrenal imaging
Suggest full androgen profile
Suggest DHEAS/DHEA
Suggest androstenedione
DHEAS to test adrenal or ovary origin
Refer to endocrinologist
Further testing required
LESS RELEVANT KEY
WORDS
UNACCEPTABLE KEY
WORDS
?PCOS
Suggest FSH/LH
Suggest testosterone/SHBG
?21 hydroxylase deficiency (CAH)
Suggest free testosterone / FAI
Raised testosterone
Suggest 17OH progesterone
?Cushing’s syndrome
Suggest progesterone/oestradiol
Suggest cortisol
Suggest urine adrenal/androgen screen
Suggest LH:FSH ratio
Suggest 24h urine cortisol
Suggest dexamethasone suppression test
Repeat and confirm results
Repeat testosterone by RIA or
LCMS/MS
?Family history
Suggest insulin resistance study
Normal thyroid function tests/prolactin
Suggest ACTH
?Acromegaly
Repeat testosterone after extraction
Suggest glucose
?Ovarian hyperthecosis
Suggest 17OH pregnenolone
Suggest 21 deoxycortisol
Suggest GH
Suggest midnight cortisol
?Trophoblastic tumour
Suggest IGF1
DHEAS/oestradiol cross react with total
testosterone
?11hydroxylase deficiency
Increased androgens associated with
cardiovascular risk
Suggest thyroid function tests and
prolactin
Not hypothyroidism
Suggest adrenal/ovarian tumour markers
Suggest CEA/CA125
?Menopause
?MEN II syndrome
Suggest Synacthen stimulation test
Suggest hCG
Exclude pregnancy
Case 11-02
Reproductive
Patient ID
22-year-old woman
Patient Location
General Practice
Clinical Notes on Request Form
Hirsutism. ?PCOS
Case Details
Fasting serum
Testosterone 3.4 nmol/L
(0.2–2.6)
SHBG
>180 nmol/L (20–120)
FAI
1.7
(<6)
Additional Information
Oestradiol
<75 pmol/L
LH
<1 U/L
Progesterone <0.6 nmol/L
FSH
<1 U/L
Suggested Comment
Elevated SHBG and previously suppressed gonadotrophins are consistent with the use of the oral contraceptive
pill (OCP). OCP use often invalidates biochemical assessment of androgen status. If OCP use is excluded,
suggested further investigation includes thyroid function tests. Note that SHBG concentrations may not return
to baseline for many months after cessation of the OCP.
Rationale
The most common cause of this pattern of results, especially in women of this age, is treatment with the OCP.
Exogenous oestrogens commonly are not detected by routine assays but are bioactive leading to low LH, FSH
and oestradiol and elevated SHBG. Further interpretation depends on confirmation that the patient is on the pill
or not. Note that in this case we are commenting on the testosterone, SHBG and free androgen index (FAI)
results rather than the gonadotrophins and hormones and the comment should focus in this direction.
Given the clinical notes (?PCOS), it is likely that the requesting doctor is seeking biochemical evidence of
hyperandrogenism to support a diagnosis of polycystic ovarian syndrome (PCOS). At this time, the normal
FAI does not support or exclude the diagnosis of PCOS. The preferred approach would be to take the patient
off the OCP and reassess the androgens. The benefits of confirming the diagnosis must be weighed against the
effects of cessation of the pill. If the patient is not receiving the OCP, investigation for other causes of raised
SHBG (and hypogonadotrophic hypogonadism) should be performed. This may include thyroid function tests
(hyperthyroidism causes raised SHBG) and prolactin. Further investigation without confirmation of the
patient’s medications is not warranted.
PREFERRED KEY WORDS
Increased SHBG contributing to
increased testosterone
Mild elevation of testosterone
?Exogenous hormone effect e.g. OCP
Oestrogens cause increase in SHBG
Review patient’s meds/drug history
Repeat tests after cessation OCP
LESS RELEVANT KEY
WORDS
Elevated SHBG
Suggest prolactin
Suppressed FSH/LH
Suggest thyroid function tests
Suggest DHEAS/DHEA
Suggest 17OH progesterone
Suppressed oestradiol/progesterone
?CAH/Late onset CAH
Suggest cortisol
?Hypogonadotrophic hypogonadism
FAI is normal
?PCOS
?Hyperprolactinaemia
Increased testosterone
?Hyperthyroidism
?Increased adrenal/ovarian androgens
Suggest androstenedione
?Excessive weight loss/exercise
Perform pelvic ultrasound
?Hypopituitarism
Hirsutism
?Cushing syndrome
?Tumour/lesion
Suggestive of hyperandrogenism
?Chronic liver disease
Repeat testing
Suggest glucose/insulin/IGF1
Hypothalamic/pituitary disorder
Suggest 24h urine cortisol
Consistent with PCOS
?Pituitary disease
Adrenal/ovarian tumour unlikely
Rotterdam criteria assist PCOS diagnosis
?Exclude phenytoin/carb interference
?Assay interference
Suggest karyotype ?Turner’s
Clinical history required
Suggest ACTH
Suggest refer to
specialist/endocrinologist
Suggest family history
Investigate for porphyria
LH usually increased in PCOS
Normal free testosterone
?FAI should be <1.7
?Mild androgen deficiency
Indicative anovulatory condition
?Ovarian failure
Suggest 3 alpha-AG
?Haemochromatosis/iron studies
Check iron studies
Suggest pituitary imaging
Suggest dexamethasone suppression test
Aromatase inhibitor
SHBG is elevated in hypogonadism
UNACCEPTABLE KEY
WORDS
Not consistent with PCOS
?Adrenal/ovarian cancer
?Pregnancy/suggest hCG
PCOS unlikely
?Idiopathic hirsutism
?Hypothyroidism
Case 13-05
Reproductive
Patient ID
30-year-old male
Patient Location
General Practice
Clinical Notes on Request Form
Erectile dysfunction. ? cause.
Case Details
Total Testosterone
SHBG
1.0 nmol/L
<2.0nmol/L
(8.0–27.8)
(10–70)
Suggested Comment
Severe hypogonadism and very low SHBG indicate exogenous androgen abuse is the more likely cause of the
results. Consider assessing fasting lipids.
Rationale
Sex hormone binding globulin (SHBG) is a protein that is regulated by sex hormones, thyroid and growth
hormone. Oestrogen and thyroid hormone elevate SHBG while androgens and growth hormone suppress it. In
hypogonadal males, SHBG concentrations are higher than in eugonadal men. Consequently, reduced
concentrations of SHBG are unexpected in uncomplicated male hypogonadism and assessment of LH in this
setting is unlikely to be helpful. Of note, pharmacologic androgens may not be detected by routine testosterone
assays and may return suppressed results.
Reduced SHBG can also occur in obesity, hyperinsulinism, hyperprolactinaemia, severe hypothyroidism,
acromegaly, nephrotic syndrome and exogenous androgen abuse. However, the extremely low SHBG
concentration in this case and clinical presentation make androgen abuse a more likely cause of this
combination of abnormal results. Hence, testing for other causes is not recommended until androgen abuse has
been excluded. Other expected biochemical findings in men who abuse androgens include elevated
triglycerides, increased LDL-cholesterol, reduced HDL-cholesterol and increased haematocrit (1).
Reference
1. Dickerman RD, McConathy WJ, Zachariah NY. Testosterone, sex hormone-binding globulin,
lipoproteins, and vascular disease risk. J Cardiovasc Risk 1997;4:363-6.
PREFERRED KEY WORDS
Very low SHBG
?Exogenous androgen use
Suggest fasting lipids
LESS RELEVANT KEY
WORDS
NOT SUPPORTED KEY
WORDS
Suggest LH, FSH, prolactin
Low testosterone
?Diabetes, obesity, metabolic syndrome
Suggest thyroid function tests
?Hypothyroidism
Repeat morning testosterone/SHBG
?Supplements/medications/drugs
Suggest glucose/glucose tolerance
test/HbA1c/BMI
Suggest ACTH, GH, IGF-1, cortisol
Suggest liver function tests/urea and
electrolytes/Iron studies
Low testosterone due to low SHBG
Suggest urine anabolic steroids
Refer to endocrinologist
Evaluate hypothalamus-pituitary-gonadal
axis
Hypogonadism
Unable to calculate free testosterone
?Nephrotic syndrome
Suggest 24h urine protein
?Primary or secondary hypogonadism
?Congenital/genetic disease
?Hyperprolactinaemia/ pituitary tumour
Suggest free testosterone
?Pituitary disease
?Cushing’s syndrome
?Hyperinsulinism
Suggest full blood examination
?Orchitis/ haemochromatosis/ irradiation/
chemotherapy
?Malnutrition, exercise, stress
?Hypothalamic
Erectile problems due to hypogonadism
Consistent with physiological not
psychological
?Liver/renal disease, anaemia
?Kallman’s syndrome, HIV,
inflammation
?Developmental trauma
Secondary hypogonadism
Suggest semen analysis
?Interferences; use different method
Suggest pituitary imaging
Suggest test for haemochromatosis
?Psychological problems
Suggest urine testosterone:epitestosterone
Calculated free testosterone high
Obesity, opiate, pituitary disease less
likely
MISLEADING KEY WORDS
Consistent with androgen deficiency
?Hyperandrogenism
Case 13-10
Reproductive
Patient ID
49-year-old woman
Patient Location
General Practice
Clinical Notes on Request Form
Amenorrhoea
Case Details
hCG 9 IU/L (<2)
Additional Information
FSH
91 U/L
Oestradiol
<37 pmol/L
Reference interval
Follicular
Mid-cycle
Luteal
FSH
U/L
2–10
7–24
1–10
Oestradiol
pmol/L
110–180
550–1650
180–840
Suggested Comment
In post-menopausal women, hCG concentrations up to10 IU/L are not uncommon due to pituitary production.
If a malignant source is suspected, suggest repeat testing after 2–3 weeks of high dose oestrogen to suppress
pituitary hCG.
Rationale
Low concentrations of hCG from pituitary production are common in peri- and post-menopausal women. The
median concentration (range) in women <50 years and women ≥50 is 7 IU/L (2–19) and 10 IU/L (4–33),
respectively (1). FSH levels have a strong negative predictive value for patients with hCG 5–14 IU/L to
discriminate between placental (pregnancy, resolving abortion, gestational trophoblastic disease) and nonplacental sources.
At FSH cut-off of <45 U/L, hCG of placental origin can be identified with 100% sensitivity and 75%
specificity. On the other hand, FSH of >45 U/L is never observed in patients with elevated hCG of placental
origin (2). Additional tests to determine the source of the hCG include: repeat testing; assay free beta hCG;
heterophile antibody testing; and repeat testing after 2–3 weeks of high dose oestrogen oral contraceptive pill
to suppress pituitary hCG (1).
References
1. Cole LA, Laidler LL, Muller CY. USA hCG reference service, 10-year report. Clin Biochem 2010;
43:1013-22.
2. Gronowski AM, Fantz CR, Parvin CA, et al. Use of serum FSH to identify perimenopausal women
with pituitary hCG. Clin Chem 2008;54:652-6.
PREFERRED KEY WORDS
LESS RELEVANT KEY
WORDS
FSH and oestradiol consistent with
peri/menopause
Low +hCG seen peri/post menopause
hCG consistent with benign pituitary
production
Elevated FSH
Mild elevation of hCG
hCG level normal for menopause
Low oestradiol
Consistent with primary ovarian failure
Amenorrhoea noted
Pregnancy not indicated
Exclude heterophile antibodies
Exclude 2° causes of +hCG
Exclude trophoblastic tumour
Exclude hCG of pituitary origin
Exclude malignancy
Exclude germ cell tumour
Exclude non-trophoblastic tumour
Hormone replacement x 2 weeks / repeat
hCG
Suggest serial hCGs
Suggest OCP for 2–3 weeks / repeat hCG
Repeat hCG
Repeat hCG in 1–2 weeks
Premenopause/non-pregnant reference
range
49 year old female
?Specificity of hCG assay
Elevated LH expected
Exclude liver disorders
Reference range of <2 noted
Suggest TSH
?Hypothyroidism
Request further clinical history
Exclude pregnancy
Suggest LH levels
?History gestational trophoblastic disease
Elevated hCG
Markedly elevated FSH
Exclude gastrointestinal tract disorders
NOT SUPPORTED KEY
WORDS
Repeat hCG at 48 h exclude pregnancy
Premature ovarian failure
Repeat FSH/oestradiol
Exclude dyslipidaemia
Exclude familial persistent hCG
Suggest prolactin
Exclude hormone therapy
Suggest progesterone levels
Exclude autoimmune disease
Exclude diabetes
hCG result not normal
Suggest renal function tests
MISLEADING KEY WORDS
Suggest ultrasound
Suggest bone scan
Exclude osteoporosis
Suggest abdominal CAT scan
Case 14-04
Reproductive
Patient ID
22-year-old female
Patient Location
Endocrinology Outpatients
Clinical Notes on Request Form
Amenorrhoea ?polycystic ovarian syndrome.
Case Details
Total Testosterone
DHEAS
SHBG
17-OH Progesterone
Prolactin
5.1 nmol/L
4.2 µmol/L
>180 nmol/L
14.4 nmol/L
2119 mIU/L
(0.5–2.6)
(4.0–10)
(27–109)
(Follicular phase 0.5–3.3; Luteal phase 2.1–9.4)
(59–619)
Suggested Comment
The combination of a markedly elevated sex hormone binding globulin (SHBG) with raised androgens and
prolactin are consistent with pregnancy. Measurement of human chorionic gonadotropin (hCG) is advised.
Rationale
Undiagnosed pregnancy (typically at the first or early second trimester) is always a differential diagnosis for
amenorrhoea. Sustained increases in hCG, oestradiol and progesterone (including 17-OH progesterone) are
early hormonal changes required for the viability of the embryo. During trimesters 1 and 2, sex hormone
binding globulin (SHBG) production is stimulated by the increasing oestrogen levels, and consequently results
in increased total (but not free) testosterone levels. At the same time, DHEAS levels decline due to increased
metabolic clearance while prolactin increases in preparation for lactation. Another significant change (not
included in this case) is a decline in TSH often to values well below those seen in non-pregnancy. The unique
combination of all these findings is only seen in pregnancy and can be easily confirmed with measurement of
human chorionic gonadotropin (hCG).
References
1. Handbook of Clinical Laboratory Testing During Pregnancy. Ed: Gronowski AM. Totowa, NJ:
Humana Press, 2004.
2. McClamrock HD. Androgen Production and metabolism in normal pregnancy. In: UpToDate, Barbieri
RL (Ed), UpToDate, Waltham, MA; 2013.
PREFERRED KEY WORDS
Results consistent with pregnancy
Raised sex hormone binding globulin
(SHBG), testosterone, prolactin
consistent with pregnancy
Exclude pregnancy
Suggest serum human chorionic
gonadotropin (hCG)
LESS RELEVANT KEY
WORDS
Suggest FSH, LH
Exclude macroprolactinaemia
Hyperprolactinaemia
Exclude oral contraceptives
?Congenital adrenal hyperplasia
Results do not suggest polycystic ovarian
syndrome (PCOS)
Elevated 17-OH progesterone
Suggest oestrogen
Raised SHBG, testosterone, prolactin
Raised SHBG - PCOS unlikely
Exclude exogenous hormonal therapy
Raised testosterone due to raised SHBG
Specialist management noted
NOT SUPPORTED KEY
WORDS
Exclude PCOS
Exclude drug related/stress
Suggest synacthen test with 17-OH
progesterone
Suggest ovarian ultrasound
?Pituitary adenoma
Suggest morning cortisol
?Prolactinoma
Exclude ovarian and adrenal tumours
Exclude Cushing's Syndrome
Exclude pituitary disease
Suggest IGF-1
?Anorexia nervosa
Suggest electrolytes and glucose
Suggest 17-OH progesterone
Suggest androstenedione
Repeat morning prolactin
Normal DHEAS - adrenal tumour
unlikely
Suggest liver function tests
Euthyroid
Exclude renal impairment
Suggest insulin
?Cirrhosis
Suggest urinary steroid profile
?Causes of androgen excess
Suggest CYP21A2 gene testing
Clinical correlation recommended.
Suggest repeat tests after 3 months off
oral contraceptive pills
Suggest dexamethasone suppression test
Exclude acromegaly
Raised SHBG ?liver disease
Suggest free testosterone
Confirm with raised blood pressure,
ACTH and progesterone
Suggest progesterone and oestradiol
Normal free androgen index
Investigate virilisation
Exclude obesity
Normal DHEAS excludes congenital
adrenal hyperplasia
If PCOS - risk of gestational diabetes
Elevated prolactin not consistent with
PCOS
?Dopamine suppression
Would not usually comment
Results possibly consistent with
oestradiol therapy
Suggest thyroid function tests
Suggest 2h oral glucose tolerance test
with HbA1c
Suggest fasting lipids
Consider idiopathic hirsutism
Consider hypothalamus amenorrhoea
Suggest HbA1c
Suggest 11-deoxy cortisol
Suggest 11-deoxy corticosterone
MISLEADING KEY WORDS
?Late onset congenital adrenal
hyperplasia
Exclude hypothyroidism
Raised testosterone and SHBG consistent
with PCOS
Results consistent with late onset
congenital adrenal hyperplasia
Hyperandrogenism with elevated free
androgen index
Suggest growth hormone
Normal TSH excludes pituitary adenoma
Raised prolactin and testosterone
consistent with PCOS
Suggest 21-hydroxylase level
Elevated testosterone suggestive of
PCOS
Case 6-08
Iron
Patient ID
44-year-old man
Patient Location
Medical Ward
Clinical Notes on Request Form
Severe anaemia
Case Details
Serum Iron Studies
Iron
Transferrin
Transferrin Saturation
Ferritin
45 µmol/L
23 µmol/L
97%
2170 µg/L
Additional Information
Previous result 4 weeks ago:
Iron
Transferrin
Transferrin Saturation
Ferritin
13 µmol/L
27 µmol/L
23%
122 µg/L
(10–30)
(18–33)
(13–47)
(20–300)
Recently transfused 2 units.
Suggested Comment
These results are incompatible with previous results and suggest iron overload. Markedly elevated ferritin
could be caused by infection/inflammation or liver damage, but one would expect low iron and transferrin in
the acute phase. Suggest liver function tests, C-reactive protein and repeat sample collection.
Rationale
While frequent transfusion, as seen in patients with thalassaemia and after bone marrow transplantation, can
cause iron overload and increase serum iron indices, a single transfusion does not elevate ferritin and only
transiently elevates (<36 hours) iron and transferrin saturation (1, 2). Haemolysis after a transfusion reaction
can release red cell ferritin, but a unit of packed red cells contains around 250 µg/L of ferritin and cannot
explain the markedly changed result. The observed changes could be due to a sample mix-up or a combination
of different factors elevating both the ferritin (inflammation, liver injury) and the iron (iron therapy).
References
1. Ho CH. The effects of blood transfusion on serum ferritin, folic acid, and cobalamin levels. Transfusion
1992;32:764-5.
2. Saxena S, Shulman IA, Johnson C. Effect of blood transfusion on serum iron and transferrin saturation.
Arch Pathol Lab Med 1993;117:622-4.
PREFERRED KEY WORDS
Ferritin too high for transfusion only
?Acute phase/infection /inflammation
?IV/IM iron therapy
?Liver disease
?Sample mix-up
Suggest liver function tests
Suggest C-reactive protein/ESR
Seek more clinical info
Recollect sample
LESS RELEVANT KEY
WORDS
UNACCEPTABLE KEY
WORDS
Increased iron/saturation/ferritin
Iron overload
Repeat 4-12 weeks
Previous normal iron studies
Suggest full blood count/haemoglobin
studies
?Haemolysis
Iron studies unreliable due to transfusion
Iron deficiency unlikely
?Haemochromatosis
?Previous transfusions
Not iron overload
Consult pathologist
Secondary to transfusion
?Transfusion reaction
Suggest haemolysis markers/screen
?Ineffective erythropoiesis
?Chronic disease/malignancy
Suggest B12/folate
Suggest HFE gene testing
Suggest renal function tests/glucose/CA125/other
Haemochromatosis excluded
Anaemia of chronic disease
?Aged/haemochromatosis blood product
Suggest liver biopsy, CT scan
Cease iron therapy
?Acute iron toxicity
Suggest protein electrophoresis
Treat with iron chelators
No comment
Ferritin too high for inflammation only
Case 7-01
Iron
Patient ID
44-year-old female
Patient Location
Medical Ward
Clinical Notes on Request Form
Inflammatory bowel disease. ?Iron deficient
Case Details
Iron
Transferrin
Transferrin Saturation
Ferritin
2 µmol/L
24 µmol/L
4%
57 µg/L
(11–32)
(23–46)
(13–48)
(30–220)
Additional Information
Haemoglobin
Platelets
Leucocytes
Haematocrit
MCV
MCH
MCHC
C-Reactive Protein
100 g/L
371 x109
12.4 x109
0.310
85 fL
27.9 pg
326 g/L
22 mg/L
(115–155)
(150–400 x109)
(4.0–10.0 x109)
(0.36–0.49)
(81–98)
(29.0–33.0)
(320–360)
(<10)
Suggested Comment
A low-normal transferrin and normal MCV suggests that anaemia due to chronic disease is more likely than
iron deficiency. However, a normal ferritin level does not exclude iron depletion in the presence of
inflammation.
Rationale
Anaemia is the most common systematic complication of inflammatory bowel disease (IBD). In patients with
mild/remitting disease, this should be screened for using haemoglobin, C-reactive protein and ferritin every 612 monthly. In active disease, the testing frequency should be at least 3 monthly.
Serum ferritin is a positive acute phase reacting protein and may be raised when inflammation is present. In
IBD patients without clinical or biochemical evidence of inflammation, a serum ferritin of <30 µg/L would
indicate iron deficiency. In the presence of inflammation, the lower limit of serum ferritin indicating normal
iron stores is >100 µg/L.
Red cell indices may help in differentiating iron deficiency anaemia from anaemia of chronic disease, with a
normal MCV favouring the latter diagnosis in this case. However, vitamin B12 and folic acid deficiency, if
present, may complicate the picture, as they cause macrocytosis. The red cell distribution width may help in
this scenario: a low value makes combined effects of vitamin and iron deficiency much less likely.
Measurement of soluble transferrin receptor (with the calculation of TrR/log ferritin ratio) has been advocated
in this situation although it is not widely accepted yet.
References
1. Gasche C, Lomer MC, Cavill I, et al. Iron, anaemia, and inflammatory bowel diseases. Gut
2004;53:1190-7.
2. Gasche C, Berstad A, Befrits R, et al. Guidelines on the diagnosis and management of iron deficiency
and anemia in inflammatory bowel diseases. Inflamm Bowel Dis 2007;13:1545-53.
PREFERRED KEY WORDS
Transferrin borderline low
Normal MCV
Inflammation
Iron study affected by inflammation
Ferritin is acute phase reactant
?Anaemia of chronic disease
?Effect chronic illness
Iron deficiency not excluded
Suggest soluble transferrin receptor
LESS RELEVANT KEY
WORDS
Iron deficiency
Ferritin normal
Elevated C-reactive protein
Iron low
Transferrin saturation low
Anaemia evident
?Infection
Elevated leucocyte count
Suggest repeat at later date
Suggest trial iron therapy
Low MCH/MCHC
Unable to assess iron status
Suggest blood film
Haemoglobin low
Reduced iron stores
Suggest vitamin B12/folate
?Blood loss
?Malabsorption
Ferritin lower than expected
Iron deficiency unlikely
?Nutritional state
Consistent with clinical notes
Suggest occult blood and faecal culture
Suggest reticulocytes
?Intestinal iron loss
Transferrin normal
Suggest bone marrow aspiration
Suggest gliadin antibodies
?Ineffective erythropoiesis
Low haematocrit
UNACCEPTABLE KEY
WORDS
Normal iron stores
?Diurnal variation
?Malignant disease
Suggest liver function tests
Suggest imaging
?Haemoglobinopathy
?Protein losing state
Suggest sigmoidoscopy
Erythrocyte parameters normal
Suggest examination
?Haemolysis
Case 8-10
Iron
Patient ID
57-year-old female
Patient Location
Medical Outpatient Department
Clinical Notes on Request Form
Iron deficiency?
Case Details
Iron
Transferrin
TIBC
Transferrin Saturation
Ferritin
6 µmol/L
3.8 g/L
86 µmol/L
7%
7 µg/L
(9–30)
(2.2–4.0)
(50–90)
(15–45)
(20–150)
Additional Information
Hb
MCV
RDW
PLT
WCC
104 g/L
69.8 fL
16.2%
252 x109
8.6 x109
(110–180)
(80–98)
(<14.5)
(150–400 x109)
(4.0–15.0 x109)
Suggested Comment
Results consistent with iron deficiency. Suggest further investigation for underlying causes such as blood loss,
malabsorption or dietary deficiency.
Rationale
In iron deficiency, serum ferritin, iron and transferrin saturation may be decreased while total iron binding
capacity (TIBC), red cell protoporphyrin and soluble transferrin receptor (sTfR) may be increased. Of these,
serum ferritin is the most powerful test. The diagnostic cut-off is 12–15 µg/L, but <50 µg/L with co-existent
disease. sTfR/logl0 serum ferritin ratio provides superior discrimination in patients with chronic disease.
Microcytosis and hypochromia are sensitive indicators of iron deficiency in the absence of co-existent B12 or
folate deficiency, but are also present in haemoglobinopathies (e.g. thalassaemia), in sideroblastic anaemia and
in some anaemias of chronic disease.
Gastrointestinal blood loss is the commonest cause in adult men and post–menopausal women (exclude colonic
and gastric carcinoma by endoscopy). Malabsorption (exclude coeliac disease), poor dietary intake, blood
donation, gastrectomy and use of non-steroidal anti-inflammatory (NSAID) medications are other common
causes.
Reference
1. British Society of Gastroenterology Guidelines for the Management of Iron Deficiency Anaemia.
http://www.bsg.org.uk/pdf_word_docs/iron_def.pdf
PREFERRED KEY WORDS
Consistent with iron deficiency
Iron deficiency Stage III (anaemia)
Microcytic anaemia
Thalassaemia unlikely
?Blood loss
?Gastrointestinal tract blood loss
?Dietary/nutritional deficiency
?Malabsorption
?Coeliac disease
Suggest gastroscopy/colonoscopy
Suggest coeliac antibodies
Iron supplementation recommended
LESS RELEVANT KEY
WORDS
Low serum ferritin
Low serum iron
Low transferrin saturation
Suggest occult blood
Clinical assessment recommended
High normal transferrin
Increased total iron binding capacity
?Thalassaernia trait
Microcytic
Increased red cell distribution width
Investigate cause, post-menopausal
Monitor iron studies/full blood count
Suggest haemoglobin electrophoresis
Suggest blood film review
Suggest soluble transferrin receptor assay
Anisocytosis
Suggest MCH, red cell count
?Intravascular haemolysis
Depleted iron stores
Monitor therapy with reticulocyte count
Iron deficiency Stage II
Suggest follow up (not specified)
?Abnormal vaginal bleeding
UNACCEPTABLE KEY
WORDS
The facts speak for themselves
?Pulmonary haemosiderosis
Case 11-04
Iron
Patient ID
28-year-old male
Patient Location
General Practice
Clinical Notes on Request Form
?Abnormal LFTs. ?Cause
Case Details
Serum Iron Studies
Iron
60 μmol/L
Transferrin
2.73 g/L
Saturation
88%
Ferritin
1230 μg/L
Haemolysis +
Icterus
++
Additional Information
T. Protein
68 g/L
Albumin
36 g/L
ALP
304 U/L
Bilirubin
134 μmol/L
GGT
300 U/L
AST
2407 U/L
ALT
2019 U/L
LD
505 U/L
(10–27)
(1.50–3.00)
(20–50)
(30–400)
(60–82)
(35–50)
(30–120)
(<25)
(<50)
(<41)
(<51)
(50–280)
Suggested Comment
This patient has acute hepatocellular injury, which can cause markedly elevated ferritin levels. The increased
serum iron concentration raises the possibility of haemochromatosis, although the latter does not usually
present with acute hepatitis. Repeat iron studies after resolution of hepatitis are suggested and if persistently
abnormal, genetic testing for haemochromatosis is indicated. The most common causes of acute hepatitis are
viral hepatitis, alcohol, drugs, toxins, autoimmune hepatitis, and Wilson's disease.
Rationale
In acute hepatocellular injury there are marked increases in ferritin. As the liver ferritin is rich in iron, the
percentage iron in transferrin stays the same. Iron is not commonly elevated in this setting (1). In acute phase
reaction, not only are ferritin levels elevated but iron and transferrin are suppressed, which is not seen here.
The typical changes seen in iron overload and genetic haemochromatosis are elevated iron and iron saturation,
with concomitant high ferritin levels; however, transaminases are not markedly abnormal and only in advanced
cirrhosis do the bilirubin levels increase. Therefore in this case the cause of the acute hepatitis needs to be
investigated and explained before further attention is given to the iron studies.
Reference
1. Brill S, Weinberg M, Graff E, et al. The status of serum iron and transferrin saturation in acute nonhepatotrophic viral infections. J Med 2000;31:271-7.
PREFERRED KEY WORDS
Abnormal iron studies secondary to liver
disease
Acute liver disease
Hepatocellular damage
Interpret iron in context of liver disease
Atypical of haemochromatosis alone
?Alcohol/drugs/toxins
?Hepatitis
?Acute viral hepatitis
?Wilson's disease
?Metabolic/toxic/ischaemic injury
?Autoimmune hepatitis
?Acute alcohol hepatitis
?Infectious mononucleosis
Suggest repeat iron studies when liver
function tests normalise
Suggest hepatitis serology
Suggest EBV serology
Suggest CMV serology
LESS RELEVANT KEY
WORDS
?Haemochromatosis
Suggest HFE gene mutation analysis
?Iron overload
High ferritin
Abnormal liver function tests
Increased transferrin saturation
Elevated serum iron
?Haemolysis
Marked transaminitis
Suggest full blood examination
Abnormal iron studies
Suggest clinical evaluation
?Haematological disorder
?Iron overdose
Suggest drug screen (including
paracetamol)
Suggest autoantibodies
Mixed hepatocellular/cholestasis
?Inflammatory process
?Cholestasis
Suggest repeat iron studies
Suggest clotting studies
?Repeat blood transfusion
?Chronic liver disease
Suggest caeruloplasmin/copper
?Porphyria cutanea tarda
?Anaemia/B thalassaemia major
Suggest conjugated bilirubin
?Acute phase response
?Infection
Repeat iron studies on fasting specimen
?Autoimmune disease
Suggest blood alcohol
Suggest testosterone/thyroid function
tests
Normal transferrin
Suggest imaging studies
?Juvenile haemochromatosis
Suggest repeat liver function tests
?Herbal supplements
Suggest alpha-1-antitrypsin/phenotype
Suggest diabetes mellitus screening
Suggest porphyrin screen
?Fatty liver
Suggest renal function tests
Suggest urine copper
Iron overload unlikely
Haemolysis not cause of abnormal iron
studies
Haemolysis can cause abnormal iron
studies
UNACCEPTABLE KEY
WORDS
Iron overload resulting in liver disease
?Neoplasia
Suggest liver biopsy
Suggest test/monitor AFP
Suggest desferrioxamine challenge test
?Vitamin B12/folate deficiency
Case 13-08
Iron
Patient ID
58-year-old male
Patient Location
General Practice
Clinical Notes on Request Form
Rheumatoid arthritis.
Case Details
Serum Iron
Transferrin
% Saturation
Ferritin
AST
5 μmol/L
3.0 g/L
7%
48 μg/L
31 U/L
Additional Information
Hb
140 g/L
MCV
85 fl
MCH
28 pg
(10–27)
(1.5–3.0)
(15–45)
(30–250)
(5–55)
(130–180)
(82–98)
(27–52)
Suggested Comment
Iron studies suggest non-anaemic iron deficiency in the context of chronic inflammatory disease. Borderline
low MCH also noted. Suggest consider oral iron replacement trial and repeat iron studies and full blood
examination in 3 months with CRP. Consider gastrointestinal investigation for malabsorption (coeliac disease)
or blood loss, if iron stores decline further or anaemia develops.
Rationale
Ferritin is a positive acute phase reactant. In the setting of a chronic inflammatory condition such as
rheumatoid arthritis, a ferritin of 48 μg/L does not guarantee normal iron stores. Rather, a level <50–60 μg/L
together with the low iron saturation, borderline low MCH and borderline high transferrin all support iron
deficiency; anaemia only occurs in the final stage of iron deficiency and is not (yet) present in this case. The
prevalence of gastrointestinal malignancy is low in iron deficiency without anaemia and given the borderline
results in this case, a conservative approach to further invasive investigation seems reasonable.
British guidelines suggest empirical oral iron replacement for 3 months and coeliac screen (by anti-tissue
transglutaminase antibody testing) in this setting, although upper and lower gastrointestinal investigations
could be considered in men >50 years depending on the clinical setting. Finally, patients with rheumatoid
arthritis and iron-deficiency anaemia are likely to have more severe joint disease, but will respond to treatment
if anaemia is corrected.
References
1. Goddard AF, James MW, McIntyre AS, et al. Guidelines for the management of iron deficiency
anaemia. Gut 2011;60:1309-16.
2. Pasricha SR, Flecknoe-Brown SC, Allen KJ, et al. Diagnosis and management of iron deficiency
anaemia: a clinical update. Med J Aust 2010;193:525-32.
3. Wilson A, Yu HT, Goodnough LT, et al. Prevalence and outcomes of anemia in rheumatoid arthritis: a
systematic review of the literature. Am J Med 2004;116 Suppl 7A:50S-57S.
PREFERRED KEY WORDS
Borderline iron stores
Ferritin <100mg/L
Chronic inflammatory rheumatoid
disease
False normal ferritin
?Fe deficiency
Absence of anaemia
Iron deficiency not excluded
Transferrin and ferritin acute phase
reactants
Inflammation masks iron deficiency
Inflammation can elevate ferritin
Possible low iron stores
?Iron deficiency due to
immunosuppressive drugs
?Chronic/acute inflammation
?Blood loss
?Chronic disease
?Iron deficiency with acute phase
reaction
Suggest soluble transferrin receptor
Suggestive of iron deficiency
Suggest CRP/ESR
Monitor full blood examination
Suggest creatinine/liver function tests
Suggest ratio sTfR and log ferritin
Diet and drug review
Monitor 1–2 monthly
LESS RELEVANT KEY
WORDS
Low serum iron
Low transferrin saturation
Normal Hb and erythrocyte indices
Normal ferritin
Borderline/high transferrin
Suggestive of diurnal variation of serum
iron
Low/normal ferritin
Suggest rheumatoid factor
Suggest vitamin B12/erythrocyte folate
Normal transferrin
Monitor ferritin
Suggest repeat fasting early am iron
studies
Low/normal MCV
Check collection time
Suggest hepcidin on GCMS
Suggest reticulocyte-haemoglobin
equivalent and use Thomas plot
NOT SUPPORTED KEY
WORDS
Anaemia of chronic disease
Possible iron deficiency anaemia
Suggest bone marrow aspirate/bone
marrow iron stores
?Normocytic normochromic anaemia
Complicated by anaemia
Suggest erythropoietin
Suggestive of normal iron studies
Iron deficiency anaemia not indicated
Mild disease/no progression or treatment
High ferritin
Anaemia of chronic disease unlikely
Acute phase reaction unlikely
Suggest gastrointestinal endoscopy
MISLEADING KEY WORDS
Suggest faecal occult blood
Iron deficiency unlikely
Suggest iron sufficiency
Normal iron stores
Case 10-03
Tumour Markers
Patient ID
70-year-old woman
Patient Location
Oncology Outpatients
Clinical Notes on Request Form
Mass in head of pancreas (of unknown aetiology) with stent placed 4 weeks ago for obstructive
jaundice.
Case Details
Serum CA 19-9
22 U/mL
Additional Information
Previous result (4 weeks ago):
Serum CA 19-9
5010 U/mL
(<38)
(<38)
Suggested Comment
Provided specimen integrity and resection are excluded, the marked decrease in CA 19-9 is consistent with
relief of biliary obstruction. The CA 19-9 concentration post-stenting does not differentiate a malignant from a
non-malignant lesion and should be evaluated in the context of further clinical investigations.
Rationale
CA 19-9 is the sialylated Lewis blood group antigen, and is absent in individuals with Lewis Le (a-b-) that
make up 5% of Caucasian and 20–25% of African Americans. CA 19-9 concentrations greater than 1000
U/mL are observed in non-malignant conditions including obstructive jaundice, cholangitis, acute liver failure,
acute hepatitis, alcoholic liver disease and acute pancreatitis. In patients with jaundice and elevated CA 19-9, it
is recommended that CA 19-9 be repeated after relief of jaundice. The half-life of CA 19-9 has been variously
estimated at between <1 day and 4-8 days. CA 19-9 is not recommended for the diagnosis of pancreatic cancer
but is recommended for monitoring of treatment of patients with pancreatic cancer.
References
1. Goonetilleke K, Siriwardena A. Systematic review of carbohydrate antigen (CA 19-9) as a biochemical
marker in the diagnosis of pancreatic cancer. Eur J Surg Oncol 2007;33:266-70.
2. Chintanaboina J, Badari AR, Gopavaram D, et al. Transient marked elevation of serum CA 19-9 levels
in a patient with acute cholangitis and biliary stent. South Med J 2008;101:661.
3. Mauer KR, Lopatin RN, Hoffman WA, et al. Decrease in a markedly elevated CA19-9 level after
stenting of a benign pancreatic ductal stricture. Gastrointest Endosc 1995;42:261-3.
PREFERRED KEY WORDS
Marked decrease in CA 19-9
CA 19-9 not a diagnostic tumour marker
Benign/malignant not differentiated
CA 19-9 <38 U/mL tumour not excluded
Half-life is 4–8 days consistent with drop
Drop consistent with obstructed duct
stenting
Non-cancer conditions can cause
elevation
?High CA 19-9 due to obstructive
jaundice
?Normal CA 19-9 due to lab error
?Obstructive hepatobiliary disease
Suggest further investigations
Monitor CA 19-9 in pancreatic cancer
LESS RELEVANT KEY
WORDS
UNACCEPTABLE KEY
WORDS
CA 19-9 normal range post surgery
Pancreatic malignancy unlikely
High CA 19-9 in pancreatic tumour
Suggest CEA to exclude malignancy
Request clinical information
Clinical information not consistent with
Suggest biopsy of pancreatic mass
CA 19-9 drop
High CA 19-9 in pancreatitis
Likely malignancy
Suggest repeat CA19-9
?Heterophile antibody
Monitor CA 19-9
Poor prognosis
Interpret CA 19-9 with caution
?Sclerosing cholangitis/
?Antigen excess/hook effect
cholangiocarcinoma
?High levels due to inflammation
Removal of tumour pancreas/
?Hepatobiliary inflammation
gastrointestinal tract
Raised CA 19-9 in gastrointestinal tract
tumours
?Other causes of high CA 19-9
?Chemo/radiotherapy/resection
Reassay CA 19-9 22 U/mL in dilution
Good response to treatment
?High CA 19-9 due to chronic liver
disease
Suggest imaging
Monitor liver function tests
?Surgery to remove mass
Reassay CA 19-9 22 U/mL sample
No comment
?High CA 19-9 due to cholangitis
CA 19-9 increased in gastrointestinal tract
conditions
CA 19-9 levels related to tumour size
Suggest liver function tests pre/post stent
High CA 19-9 in ovary/ breast tumour
High CA 19-9 in biliary cancer
?Both CA 19-9 done on same platform
CA 19-9 excreted in bile
Pancreatic mass causes jaundice
Post surgery
Exclude autoimmune pancreatitis
Assume hook effect excluded
No evidence of resection
Input of oncologist not required
Case 11-09
Tumour Markers
Patient ID
44-year-old male
Patient Location
General Practice
Clinical Notes on Request Form
Screening
Case Details
PSA
2.2 µg/L
(<2.51)
Additional Information
No previous results
Suggested Comment
A total PSA above the median for this age may be associated with a modest increased risk of prostatic
neoplasia. Suggest repeat with free to total PSA ratio and mid-stream urine examination. PSA sample should
be collected at least 48 hours after any prostate manipulation or ejaculation.
Rationale
The Urological Society of Australia and New Zealand supports selected screening with PSA and digital rectal
examination (DRE) in men after 40 years. It is assumed knowledge for the medical practitioner that the
positive predictive value of PSA is increased when used in combination with DRE. This man’s PSA is above
his age-related median, which is approximately 0.6–0.8 µg/L depending on method. It confers increased risk
for prostate cancer. Assuming that the request in a man of this age was generated due to family history or
significant personal concern, it seems reasonable to try and clarify the patient’s risk in a timely manner rather
than waiting 12 months or more, even though his absolute risk of prostate cancer is small.
There are no guidelines on repeat testing interval. However, in Australia, the Medicare Benefits Schedule will
rebate for one measurement of free to total PSA ratio in a calendar year for this case, where the total PSA is
between the median and the upper limit of age-related reference interval to support early diagnosis. Urinary
tract infections or prostate manipulation due to DRE, sexual intercourse, strenuous bike riding etc can elevate
PSA. Free PSA is labile under normal laboratory storage and handling conditions. Retrospective testing for
free PSA is unreliable unless performed within 24 hours of collection or the serum has been frozen at the time
of sample preparation. A fresh, optimal sample is preferred for free to total PSA ratio estimation.
Reference
1. Urological Society of Australia and New Zealand PSA Testing Policy 2009.
PREFERRED KEY WORDS
PSA above age-related median
Prostatic carcinoma not excluded
?Increased risk of prostatic neoplasia
?Preanalytical variables
Suggest follow-up
Suggest free/total PSA ratio
Suggest clinical correlation
Suggest digital rectal exam
Suggest repeat PSA
Exclude lower urinary tract infection
Monitor PSA using same assay
Consider complexed PSA
Exclude acute causes
LESS RELEVANT KEY
WORDS
PSA within reference limits
PSA within age-related reference limits
Review family history
Suggest repeat PSA in 12 months
Consider PSA velocity
Not an ideal screening test
Suggest repeat PSA ratio in 12 months
Assess other risk factors
Patient should be informed before testing
PSA at upper end of reference range
Suggest repeat PSA in 6 months
Screening benefits not proven
Suggest repeat PSA in 6–12 months
?Reference range
PSA <10 µg/L
Suggest repeat PSA in 2 years
Suitable as a screening test
?Medians for PSA
UNACCEPTABLE KEY
WORDS
Suggest repeat screening in 2–4 years
Check PSA for hook effect
Low PSA
Suggest repeat PSA in 5 years
PSA testing not useful at this age
No comment
Inform clinician of assay calibration
Recheck result with different assay
Case 7-10
Toxicology/TDM
Patient ID
45-year-old man
Patient Location
Emergency Department
Clinical Notes on Request Form
Acute liver inflammation. ?cause
Case Details
Mini toxicology screen (serum):
Paracetamol Not detected
Salicylate
116 mg/L
Ethanol
Not detected
Additional Information
Sodium
141 mmol/L
Potassium
3.9 mmol/L
Urea
1.8 mmol/L
Creatinine
65 µmol/L
Albumin
21 g/L
Total Protein 52 g/L
Bilirubin
408 µmol/L
Alk. Phos.
155 U/L
ALT
3670 U/L
(134–146)
(3.4–5.3)
(3.0–8.0)
(60–110)
(35–50)
(63–80)
(<20)
(35–105)
(<41)
Suggested Comment
Undetectable serum paracetamol does not exclude paracetamol-induced liver toxicity, if the sample was
collected sufficiently long after exposure. Salicylate concentration is below the toxic range for recent acute
exposure or ongoing chronic exposure. Note that hepatocellular damage is an uncommon outcome of salicylate
exposure in adults. Viral studies as directed by history are suggested. Addition of serum AST is suggested.
Rationale
A set of toxicology results has been produced for a patient with liver function tests consistent with acute on
chronic liver disease with a note on the request form recognising the acute liver inflammation and querying the
cause. Of the drugs measured on this request, paracetamol is the most important liver toxin and most frequent
toxicologic cause of liver transplant. Importantly, a negative result doesn’t exclude paracetamol overdose as a
cause of the acute liver damage at a time sufficiently long before the blood test for the paracetamol to have
cleared from the circulation (half-life of paracetamol: 1–3 hours).
Salicylate overdose is a rare cause of acute hepatitis in adults, particularly when no other metabolic
consequences of salicylate toxicity are noted. The undetectable serum alcohol makes this a less likely cause of
the acute hepatitis. Note that for interpretation of all the drug concentrations, history taking and other evidence
are important to determine the possible timing and extent of any overdose. Addition of AST may be of
assistance as an elevated AST/ALT ratio would be consistent with hepatic necrosis due to a toxic insult. Note
only those comments directly related to the results currently being reported are being scored as ‘preferred’ with
exception of a suggestion for viral studies. Comments related to less common causes of hepatitis are marked
‘less relevant’ as they would most likely be second-line tests after common causes have been excluded. Many
submitted comments were directed at interpreting the liver function tests and would not normally be attached
to a set of toxicology results.
PREFERRED KEY WORDS
Paracetamol toxicity not excluded
Salicylate below toxic levels
Salicylate unlikely cause
Ethanol toxicity not excluded
?Time of ingestion
?Drug history
?Viral infection
Suggest hepatitis serology
Suggest ALT/AST ratio
Bilirubin interferes with salicylate
analytically
LESS RELEVANT KEY
WORDS
Acute onset
Hepatocellular damage
Hepatic inflammation/hepatitis
Low albumin
Salicylate toxicity not excluded
High bilirubin
?Toxic insult
Suggest coagulation studies
High ALT
Severe disease
Low urea
Synthetic/conjugation impairment
Underlying chronic disease
Suggest AST/GGT/LDH
Hepatic failure
Paracetamol not detected
Mixed obstructive hepatic
Suggest acid-base investigations
?Clinical history
Monitor salicylate
ALT mildly elevated
Suggest ammonia
Suggest liver imaging
Ongoing insult
?Protein intake/loss
Suggest drug screen
Suggest liver autoantibodies
Suggest further investigations
No evidence renal failure
Suggest clinical examination
?Wilson’s disease
Suggest full blood examination
Suggest haemolysis markers
?Adult Reye’s Syndrome
?Acute biliary obstruction
Suggest conjugated and unconjugated
bilirubin
?Ischaemic insult
Suggest alpha-l-antitrypsin
ALP due to cells swelling
Repeat liver function tests
?Haemolysis
Suggest copper studies
Suggest iron studies
?Alcohol abuser
Discuss with requesting doctor
?Reduced drug clearance
?Autoimmune hepatitis
?Primary biliary cirrhosis
Monitor
Risk of liver failure
Suggest monitor creatinine
Risk of metabolic acidosis
?Non-viral infection
Suggest glucose
?Acute or chronic disease
UNACCEPTABLE KEY
WORDS
Refer to pathologist
Suggest immunoglobulins
Vitamin K may be required
Suggest AFP
Repeat paracetamol
Chronic onset
?Malignancy
Case 8-02
Toxicology/TDM
Patient ID
92-year-old woman
Patient Location
Nursing Home
Clinical Notes on Request Form
?Temporal arteritis. Increased ESR. Diltiazem CR 240mg.
Case Details
Serum carbamazepine level: <3 umol/L
Additional Information
List of medications: isosorbide, diltiazem, aspirin, hydrochlorothiazide, thyroxine, neo-cytamen,
caltrate, ostelin, macu-vision, losec and prednisone.
Suggested Comment
Carbamazepine was undetectable in the sample. Suggest check authenticity of request and compliance to
medication.
Rationale
Carbamazepine is not detected in the patient sample and is not amongst the many medications listed. One
therefore questions whether the request is an authentic one. If so, one may question medication compliance,
although this is usually expected to be well supervised in a nursing home environment. One might consider
drug interaction with cytochrome P450 3A4 (CYP3A4)-inducing agents, leading to a reduced concentration,
although none of the other drugs listed fall into this category. On the contrary, diltiazem is an inhibitor of
CYP3A4 and might be expected to increase carbamazepine levels.
PREFERRED KEY WORDS
Carbamazepine not detected
Check if patient is taking carbamazepine
Carbamazepine not included in drug
history
Subtherapeutic carbamazepine level
?Correct test ordered
Check request correct
Check if carbamazepine prescribed
Results suggest patient not taking
carbamazepine
Other medications unlikely to cause low
carbamazepine
?Correct dose carbamazepine given
LESS RELEVANT KEY
WORDS
Check collection time
?Drug interaction (not specified)
?Hepatic enzyme induction
Review patient’s medication
Repeat/monitor level
?Preanalytical error
No comment
?Malabsorption
?Patient withdrawn from diltiazem
Check electrolytes/renal function
Test because ?accidental ingestion
UNACCEPTABLE KEY
WORDS
Diltiazem increases levels/decreases
clearance carbamazepine
?Non-steady state
Suggest thyroid function tests
Suggest liver function tests
Note diltiazem levels unavailable
Urgent review required
Monitor patient
?Due to interaction with diltiazem
Suggest other tests
Suggest increase carbamazepine dose
Repeat test in steady-state
Consider peak levels
?Patient fitting
?Interference
Suggest measure free carbamazepine
Consider preanalytical factors
Suggest temporal artery biopsy
Review thyroxine replacement
Carbamazepine can alter thyroid function
test results
Case 9-09
Toxicology/TDM
Patient ID
51-year-old female
Patient Location
Emergency Department
Clinical Notes on Request Form
Paracetamol overdose 16 hours ago.
Case Details
Paracetamol
30 mg/L
Additional Information
ALT 165 U/L (<40)
Suggested Comment
Serum paracetamol concentration is potentially toxic and may warrant N-acetylcysteine (NAC) therapy since
the blood concentration is above the toxic line on the treatment nomogram at 16 hours post ingestion.
Moderately increased ALT suggests liver injury. Repeat ALT at end of NAC infusion together with INR, urea
and electrolytes, blood gases and glucose.
Rationale
The absorption of paracetamol in the small intestine is rapid, giving peak serum concentrations within 1-2
hours with pill formulations and sooner with liquid preparations (30 minutes). Ninety percent of absorbed
paracetamol undergoes hepatic conjugation with sulphate and glucuronide, and are excreted in the urine. The
remainder is metabolised into the highly reactive intermediary compound N-acetyl-p-benzoquinone imine
(NAPQI) via the cytochrome P450 (mainly 2E1 and 3A4) pathways.
Normally, the NAPQI is immediately conjugated by intracellular glutathione and excreted in the urine. In the
presence of excess paracetamol, and thence NAPQI, glutathione stores may become depleted, leaving NAPQI
to bind with other proteins, causing hepatotoxicity. When this happens, N-acetylcysteine (NAC) is
administered and works by serving as a glutathione replacement, a free radical scavenger, binding NAPQI
directly and increasing microcirculatory oxygenation. Treatment with NAC within 8 hours can prevent all
serious hepatic injury, but the efficacy decreases with increasing delay.
There are now well-defined guidelines for management of a single episode of acute paracetamol overdose. The
treatment nomogram relates the paracetamol concentration and the known time elapsed since ingestion to
determine potential toxicity and treatment recommendations. However, the treatment nomogram cannot be
applied for repeated or staggered doses, or when time of ingestion is uncertain. ALT is used for assessment of
liver damage and response to NAC therapy. Most patients develop AST/ALT elevations within 24 hours of
ingestion and almost all will have elevations at 36 hours. Maximal hepatotoxicity occurs at 72–96 hours. A
schedule for recommended investigations is detailed.
References
1. Daly FF, Fountain JS, Murray L, et al. Guidelines for the management of paracetamol poisoning in
Australia and New Zealand—explanation and elaboration. A consensus statement from clinical
toxicologists consulting to the Australasian poisons information centres. Med J Aust 2008;188:296301.
2. Algren DA. Review of N-acetylcysteine for the treatment of acetaminophen (paracetamol) toxicity
in paediatrics. Geneva: World Health Organization; 2008.
PREFERRED KEY WORDS
Toxic paracetamol level at 16 hours
Nomogram paracetamol above treatment
line
Elevated ALT consistent with
hepatotoxicity
Risk of hepatotoxicity
Refer to Med J Aust 2008 Guidelines
Treatment required
Treat immediately
N-acetylcysteine therapy recommended
Monitor treatment
Repeat ALT
?Pre-existing liver dysfunction
?Aetiology of increased ALT
N-acetylcysteine effective up to 36 hour
LESS RELEVANT KEY
WORDS
Elevated ALT/transaminase
ALT consistent with paracetamol
hepatotoxicity
Suggest liver function tests
Suggest repeat paracetamol level
Request drug and alcohol history
Suggest electrolytes
Assess extent of hepatic damage
Confirm dose information
?Normal or slow release paracetamol
Suggest AST/ALT
Suggest platelet count
Suggest biochemical reassessment
Suggest blood gas
Liver damage usually >24 hour post
paracetamol overdose
N-acetylcysteine efficacy declines at 18
hour
Contact lab for further advice
Clinical hepatitis
Phone results
?No clinical contraindications
?Fasting state
Suggest anion gap
Suggest full blood picture
Consult poisons information
Pancreatitis reported in paracetamol
overdose
Suggest prothrombin time/international
normalised ratio
Suggest urea and creatinine
Suggest glucose
Is metabolic acidosis present?
UNACCEPTABLE KEY
WORDS
Likelihood of paracetamol toxicity low
Suggest NH4, lactate
Borderline paracetamol level
No comment
Suggest calculation of half life
Case 11-06
Toxicology/TDM
Patient ID
54-year-old male
Patient Location
General Practice
Clinical Notes on Request Form
Polyuria for investigation
Case Details
Serum osmolality
Urine osmolality
315 mmol/kg (275–295)
210 mmol/kg
Additional Information
Sodium
Potassium
Bicarbonate
Urea
Creatinine
Glucose
Calcium
Albumin
Corrected calcium
140 mmol/L
3.8 mmol/L
28 mmol/L
4.5 mmol/L
63 µmol/L
4.3 mmol/L
2.32 mmol/L
43 g/L
2.30 mmol/L
(134–146)
(3.4–5.5)
(22–32)
(3.0–8.0)
(60–105)
(fasting <5.5)
(2.15–2.65)
(38–50)
(2.15–2.65)
Suggested Comment
Consider ethanol ingestion given the raised osmolar gap of 26 mmol/kg in the setting of history of polyuria.
Ethanol can be measured, if required, in blood or detected in urine. Measurement of liver function tests may
support regular ethanol use. A timed urine sample can be useful to separate urinary frequency from polyuria.
Rationale
The raised osmolar gap [= measured osmolality – (2 × sodium + urea + glucose), normally ≤ 10 mmol/kg]
suggests a substance found in significant concentrations such as ethanol, which can cause polyuria due to
nephrogenic diabetes insipidus, rather than lithium etc, which are not found in concentrations to explain an
osmolar gap of >20 mmol/kg. Urine osmolality suggests water diuresis rather than osmotic diuresis. Methanol,
ethylene glycol etc, can cause a raised osmolar gap but in such concentrations would be expected to cause a
significant acidosis and toxic effects. Metabolic acidosis is unlikely given the normal bicarbonate.
References
1. Purssell RA, Pudek M, Brubacher J, et al. Derivation and validation of a formula to calculate the
contribution of ethanol to the osmolal gap. Ann Emerg Med 2001;38:653-9.
2. Ogata M, Mendelson JH, Mello NK. Electrolyte and osmolality in alcoholics during experimentally
induced intoxication. Psychosom Med 1968;30:463-88.
PREFERRED KEY WORDS
LESS RELEVANT KEY
WORDS
Increased serum osmol gap
?Unmeasured osmoles
No metabolic acidosis
?Alcohol
?Ethanol
?Ethanol- induced polyuria
Suggest serum ethanol level
Urine osmolality consistent with water
diuresis
Suggest 24 h/timed urine - check volume
Suggest check liver function tests
Diabetes mellitus excluded
?Medical history/medication
Suggest repeat serum osmolality
Inappropriately low urine osmolality
?Diabetes insipidus
Elevated serum osmolality
?Ethylene glycol
?Methanol
?Mannitol
?Psychogenic polydipsia
Suggest urinary electrolytes
?Polydipsia
Exclude neurogenic diabetes insipidus
Exclude nephrogenic diabetes insipidus
Exclude diuretics
Suggestive of diabetes insipidus
?Hyperproteinaemia
?Hyperlipidaemia
Differentiate central from nephrogenic
diabetes insipidus
Suggest first morning urine osmolality
?Isopropyl alcohol
?Pituitary/head injury
Suggest recollection of urine and serum
Endocrinologist referral
?Sorbitol
Suggest drug screen
?Genetic nephrogenic diabetes insipidus
Nephrogenic diabetes insipidus not
indicated
?Propylene glycol
Renal deficiency not indicated
?Tetracyclines
Consider further investigation
Suggest plasma chloride
Exclude analytical error
Suggest carbohydrate-deficient transferrin
Exclude renal disease/concentration
defect
Suggest urine creatinine level
Exclude specimen contamination
Suggest urine microscopy, culture and
sensitivity
UNACCEPTABLE KEY
WORDS
Suggest water deprivation test
Suggest DDAVP in water deprivation
test
Suggest lithium levels
Suggest ADH levels
?Ketoacidosis
Suggest blood gas
?Lactic acidosis
Suggest head-spacer gas analysis
?Fanconi syndrome
Suggest phenytoin levels
Suggest MRI of brain
Suggest lateral X-ray of skull
Suggest chromosome analysis
Suggest urine amino and organic acids
Suggest renin/aldosterone
Suggest TSH levels
Contact duty biochemist
?Evaporation artefact
Suggest urine glucose
Normal serum chemistry
Case 14-05
Inborn Errors of Metabolism
Patient ID
9-day-old baby boy
Patient Location
Emergency Department
Clinical Notes on Request Form
Term baby, 600 g weight loss and jaundice.
Case Details
Total bilirubin
Conjugated bilirubin
Unconjugated bilirubin
442 µmol/L
74 µmol/L (0–10)
368 µmol/L
Additional Information
Random glucose
1.9 mmol/L
(Urine reducing substances +++)
Suggested Comment
The results are consistent with hyperbilirubinaemia due to galactosaemia. Recommend patient be placed on a
lactose free diet until confirmation of galactosaemia by erythrocyte galactose-1-phosphate uridyltransferase
(GALT) test. During this time other causes of hyperbilirubinaemia should be considered and assessed where
appropriate.
Rationale
This case is a typical presentation of classical galactosaemia in a newborn baby with hyperbilirubinaemia,
hypoglycaemia and positive urine reducing substances. This combination should trigger the possibility of
galactosaemia, even if the baby has had a normal newborn screening test (NBS). This is because the clinical
features of galactosaemia may be delayed until breast-feeding has commenced. Furthermore, false negative
NBS may be observed in patients who have a blood transfusion or soy-based formula prior to the NBS.
Classic galactosaemia is an autosomal recessive disorder of carbohydrate metabolism secondary to severe
deficiency of the galactose-1-phosphate uridyltransferase (GALT) enzyme. Lactose should be eliminated from
the diet immediately and samples should be collected for erythrocyte GALT and galactose-l-phosphate (Gal-1P). In some centres, urinary galactitol is measured. Failure to cease lactose allows toxic metabolites to continue
to accumulate resulting in rapid progression to liver failure, sepsis and death. Other causes of
hyperbilirubinaemia should be excluded.
Note: not all Australian States (e.g. Victoria) offer galactosaemia testing as a part of NBS.
References
1. Gilmour SM. Prolonged neonatal jaundice: when to worry and what to do. Paediatr Child Health
2004;9:700‐4.
2. Malone JI, Diaz‐Thomas A, Swan K. Problems with the newborn screen for galactosaemia. BMJ
Case Rep 2011; doi 10.1136/bcr.01.2011.3769.
3. Berry GT. Galactosemia: when is it a newborn screening emergency? Mol Genet Metab
2012;106:7‐11.
PREFERRED KEY WORDS
Conjugated and unconjugated
hyperbilirubinaemia
?Galactosaemia
?Inborn error of metabolism
Suggest red cell GALT activity
Suggest lactose free diet
Suggest galactose-1-phosphate
Suggest galactitol
LESS RELEVANT KEY
WORDS
NOT SUPPORTED KEY
WORDS
Hyperbilirubinaemia - investigate
Hypoglycaemia
Reducing substance in urine
Suggest liver function test/ thyroid
function tests/ renal function tests/
electrolytes
Suggest specialist referral
Weight loss/failure to thrive
Unconjugated hyperbilirubinaemia
Check newborn screening results
Conjugated hyperbilirubinaemia
Suggest metabolic screen
Suggest amino acids
Suggest liver ultrasound
Liver failure/dysfunction/obstruct
Conjugated bilirubin >15% of total investigate
At risk of kernicterus
Suggest C-reactive protein/ erythrocyte
sedimentation rate
Suggest blood gas
Suggest urine chromatography for
galactose
Suggest α-1 antitrypsin
Hospital admission
Breast fed/lactose formula
Suggest ketones
?Fatty/organic acid disorders
Review neonatal hypoglycaemia
Suggest urine non-glucose reducing
substances
?Glycosuria
Give glucose
Suggest test parents for galactosaemia
Accumulation of galactose-1-phosphate/
galactose
Isoelectric focusing for Duarte variants
?Deficiency of gluconeogenic enzyme
Repeat LFT to monitor progress
Conjugated bilirubin >30 µmol/L
pathological
Suggest lactate
Suggest urine glucose
?Acute infection/sepsis
Suggest full blood count including
morphology
?Red cell destruction/haemolysis
?Hypothyroidism
Suggest Coombs test
Suggest septic screen
Phototherapy/exchange transfusion
?Neonatal hepatitis
Suggest Rubella/Toxo/CMV/HCV/HepB
?Glucose-6-phosphate dehydrogenase
deficiency/phenylketonuria
Suggest ammonia
?Hereditary fructose intolerance
Suggest blood group/antibody test
?Tyrosinaemia
Suggest clotting studies
?Maternal diabetes
Suggest urine microscopy
Suggest urine bile acids
Suggest insulin
? α-1 antitrypsin deficiency
Suggest cortisol/growth hormone
Suggest DNA testing
ABO/Rh incompatibility
?Endocrine disorder
?Lactose intolerance
Dubin Johnson and Rotor syndrome
?Lipoidosis
Suggest acylcarnitine
Check for delta bilirubin on Vitros
?Renal tubular problems
Suggest haptoglobin/lactate
dehydrogenase/plasma viscosity
?Hypopituitarism
Some POCT meters interference by
galactose
?Trisomy 18,21/hereditary syndromes
?Glycogen storage disease
Suggest galactose-free diet
MISLEADING KEY WORDS
?Biliary atresia
Exclude inadequate breastfeeding
?Breast milk jaundice
?Delayed bowel motility
Suggest haemoglobin electrophoresis/
stool exam